bartasevicius.com.brbartasevicius.com.br/sites/default/files/2018-12/... · Sobre este trabalho. E...

207
MATEM ´ ATICA 200 quest˜ oes de concursos resolvidas (1 a Revis˜ ao) Jos´ e Bartasevicius

Transcript of bartasevicius.com.brbartasevicius.com.br/sites/default/files/2018-12/... · Sobre este trabalho. E...

Page 1: bartasevicius.com.brbartasevicius.com.br/sites/default/files/2018-12/... · Sobre este trabalho. E permitido: • Compartilhar - copiar e redistribuir o material em qualquer meio

MATEMATICA200 questoes de concursos resolvidas

(1a Revisao)

Jose Bartasevicius

Page 2: bartasevicius.com.brbartasevicius.com.br/sites/default/files/2018-12/... · Sobre este trabalho. E permitido: • Compartilhar - copiar e redistribuir o material em qualquer meio

Sobre este trabalho.E permitido:

• Compartilhar - copiar e redistribuir o material em qualquer meio ouformato; e

• Adaptar - mesclar, transformar e construir outro material baseadoneste trabalho.

Sob as seguintes condicoes:

• Atribuicao - deve ser dado o devido credito, prover o link ou a licencae indicar quaisquer mudancas realizadas. Isto pode ser feito de qualquermaneira inteligıvel, mas nao de uma maneira que sugira que o autorrecomende voce ou seu uso;

• Nao-Comercial - nao e permitido usar o material para fins comerciais;

• Sem restricoes adicionais - nao e permitido aplicar termos legaisou medidas tecnologicas que efetivamente restrinjam outras pessoas derealizar as atividades permitidas por esta licenca.

AVISO:Esta edicao nao foi exaustivamente revisada e pode conter erros.

Pede-se gentilmente avisar dos erros encontrados ao autor.

jose[dot]bartasevicius[at]gmail[dot]com

br.linkedin.com/in/bartasevicius

@jbartasevicius

/bartasevicius

2 J. Bartasevicius

Page 3: bartasevicius.com.brbartasevicius.com.br/sites/default/files/2018-12/... · Sobre este trabalho. E permitido: • Compartilhar - copiar e redistribuir o material em qualquer meio

Revisoes

1ª Revisao (20/Fev/2015)

1. Correcao da Solucao 4.

2. Troca de 2 por 3 na quarta linha da coluna de k5 da Solucao 14.

3. Troca de 21 por 42 na expressao “todos os multiplos de 21” na Solucao15.

4. Troca de “cosnϕ” por “nϕ” na fase da representacao polar na Solucao18.

5. Troca de aky por ay no desenvolvimento da Solucao 26.

6. Correcao de erro de digitacao na Solucao 54.

7. Correcao de erro de digitacao na Solucao 55.

8. Alteracoes esteticas para melhor visualizacao e acomodamento na folhaA4.

3

Page 4: bartasevicius.com.brbartasevicius.com.br/sites/default/files/2018-12/... · Sobre este trabalho. E permitido: • Compartilhar - copiar e redistribuir o material em qualquer meio

Sumario

1 Problemas 5

2 Solucoes 58

4

Page 5: bartasevicius.com.brbartasevicius.com.br/sites/default/files/2018-12/... · Sobre este trabalho. E permitido: • Compartilhar - copiar e redistribuir o material em qualquer meio

Capıtulo 1

Problemas

Problema 1(IME-1996) Resolva o sistema abaixo:xy = yx

y = ax, onde a 6= 1 e a > 0

Problema 2(IME-1996) Se tan a e tan b sao raızes da equacao x2 + px + q = 0,calcule, em funcao de p e q, o valor simplificado da expressao:y = sin2(a+ b) + p sin(a+ b) cos(a+ b) + q cos2(a+ b).Considere p, q ∈ R com q 6= 1.

Problema 3(IME-1996) Determine o resto da divisao do polinomio (cosϕ+x sinϕ)n

por(x2 + 1

), onde n e um numero natural.

Problema 4(IME-1997) Determine a solucao da equacao trigonometricasinx+

√3 cosx = 1, x ∈ R.

Problema 5(IME-1997) Determine α, β e γ de modo que o polinomio,

αxγ+1 + βxγ + 1, racional inteiro em x, seja divisıvel por (x− 1)2 eque o valor numerico do quociente seja igual a 120 para x = 1.

Problema 6(IME-1997) Uma soma finita de numeros inteiros consecutivos, ımpares,positivos ou negativos, e igual a 73.Determine os termos desta soma.

5

Page 6: bartasevicius.com.brbartasevicius.com.br/sites/default/files/2018-12/... · Sobre este trabalho. E permitido: • Compartilhar - copiar e redistribuir o material em qualquer meio

CAPITULO 1. PROBLEMAS

Problema 7(IME-1998) Determine as raızes de z2 + 2iz + 2− 4i = 0 e localize-asno plano complexo, sendo i =

√−1.

Problema 8(IME-1999) Calcule o determinante:

D =

∣∣∣∣∣∣∣∣∣∣∣∣∣∣

1 1 1 1 1 1 11 3 1 1 1 1 11 1 5 1 1 1 11 1 1 7 1 1 11 1 1 1 9 1 11 1 1 1 1 11 11 1 1 1 1 1 13

∣∣∣∣∣∣∣∣∣∣∣∣∣∣Problema 9

(IME-1999) Determine o polinomio em n, com no maximo 4 termos, querepresenta o somatorio dos quadrados dos n primeiros numeros naturais(

n∑k=1

k2

).

Problema 10(IME-1999) Tres jogadores, cada um com um dado, fizeram lancamentossimultaneos. Essa operacao foi repetida cinquenta vezes. Os dadoscontem tres faces brancas e tres faces pretas. Dessas 50 vezes:

a. em 28 saiu uma face preta para o jogador I;

b. em 25 saiu uma face branca para o jogador II;

c. em 27 saiu uma face branca para o jogador III;

d. em 8 saıram faces pretas para os jogadores I e III e branca para ojogador II;

e. em 7 saıram faces brancas para os jogadores II e III e preta para ojogador I;

f. em 4 saıram faces pretas para os tres jogadores;

g. em 11 saıram faces pretas para os jogadores II e III.

Determine quantas vezes saiu uma face preta para pelo menos umjogador.

6 J. Bartasevicius

Page 7: bartasevicius.com.brbartasevicius.com.br/sites/default/files/2018-12/... · Sobre este trabalho. E permitido: • Compartilhar - copiar e redistribuir o material em qualquer meio

CAPITULO 1. PROBLEMAS

Problema 11(IME-1999) Considere quatro numeros inteiros a, b, c e d. Prove que oproduto:

(a− b)(c− a)(d− a)(d− c)(d− b)(c− b)

e divisıvel por 12.

Problema 12(IME-2000) Sejam a e b numeros reais positivos e diferentes de 1.Dado o sistema abaixo:

ax.b1/y =√ab

2 loga x = log1/b y. log√a b

determine os valores de x e y.

Problema 13(IME-2000) Um comandante de uma companhia convocou voluntariospara a constituicao de 11 patrulhas. Todas elas sao formadas pelomesmo numero de homens. Cada homem participa de exatamente duaspatrulhas. Cada duas patrulhas tem somente um homem em comum.Determine o numero de voluntarios e o de integrantes de uma patrulha.

Problema 14(IME-2000) Prove que para qualquer numero inteiro k, os numeros k ek5 terminam sempre com o mesmo algarismo (algarismo das unidades).

Problema 15(IME-2001) Calcule a soma dos numeros entre 200 e 500 que saomultiplos de 6 ou de 14, mas nao sao simultaneamente multiplos deambos.

Problema 16(IME-2001) Sejam x, y e z numeros reais positivos. Prove que:

x+ y + z

3> 3√x.y.z

Em que condicoes a igualdade se verifica?

Problema 17(IME-2001) Resolva a equacao

√5−√

5− x = x, sabendo que x > 0.

J. Bartasevicius 7

Page 8: bartasevicius.com.brbartasevicius.com.br/sites/default/files/2018-12/... · Sobre este trabalho. E permitido: • Compartilhar - copiar e redistribuir o material em qualquer meio

CAPITULO 1. PROBLEMAS

Problema 18(IME-2002) Seja z um numero complexo de modulo unitario que satisfaza condicao z2n 6= −1, onde n e um numero inteiro positivo.

Demonstre quezn

1 + z2ne um numero real.

Problema 19(IME-2002) Determine todos os valores reais de x que satisfazem aequacao:∣∣∣log

(12x3 − 19x2 + 8x

)∣∣∣ = log(

12x3 − 19x2 + 8x),

onde log(y) e |y| representam, respectivamente, o logaritmo na base 10e o modulo de y.

Problema 20(IME-2002) Demonstre que

3√

20 + 14.√

2 +3√

20− 14.√

2 e um numerointeiro multiplo de quatro.

Problema 21(IME-2003) Considere o polinomio P (x) = x3 + ax+ b de coeficientesreais, com b 6= 0. Sabendo que suas raızes sao reais, demonstre quea < 0.

Problema 22(IME-2003) Demonstre que o numero 11 . . . 1︸ ︷︷ ︸

(n−1)vezes

222 . . .︸ ︷︷ ︸n vezes

5 e um quadrado

perfeito.

Problema 23

(IME-2004) Dada a funcao f(x) =156x + 156−x

2, demonstre que:

f(x+ y) + f(x− y) = 2f(x)f(y)

Problema 24(IME-2005) Seja Dn = det(An), onde

An =

2 −1 0 0 . . . 0 0−1 2 −1 0 . . . 0 00 −1 2 −1 . . . 0 0. . . . . . . . . . . . . . . . . . . . .0 0 0 0 . . . 2 −10 0 0 0 . . . −1 2

n×n

Determine Dn em funcao de n (n ∈ N, n > 1).

8 J. Bartasevicius

Page 9: bartasevicius.com.brbartasevicius.com.br/sites/default/files/2018-12/... · Sobre este trabalho. E permitido: • Compartilhar - copiar e redistribuir o material em qualquer meio

CAPITULO 1. PROBLEMAS

Problema 25(IME-2005) Os angulos de um triangulo estao em progressao aritmeticae um deles e solucao da equacao trigonometrica

(sinx+ cosx)(

sin2 x− sinx cosx+ cos2 x)

= 1

Determine os valores destes angulos (em radianos).

Problema 26(IME-2005) Determine o conjunto solucao S = (x, y) | x ∧ y ∈ Z daequacao

(x+ y)k = xy

sabendo que k e um numero primo.

Problema 27(IME-2006) Considere uma sequencia de triangulos retangulos cuja leide formacao e dada por

aK+1 =2

3aK

bK+1 =4

5bK

onde aK e bK , para K > 1, sao os comprimentos dos catetos do K-esimotriangulo retangulo. Se a1 = 30cm e b1 = 42cm, determine o valor dasoma das areas de todos os triangulos quando K →∞.

Problema 28(IME-2006) Considere o sistema de equacoes dado por

3 log3 α + log9 β = 10

log9 α− 2 log3 β = 10

onde α e β sao numeros reais positivos. Determine o valor de P = αβ.

Problema 29(IME-2006) Resolva a equacao logsinx+cosx (1 + sin 2x) = 2, x ∈

[−π

2,π

2

].

Problema 30(IME-2006) Considere o conjunto formado por m bolas pretas e n bolasbrancas. Determine o numero de sequencias simetricas que podem serformadas utilizando-se todas as m+ n bolas.Observacao: uma sequencia e dita simetrica quando ela possui a mesmaordem de cores ao ser percorrida da direita para a esquerda e da esquerdapara a direita.

J. Bartasevicius 9

Page 10: bartasevicius.com.brbartasevicius.com.br/sites/default/files/2018-12/... · Sobre este trabalho. E permitido: • Compartilhar - copiar e redistribuir o material em qualquer meio

CAPITULO 1. PROBLEMAS

Problema 31(IME-2006) Sejam a, b e c numeros reais nao nulos. Sabendo quea+ b

c=b+ c

a=a+ c

b, determine o valor numerico de

a+ b

c.

Problema 32

(IME-2006) Seja f : N→ R uma funcao tal quen∑k=0

f(k) = 2008n+ 1

n+ 2,

onde N e R sao, respectivamente, o conjunto dos numeros naturais e o

dos numeros reais. Determine o valor numerico de1

f(2006).

Problema 33(IME-2007) Determine o conjunto-solucao da equacao

sin3 x+ cos3 x = 1− sin2 x. cos2 x

Problema 34(IME-2007) Encontre o polinomio P (x) tal que Q(x)+1 = (x−1)3.P (x)e Q(x) + 2 e divisıvel por x4, onde Q(x) e um polinomio do 6º grau.

Problema 35(IME-2007) Determine a expressao da soma a seguir, onde n e um inteiromultiplo de 4.

1 + 2i+ 3i2 + . . .+ (n+ 1) in

Problema 36(IME-2007) Considere todos os pontos de coordenadas (x, y) que per-tencam a circunferencia de equacao x2+y2−6x−6y+14 = 0. Determine

o maior valor possıvel dey

x.

Problema 37(IME-2008) Resolva a seguinte inequacao, para 0 6 x 6 2π:

3 sin2 x+ 2 cos2 x+ 4 sinx−(1 + 4

√2)

sinx cosx+ 4 cosx−(2 + 2

√2)

2 sinx− 2√

2 sinx cosx+ 2 cosx−√

2> 2

Problema 38(IME-2009) Tres dados iguais, honestos e com seis faces numeradas deum a seis sao lancados simultaneamente. Determine a probabilidade deque a soma dos resultados de dois quaisquer deles ser igual ao resultadodo terceiro dado.

10 J. Bartasevicius

Page 11: bartasevicius.com.brbartasevicius.com.br/sites/default/files/2018-12/... · Sobre este trabalho. E permitido: • Compartilhar - copiar e redistribuir o material em qualquer meio

CAPITULO 1. PROBLEMAS

Problema 39(IME-2009) Seja a equacao pn + 144 = q2, onde n e q sao numerosinteiros positivos e p e um numero primo. Determine os possıveis valoresde n, p e q.

Problema 40(IME-2010) Seja x um numero inteiro positivo menor ou igual a 20.000.Sabe-se que 2x − x2 e divisıvel por 7. Determine o numero de possıveisvalores de x.

Problema 41(IME-2011) Os numeros reais positivos x1, x2 e x3 sao raızes da equacao

x3− ax2 = ab− b

2x, sendo b ∈ N (natural), a ∈ R (real) e a 6= 1. Deter-

mine, em funcao de a e b, o valor de[x1x2x3 (x1 + x2 + x3)x

21+x22+x23

]b.

Problema 42(IME-2011) Sejam r e s ∈ Z (inteiro). Prove que (2r + 3s) e multiplode 17 se e somente se (9r + 5s) e multiplo de 17.

Problema 43(IME-2012) Considere log√

b(a)2 = 4, com a e b numeros reais po-

sitivos. Determine o valor de m, numero real, para que a equacaox3 − 18x2 + [logb(ab)

m + 8−m]x − logb(a)2m = 0 tenha tres raızesreais em progressao aritmetica.

Problema 44(IME-2012) Considere a, b e c numeros inteiros e 2 < a < b < c.Determine o(s) valor(es) de x, y e z, que satisfacam o sistema de

equacoes

ax− 2by + 3cz = 2abc

3ax− 4by = −abc−by + cz = 0

xyz = 20132

Problema 45(IME-2012) Considere, Z1 e Z2, complexos que satisfazem a equacaox2 +px+ q = 0, onde p e q sao numeros reais diferentes de zero. Sabe-seque os modulos de Z1 e Z2 sao iguais e que a diferenca entre os seusargumentos vale α, onde α e diferente de zero. Determine o valor de

cos2(α

2

)em funcao de p e q.

J. Bartasevicius 11

Page 12: bartasevicius.com.brbartasevicius.com.br/sites/default/files/2018-12/... · Sobre este trabalho. E permitido: • Compartilhar - copiar e redistribuir o material em qualquer meio

CAPITULO 1. PROBLEMAS

Problema 46(IME-2013) O polinomio P (x) = x5 − 3x4 + 10x3 − 30x2 + 81x− 243possui raızes complexas simetricas e uma raiz com valor igual ao modulodas raızes complexas. Determine todas as raızes do polinomio.

Problema 47(IME-2013) Determine o(s) valor(es) de x, inteiro(s) e positivo(s), quesatisfaz(em) a equacao

x2 =x∑y=1

y−1∑z=0

(y − z)

Problema 48

(IME-2013) Resolva a equacao(logcosx sin2 x

).(logcos2 x sinx

)= 4

Problema 49(IME-2013) Calcular o valor da expressao abaixo

3

√370370 . . . 037︸ ︷︷ ︸

89 algarismos

− 11 . . . 1︸ ︷︷ ︸30 algs “1”

00 . . . 0︸ ︷︷ ︸30 algs “0”

Obs.: algs = algarismos

Problema 50(IME-2013) Um professor da um teste surpresa para uma turma de 9alunos, e diz que o teste pode ser feito sozinho ou em grupos de 2 alunos.De quantas formas a turma pode ser organizar para fazer o teste?(Por exemplo, uma turma de 3 alunos pode ser organizar de 4 formas euma turma de 4 alunos pode se organizar de 10 formas)

Problema 51

(IME-2013) Resolver o sistema de equacoes

√x−√y = log3

y

x2x+2 + 8x = 5.4y

Problema 52(IME-2014) Determine os valores reais de x que satisfazem a inequacao:

4

log3 x2 − 2

+ logx1

9> 1

12 J. Bartasevicius

Page 13: bartasevicius.com.brbartasevicius.com.br/sites/default/files/2018-12/... · Sobre este trabalho. E permitido: • Compartilhar - copiar e redistribuir o material em qualquer meio

CAPITULO 1. PROBLEMAS

Problema 53(IME-2014) Encontre as solucoes reais da equacao:√

x+√

4x− 4 +

√x−√

4x− 4 =√x+ 3

Problema 54(IME-2014) Seja n um inteiro positivo cuja representacao decimal eam . . . a1a0 e f a funcao que troca a posicao dos dıgitos a2i e a2i+1, deforma que f (a2k+1a2k . . . a1a0) = a2ka2k+1 . . . a0a1. Por exemplo:f(123456) = 214365f(1034) = 143f(123) = 1032f(10) = 1Determine o menor numero maior que 99 que satisfaca a equacao

x2 = 9x+ 9f(x) + (f(x))2

Problema 55(IME-2014) Os coeficientes a0, . . . , a2014 do polinomio P (x) = x2015 +a2014x

2014 + . . . + a1x+ a0 sao tais que ai ∈ 0, 1, para 0 6 i 6 2014.

a) Quais sao as possıveis raızes inteiras de P (x)?

b) Quantos polinomios da forma acima tem duas raızes inteiras dis-tintas?

Problema 56(ITA-1990) O conjunto das solucoes reais da equacao| ln(sin2 x)| = ln(sin2 x) e dado por:

a) x ∈ R : x =π

2+ kπ, k ∈ Z

b) x ∈ R : x = π + kπ

2, k ∈ Z

c) x ∈ R : x = 2kπ, k ∈ Zd) x ∈ R : −1 6 x 6 1e) x ∈ R : x > 0

Problema 57(ITA-1990) Sabendo-se que 3x− 1 e fator de 12x3− 19x2 + 8x− 1 entaoas solucoes reais da equacao 12

(33x)− 19

(32x)

+ 8 (3x)− 1 = 0 somam:

a) − log3 12

J. Bartasevicius 13

Page 14: bartasevicius.com.brbartasevicius.com.br/sites/default/files/2018-12/... · Sobre este trabalho. E permitido: • Compartilhar - copiar e redistribuir o material em qualquer meio

CAPITULO 1. PROBLEMAS

b) 1

c) −1

3log3 12

d) −1

e) log3 7

Problema 58(ITA-1990) Numa progressao geometrica de tres termos a razao e e−2a,a soma dos termos e 7 enquanto que a diferenca do ultimo termo com oprimeiro e 3. Nestas condicoes o valor de a e:

a) ln√

2

b) − ln5

2

c) ln√

3

d) − ln√

2

e) nao existe numero real a nestas condicoes

Problema 59(ITA-1990) Sejam as funcoes f e g dadas por:

f : R→ R, f(x) =

1, se |x| < 1

0, se |x| > 1

g : R− 1 → R, g(x) =2x− 3

x− 1Sobre a composta (f g) (x) = f (g (x)) podemos garantir que:

a) se x >3

2, f (g (x)) = 0

b) se 1 < x <3

2, f (g (x)) = 1

c) se4

3< x < 2, f (g (x)) = 1

d) se 1 < x 64

3, f (g (x)) = 1

e) n.d.a.

Problema 60(ITA-1990) Seja C o centro da circunferencia x2 + y2 − 6

√2y = 0.

Considere A e B os pontos de intersecao desta circunferencia com a retay =√

2x. Nestas condicoes o perımetro do triangulo de vertices A, B eC e:

14 J. Bartasevicius

Page 15: bartasevicius.com.brbartasevicius.com.br/sites/default/files/2018-12/... · Sobre este trabalho. E permitido: • Compartilhar - copiar e redistribuir o material em qualquer meio

CAPITULO 1. PROBLEMAS

a) 6√

2 +√

3

b) 4√

3 +√

2

c)√

2 +√

3

d) 5√

3 +√

2

e) n.d.a.

Problema 61(ITA-1991) Sejam a ∈ R, a > 1 e f : R → R definida por f(x) =ax − a−x

2. A funcao inversa de f e dada por:

a) loga

(x−√x2 − 1

), para x > 1

b) loga

(−x+

√x2 + 1

), para x ∈ R

c) loga

(x+√x2 + 1

), para x ∈ R

d) loga

(−x+

√x2 − 1

), para x < −1

e) n.d.a.

Problema 62(ITA-1991) Considere as afirmacoes:

I - A equacao 3x4 − 10x3 + 10x− 3 = 0 so admite raızes reais.

II - Toda equacao recıproca admite um numero par de raızes.

III - As raızes da equacao x3 + 4x2 − 4x − 16 = 0 sao exatamente odobro das raızes de x3 + 2x2 − x− 2 = 0.

Entao:

a) Apenas I e verdadeira

b) Apenas II e falsa

c) Apenas III e verdadeira

d) Todas sao verdadeiras

e) n.d.a.

Problema 63(ITA-1991) Uma escola possui 18 professores sendo 7 de Matematica,3 de Fısica e 4 de Quımica. De quantas maneiras podemos formar

J. Bartasevicius 15

Page 16: bartasevicius.com.brbartasevicius.com.br/sites/default/files/2018-12/... · Sobre este trabalho. E permitido: • Compartilhar - copiar e redistribuir o material em qualquer meio

CAPITULO 1. PROBLEMAS

comissoes de 12 professores de modo que cada uma contenha exatamente5 professores de Matematica, com no mınimo 2 de Fısica e no maximo2 de Quımica?

a) 875

b) 1877

c) 1995

d) 2877

e) n.d.a.

Problema 64

(ITA-1991) Sejam A =n∑k=0

[nk

]3k e B =

n−1∑k=0

[n− 1k

]11k.

Se lnB − lnA = ln6561

4entao n e igual a:

a) 5

b) 6

c) 7

d) 8

e) n.d.a.

Problema 65(ITA-1992) Numa progressao geometrica de razao inteira q > 1. Sabe-seque a1an = 243, logq Pn = 20 e logq an = 6, onde an e o enesimo termoda progressao geometrica e Pn e o produto dos n primeiros termos.Entao a soma dos n primeiros termos e igual a:

a)39 − 1

6

b)310 − 1

6

c)38 − 1

6

d)39 − 1

3

e) n.d.a.

16 J. Bartasevicius

Page 17: bartasevicius.com.brbartasevicius.com.br/sites/default/files/2018-12/... · Sobre este trabalho. E permitido: • Compartilhar - copiar e redistribuir o material em qualquer meio

CAPITULO 1. PROBLEMAS

Problema 66(ITA-1992) No desenvolvimento (x+y)6, ordenado segundo as potencias

decrescentes de x, a soma do 2° termo com1

10do termo de maior

coeficiente e igual a oito vezes a soma de todos os coeficientes. Se

x = (2)z+1 e y =

(1

4

)z−1/2

, entao:

a) z ∈ [0, 1]

b) z ∈ (20, 50)

c) z ∈ (−∞, 0]

d) z ∈ [1, 15]

e) n.d.a.

Problema 67(ITA-1993) Seja a o modulo de numero complexo

(2− 2

√3i)10

. Entaoo valor de x que verifica a igualdade (4a)x = a e:

a)10

11

b) −2

c)5

8

d)3

8

e)1

5

Problema 68

(ITA-1993) Analisando o sistema

3x− 2y + z = 7

x+ y − z = 0

2x+ y − 2z = −1

concluımos que

este e:

a) Possıvel e determinado com xyz = 7.

b) Possıvel e determinado com xyz = −8.

c) Possıvel e determinado com xyz = 6.

d) Possıvel e indeterminado.

e) Impossıvel.

J. Bartasevicius 17

Page 18: bartasevicius.com.brbartasevicius.com.br/sites/default/files/2018-12/... · Sobre este trabalho. E permitido: • Compartilhar - copiar e redistribuir o material em qualquer meio

CAPITULO 1. PROBLEMAS

Problema 69(ITA-1993) Um acidente de carro foi presenciado por 1/65 da populacaode Votuporanga (SP). O numero de pessoas que soube do acontecimento

t horas apos e dado por: f(t) =B

1 + Ce−ktonde B e a populacao da

cidade. Sabendo-se que 1/9 da populacao soube do acidente 3 horasapos, entao o tempo passou ate que 1/5 da populacao soubesse danotıcia foi de:

a) 4 horas

b) 5 horas

c) 6 horas

d) 5 horas e 24 min

e) 5 horas e 30 min

Problema 70(ITA-1993) Uma das circunferencias que passa pelo ponto P : (0, 0) etangencia as retas (r1) : x−y = 0 e (r2) : x+y−2 = 0 tem sua equacaodada por:

a) (x− 1)2 + (y + 1)2 =√

2

b) (x− 1)2 + (y + 1)2 = 2

c) (x− 1)2 + (y − 1)2 = 2

d) (x+ 1)2 + (y − 1)2 =√

2

e) (x+ 1)2 + (y + 1)2 = 2

Problema 71

(ITA-1994) A identidade:x3 + 4

x3 + 1= 1 +

a

x+ 1+

bx+ c

x2 − x+ 1e valida

para todo real x 6= − 1. Entao a+ b+ c e igual a:

a) 5

b) 4

c) 3

d) 2

e) 1

18 J. Bartasevicius

Page 19: bartasevicius.com.brbartasevicius.com.br/sites/default/files/2018-12/... · Sobre este trabalho. E permitido: • Compartilhar - copiar e redistribuir o material em qualquer meio

CAPITULO 1. PROBLEMAS

Problema 72

(ITA-1994) No desenvolvimento de A =

(3a2

2+

2m

3

)10

, a razao entre a

parcela contendo o fator a16m2 e a parcela contendo o fator a14m3 e igual

a 9/16. Se a e m sao numeros reais positivos tais que A =(m2 + 4

)5entao:

a) a.m = 2/3

b) a.m = 1/3

c) a+m = 5/2

d) a+m = 5

e) a−m = 5/2

Problema 73(ITA-1994) Seja (a1, a2, . . . , an) uma progressao geometrica com umnumero ımpar de termos e razao q > 0. O produto de seus termos eigual a 225 e o termo do meio e 25. Se a soma dos (n − 1) primeirostermos e igual a 2(1 + q)

(1 + q2

), entao:

a) a1 + q = 16

b) a1 + q = 12

c) a1 + q = 10

d) a1 + q + n = 20

e) a1 + q + n = 11

Problema 74(ITA-1994) Seja (a, b, c, d, e) uma progressao geometrica de razao a, coma 6= 0 e a 6= 1. Se a soma de seus termos e igual a (13a+ 12) e x e umnumero real positivo diferente de 1 tal que:

1

loga x+

1

logb x+

1

logc x+

1

logd x+

1

loge x=

5

2

a) 33

b) 23

c) (5/2)2

d) (5/2)3/2

e) (2/5)2

J. Bartasevicius 19

Page 20: bartasevicius.com.brbartasevicius.com.br/sites/default/files/2018-12/... · Sobre este trabalho. E permitido: • Compartilhar - copiar e redistribuir o material em qualquer meio

CAPITULO 1. PROBLEMAS

Problema 75(ITA-1994) Sejam x e y numeros reais, positivos e ambos diferentes de 1,

satisfazendo o sistema:

xy =

1

y2

log x+ log y = log1√x

. Entao o conjunto

(x, y) esta contido no intervalo:

a) [2, 5]

b) ]0, 4[

c) [−1, 2]

d) [4, 8[

e) [5,∞[

Problema 76(ITA-1995) Sabendo que 4 + i

√2 e√

5 sao raızes do polinomio 2x5 −22x4 + 74x3 + 2x2 − 420x+ 540, entao a soma dos quadrados de todasas raızes reais e:

a) 17

b) 19

c) 21

d) 23

e) 25

Problema 77(ITA-1995) Sejam z1 e z2 numeros complexos com |z1| = |z2| = 4. Se1 e uma raiz da equacao z1z

6 + z2z3 − 8 = 0 entao a soma das raızes

reais e igual a:

a) −1

b) −1 + 21/2

c) 1− 21/3

d) 1 + 31/2

e) −1 + 31/2

Problema 78(ITA-1996) Considere o polinomio:P (z) = z6 + 2z5 + 6z4 + 12z3 + 8z2 + 16z

20 J. Bartasevicius

Page 21: bartasevicius.com.brbartasevicius.com.br/sites/default/files/2018-12/... · Sobre este trabalho. E permitido: • Compartilhar - copiar e redistribuir o material em qualquer meio

CAPITULO 1. PROBLEMAS

a) Apenas uma e real.

b) Apenas duas raızes sao reais e distintas.

c) Apenas duas raızes sao reais e iguais.

d) Quatro raızes sao reais, sendo duas a duas distintas.

e) Quatro raızes sao reais, sendo apenas duas iguais.

Problema 79

(ITA-1996) O valor da potencia

( √2

1 + i

)93

e:

a)−1 + i√

2

b)1 + i√

2

c)−1− i√

2

d)(√

2)93

i

e)(√

2)93

+ i

Problema 80(ITA-1996) Sejam a1, a2, a3 e a4 quatro numeros reais (com a1 6= 0),formando nessa ordem uma progressao geometrica.

Entao, o sistema em x e y

a1x+ a3y = 1

a1a2x+ a1a4y = a2e um sistema:

a) Impossıvel.

b) Possıvel e determinado.

c) Possıvel e indeterminado.

d) Possıvel determinado para a1 > 1.

e) Possıvel determinado para a1 < −1.

Problema 81(ITA-1997) Os numeros reais x, y e z formam, nesta ordem, umaprogressao aritmetica de razao r. Seja α um numero real com α > 0 eα 6= 1 satisfazendo 3ax + 2ay − az = 0. Entao r e igual a

a) a2

J. Bartasevicius 21

Page 22: bartasevicius.com.brbartasevicius.com.br/sites/default/files/2018-12/... · Sobre este trabalho. E permitido: • Compartilhar - copiar e redistribuir o material em qualquer meio

CAPITULO 1. PROBLEMAS

b)

(1

2

)ac) log2a 4

d) loga(3/2)

e) loga 3

Problema 82(ITA-1998) Sejam x e y numeros reais tais que:

x3 − 3xy2 = 1

3x2y − y3 = 1

Entao, o numero complexo z = x+ iy e tal que z3 e |z|, valem respecti-vamente:

a) 1− i e 6√

2

b) 1 + i e 6√

2

c) i e 1

d) −i e 1

e) 1 + i e 3√

2

Problema 83(ITA-1998) O valor de y ∈ R que satisfaz a igualdade:logy 49 = logy2 7 + log2y 7, e:

a)1

2

b)1

3c) 3

d)1

8e) 7

Problema 84(ITA-1998) O numero de anagramas da palavra VESTIBULANDO, quenao apresentam as cinco vogais juntas, e:

a) 12!

b) (8)! (5)!

22 J. Bartasevicius

Page 23: bartasevicius.com.brbartasevicius.com.br/sites/default/files/2018-12/... · Sobre este trabalho. E permitido: • Compartilhar - copiar e redistribuir o material em qualquer meio

CAPITULO 1. PROBLEMAS

c) 12!− (8)! (5)!

d) 12!− 8!

e) 12!− (7)! (5)!

Problema 85(ITA-1999) Seja a ∈ R com a > 1. O conjunto de todas as solucoes

reais da inequacao a2x(1−x) > ax−1, e:

a) ]− 1, 1[

b) ]1,+∞[

c) ]− 1/2, 1[

d) ]−∞, 1[

e) vazio

Problema 86

(ITA-1999) Se x ∈ [0, π/2[ e tal que 4 tan4 x =1

cos4 x+ 4, entao o valor

de sin 2x+ sin 4x

a)

√15

4

b)

√15

8

c)3√

5

8

d)1

2

e) 1

Problema 87(ITA-1999) Considere a circunferencia C de equacao x2+y2+2x+2y+1 =0 e a elipse E de equacao x2 + 4y2 − 4x+ 8y + 4 = 0. Entao:

a) C e E interceptam-se em dois pontos distintos.

b) C e E interceptam-se em quatro pontos distintos.

c) C e E sao tangentes exteriormente.

d) C e E sao tangentes interiormente.

e) C e E tem o mesmo centro e nao se interceptam.

J. Bartasevicius 23

Page 24: bartasevicius.com.brbartasevicius.com.br/sites/default/files/2018-12/... · Sobre este trabalho. E permitido: • Compartilhar - copiar e redistribuir o material em qualquer meio

CAPITULO 1. PROBLEMAS

Problema 88(ITA-2000) Quantos numeros de seis algarismos distintos podemosformar usando os dıgitos 1,2,3,4,5 e 6, nos quais o 1 e o 2 nunca ocupamposicoes adjacentes, mas o 3 e o 4 sempre ocupam posicoes adjacentes?

a) 144

b) 180

c) 240

d) 288

e) 360

Problema 89(ITA-2000) A soma das raızes reais e positivas da equacao

4x2 − 5.2x

2+ 4 = 0 vale:

a) 2

b) 5

c)√

2

d) 1

e)√

3

Problema 90(ITA-2000) O valor de n que torna a sequencia (2 + 3n,−5n, 1 − 4n)uma progressao aritmetica pertence ao intervalo:

a) [−2,−1]

b) [−1, 0]

c) [0, 1]

d) [1, 2]

e) [2, 3]

Problema 91(ITA-2001) Sabendo que e de 1024 a soma dos coeficientes do polinomioem x e y, obtido pelo desenvolvimento do binomio (x+ y)m, temos queo numero de arranjos sem repeticao de m elementos, tomados 2 a 2, e:

a) 80

b) 90

24 J. Bartasevicius

Page 25: bartasevicius.com.brbartasevicius.com.br/sites/default/files/2018-12/... · Sobre este trabalho. E permitido: • Compartilhar - copiar e redistribuir o material em qualquer meio

CAPITULO 1. PROBLEMAS

c) 70

d) 100

e) 60

Problema 92(ITA-2001) O conjunto de todos os valores de m para os quais a funcao

f(x) =x2 + (2m+ 3)x+

(m2 + 3

)√x2 + (2m+ 1)x+

(m2 + 2

) esta definida e e nao negativa

para todo x real e:

a) [1/4, 7/4[

b) ]1/4,∞[

c) ]0, 7/4[

d) ]−∞, 1/4]

e) ]1/4, 7/4[

Problema 93(ITA-2001) A parte imaginaria de ((1 + cos 2x) + i sin 2x)k, k inteiropositivo, x real e

a) 2 sink x. cosk x

b) sink x. cosk x

c) 2k sin kx. cosk x

d) 2k sink x. cosk x

e) sin kx. cosk x

Problema 94(ITA-2002) Os valores de x ∈ R, para os quais a funcao real dada porf(x) =

√5− ||2x− 1| − 6| esta definida, formam o conjunto

a) [0, 1].

b) [−5, 6].

c) [−5, 0] ∪ [1,∞).

d) (−∞, 0] ∪ [1, 6].

e) [−5, 0] ∪ [1, 6].

J. Bartasevicius 25

Page 26: bartasevicius.com.brbartasevicius.com.br/sites/default/files/2018-12/... · Sobre este trabalho. E permitido: • Compartilhar - copiar e redistribuir o material em qualquer meio

CAPITULO 1. PROBLEMAS

Problema 95(ITA-2002) Sejam f e g duas funcoes definidas por

f(x) =(√

2)3 sinx−1

e g(x) =

(1

2

)3 sin2 x−1

, x ∈ R

A soma do valor mınimo de f com o valor mınimo de g e igual a

a) 0.

b) −1

4.

c)1

4.

d)1

2.

e) 1.

Problema 96(ITA-2002) Sabendo que a equacao

x3 − px2 = qm, p, q 6= 1,m ∈ N

possui tres raızes reais positivas a, b e c, entao

logq

[abc(a2 + b2 + c2

)a+b+c]

e igual a

a) 2m+ p logq p

b) m+ 2p logq p

c) m+ p logq p

d) m− p logq p

e) m− 2p logq p

Problema 97(ITA-2002) Mostre que (

x

y+ 2 +

y

x

)4

> C8,4

para quaisquer x e y reais positivos.Obs.: Cn,p denota a combinacao de n elementos tomados p a p.

26 J. Bartasevicius

Page 27: bartasevicius.com.brbartasevicius.com.br/sites/default/files/2018-12/... · Sobre este trabalho. E permitido: • Compartilhar - copiar e redistribuir o material em qualquer meio

CAPITULO 1. PROBLEMAS

Problema 98(ITA-2003) O numero de divisores de 17640 que, por sua vez, saodivisıveis por 3 e:

a) 24

b) 36

c) 48

d) 54

e) 72

Problema 99(ITA-2003) Considere a seguinte situacao baseada num dos paradoxosde Zenao de Eleia, filosofo grego do seculo V A.C. Suponha que oatleta Aquiles e uma tartaruga apostam uma corrida em linha reta,correndo com velocidades constantes vA e vT , com 0 < vT < vA. Comoa tartaruga e mais lenta, e-lhe dada uma vantagem inicial, de modoa comecar a corrida no instante t = 0 a uma distancia d1 > 0 nafrente de Aquiles. Calcule os tempos t1, t2, t3, . . . que Aquiles precisapara percorrer as distancias d1, d2, d3, . . ., respectivamente, sendo que,para todo n > 2, dn denota a distancia entre a tartaruga e Aquiles no

instanten−1∑k=1

tk da corrida. Verifique que os termos tk, k = 1, 2, 3, . . .,

formam uma progressao geometrica infinita, determine sua soma e de osignificado desta soma.

Problema 100(ITA-2004) O termo independente de x no desenvolvimento do binomio(√

3 3√x

5x− 3

√5x

3√x

)12

a) 729 3√

45

b) 972 3√

15

c) 891 3

√3

5

d) 376 3

√5

3

e) 165 3√

75

J. Bartasevicius 27

Page 28: bartasevicius.com.brbartasevicius.com.br/sites/default/files/2018-12/... · Sobre este trabalho. E permitido: • Compartilhar - copiar e redistribuir o material em qualquer meio

CAPITULO 1. PROBLEMAS

Problema 101(ITA-2004) Uma caixa branca contem 5 bolas verdes e 3 azuis, e umacaixa preta contem 3 bolas verdes e 2 azuis. Pretende-se retirar umabola de uma das caixas. Para tanto, 2 dados sao atirados. Se a somaresultante dos dois dados for menor que 4, retira-se uma bola da caixabranca. Nos demais casos, retira-se uma bola da caixa preta. Qual e aprobabilidade de se retirar uma bola verde?

Problema 102(ITA-2004) Considere a equacao x3 + 3x2− 2x+ d = 0, em que d e umaconstante real. Para qual valor de d a equacao admite uma raiz duplano intervalo ]0, 1[?

Problema 103(ITA-2005) Sobre o numero x =

√7− 4

√3 +√

3 e correto afirmar que

a) x ∈]0, 2[

b) x e racional

c)√

2x e irracional

d) x2 e irracional

e) x ∈]2, 3[

Problema 104(ITA-2005) No desenvolvimento de (ax2 − 2bx+ c+ 1)5 obtem-se umpolinomio p(x) cujos coeficientes somam 32. Se 0 e −1 sao raızes dep(x), entao a soma a+ b+ c e igual a

a) −1

2

b) −1

4

c)1

2d) 1

e)3

2

Problema 105(ITA-2005) O menor inteiro positivo n para o qual a diferenca√n−√n− 1 fica menor que 0, 01 e

a) 2499

28 J. Bartasevicius

Page 29: bartasevicius.com.brbartasevicius.com.br/sites/default/files/2018-12/... · Sobre este trabalho. E permitido: • Compartilhar - copiar e redistribuir o material em qualquer meio

CAPITULO 1. PROBLEMAS

b) 2501

c) 2500

d) 3600

e) 4900

Problema 106(ITA-2005) (a) Mostre que o numero real α =

3√

2 +√

5 +3√

2−√

5 eraiz da equacao x3 + 3x− 4 = 0.(b) Conclua de (a) que α e um numero racional.

Problema 107(ITA-2005) Considere a equacao em x ∈ R

√1 +mx = x+

√1−mx

sendo m um parametro real.(a) Resolva a equacao em funcao do parametro m.(b) Determine todos os valores de m para os quais a equacao admitesolucao nao nula.

Problema 108

(ITA-2006) Considere a equacao(ax − a−x)

(ax + a−x)= m, na variavel real x,

com 0 < a 6= 1. O conjunto de todos os valores de m para os quais estaequacao admite solucao real e

a) (−1, 0) ∪ (0, 1)

b) (−∞,−1) ∪ (1,+∞)

c) (−1, 1)

d) (0,∞)

e) (−∞,+∞)

Problema 109(ITA-2006) Considere uma prova com 10 questoes de multipla escolha,cada questao com 5 alternativas. Sabendo que cada questao admiteuma unica alternativa correta, entao o numero de formas possıveis paraque um candidato acerte somente 7 das 10 questoes e

a) 44.30

b) 43.60

c) 53.60

J. Bartasevicius 29

Page 30: bartasevicius.com.brbartasevicius.com.br/sites/default/files/2018-12/... · Sobre este trabalho. E permitido: • Compartilhar - copiar e redistribuir o material em qualquer meio

CAPITULO 1. PROBLEMAS

d)

(73

).43

e)

(107

)Problema 110

(ITA-2006) Considere as seguintes afirmacoes sobre a expressao

S =101∑k=0

log8

(4k√

2)

:

I. S e a soma dos termos de uma progressao geometrica finita

II. S e a soma dos termos de uma progressao aritmetica finita derazao 2/3

III. S = 3451

IV. S 6 3434 + log8

√2

Entao, pode-se afirmar que e (sao) verdadeira(s) apenas

a) I e III

b) II e III

c) II e IV

d) II

e) III

Problema 111(ITA-2006) Seja o sistema linear nas incognitas x e y, com a e b reais,dado por

(a− b)x− (a+ b) y = 1

(a+ b)x+ (a− b) y = 1

Considere as seguintes afirmacoes:

1. O sistema e possıvel e indeterminado se a = b = 0

2. O sistema e possıvel e determinado se a e b nao sao simultaneamentenulos

3. x2 + y2 =(a2 + b2

)−1, se a2 + b2 6= 0

Entao pode-se afirmar que e(sao) verdadeira(s) apenas

a) I

30 J. Bartasevicius

Page 31: bartasevicius.com.brbartasevicius.com.br/sites/default/files/2018-12/... · Sobre este trabalho. E permitido: • Compartilhar - copiar e redistribuir o material em qualquer meio

CAPITULO 1. PROBLEMAS

b) II

c) III

d) I e II

e) II e III

Problema 112(ITA-2006) Determine o coeficiente de x4 no desenvolvimento de

(1 + x+ x2

)9.

Problema 113(ITA-2007) Seja A um conjunto com 14 elementos e B um subconjuntode A com 6 elementos. O numero de subconjuntos de A com um numerode elementos menor ou igual a 6 e disjuntos de B e

a) 28 − 9

b) 28 − 1

c) 28 − 26

d) 214 − 28

e) 28

Problema 114(ITA-2007) Sejam x, y e z numeros reais positivos tais que seus logarit-mos numa dada base k sao numeros primos satisfazendo

logk(xy) = 49,

logk(x/z) = 44.

Entao, logk(xyz) e igual a

a) 52

b) 61

c) 67

d) 80

e) 97

Problema 115(ITA-2007) Sejam x e y dois numeros reais tais que ex, ey e o quociente

ex − 2√

5

4− ey√

5

sao todos racionais. A soma x+ y e igual a

J. Bartasevicius 31

Page 32: bartasevicius.com.brbartasevicius.com.br/sites/default/files/2018-12/... · Sobre este trabalho. E permitido: • Compartilhar - copiar e redistribuir o material em qualquer meio

CAPITULO 1. PROBLEMAS

a) 0

b) 1

c) 2 log5 3

d) log5 2

e) 3 loge 2

Problema 116(ITA-2007) Sendo x, y, z e w numeros reais, encontre o conjunto solucaodo sistema

log[(x+ 2y) (w − 3z)−1

]= 0,

2x+3z − 8.2y−3z+w = 0,

3√

2x+ y + 6z − 2w − 2 = 0.

Problema 117(ITA-2008) Para x ∈ R, o conjunto-solucao de∣∣53x − 52x+1 + 4.5x

∣∣ = |5x − 1| e

a)

0, 2±√

5, 2±√

3

b)

0, 1, log5

(2 +√

5)

c)

0,

1

2log5 2,

1

2log5 3, log5

(√2

2

)d)

0, log5

(2 +√

5), log5

(2 +√

3), log5

(2−√

3)

e) A unica solucao e x = 0

Problema 118(ITA-2009) Uma empresa possui 1000 carros, sendo uma parte commotor a gasolina e o restante com motor “flex” (que funciona comalcool e com gasolina). Numa determinada epoca, neste conjunto de1000 carros, 36% dos carros com motor a gasolina e 36% dos carroscom motor “flex” sofrem conversao para tambem funcionar com gasGNV. Sabendo-se que, apos esta conversao, 556 dos 1000 carros destaempresa sao bicombustıveis, pode-se afirmar que o numero de carrostricombustıveis e igual a

a) 246

b) 252

c) 260

32 J. Bartasevicius

Page 33: bartasevicius.com.brbartasevicius.com.br/sites/default/files/2018-12/... · Sobre este trabalho. E permitido: • Compartilhar - copiar e redistribuir o material em qualquer meio

CAPITULO 1. PROBLEMAS

d) 268

e) 284

Problema 119(ITA-2009) Se as solucoes da equacao algebrica 2x3− ax2 + bx+ 54 = 0com coeficientes a, b ∈ R, b 6= 0, formam, numa determinada ordem,

uma progressao geometrica, entao,a

be igual a

a) −3

b) −1

3

c)1

3d) 1

e) 3

Problema 120(ITA-2009) A expressao

2

[sin

(x+

11

)+ cotan2 x

]tan

x

2

1 + tan2 x

2

e equivalente a

a)[cosx− sin2 x

]cotanx

b) [sinx+ cosx] tanx

c)[cos2 x− sinx

]cotan2 x

d)[1− cotan2 x

]sinx

e)[1 + cotan2 x

][sinx+ cosx]

Problema 121(ITA-2009) Suponha que a equacao algebrica

x11 +10∑n=1

anxn + a0 = 0

tenha coeficientes reais a0, a1, . . . , a10 tais que as suas onze raızes sejamtodas simples e da forma β + iγn, em que β, γn ∈ R formam umaprogressao aritmetica de razao real γ 6= 0. Considere as tres afirmacoesabaixo e responda se cada uma delas e, respectivamente, verdadeira oufalsa, justificando sua resposta:

J. Bartasevicius 33

Page 34: bartasevicius.com.brbartasevicius.com.br/sites/default/files/2018-12/... · Sobre este trabalho. E permitido: • Compartilhar - copiar e redistribuir o material em qualquer meio

CAPITULO 1. PROBLEMAS

I. Se β = 0, entao a0 = 0

II. Se a10 = 0, entao β0 = 0

III. Se β = 0, entao a1 = 0

Problema 122(ITA-2010) A expressao

(2√

3 +√

5)5 − (2√3−

√5)5

e igual a

a) 2630√

5

b) 2690√

5

c) 2712√

5

d) 1584√

15

e) 1604√

15

Problema 123(ITA-2010) Um palco possui 6 refletores de iluminacao. Num certoinstante de um espetaculo moderno os refletores sao acionados ale-

atoriamente de modo que, para cada um dos refletores, seja de2

3a

probabilidade de ser aceso: Entao, a probabilidade de que, neste instante,4 ou 5 refletores sejam acesos simultaneamente, e igual a

a)16

27

b)49

81

c)151

243

d)479

729

e)24

34+

25

35

Problema 124

(ITA-2010) Analise a funcao f : R→ R, f(x) =3x − 3−x

2e bijetora e,

em caso afirmativo, determine a funcao inversa f−1.

Problema 125(ITA-2010) Uma urna de sorteio contem 90 bolas numeradas de 1 a 90,sendo que a retirada de uma bola e equiprovavel a retirada de cadauma das demais.

34 J. Bartasevicius

Page 35: bartasevicius.com.brbartasevicius.com.br/sites/default/files/2018-12/... · Sobre este trabalho. E permitido: • Compartilhar - copiar e redistribuir o material em qualquer meio

CAPITULO 1. PROBLEMAS

a) Retira-se aleatoriamente uma das 90 bolas desta urna. Calcule aprobabilidade de o numero desta bola ser um multiplo de 5 ou de6.

b) Retira-se aleatoriamente uma das 90 bolas desta urna e, sem repo-la, retira-se uma segunda bola. Calcule a probabilidade de onumero da segunda bola retirada nao ser um multiplo de 6.

Problema 126(ITA-2011) O sistema

x+ 2y + 3z = a

y + 2z = b

3x− y − 5cz = 0

a) e possıvel ∀a, b, c ∈ R

b) e possıvel quando a =7b

3ou c 6= 1

c) e impossıvel quando c = 1,∀a, b ∈ R

d) e impossıvel quando a 6= 7b

3∀c ∈ R

e) e possıvel quando c = 1 e a 6= 7b

3

Problema 127(ITA-2011) A expressao 4e2x + 9e2y − 16ex − 54ey + 61 = 0, com x e yreais, representa

a) o conjunto vazio

b) um conjunto unitario

c) um conjunto nao-unitario com um numero finito de pontos

d) um conjunto com um numero infinito de pontos

e) o conjunto

(x, y) ∈ R2 | 2 (ex − 2)2 + 3 (ey − 3)2 = 1

Problema 128

(ITA-2011) Resolva a inequacao em R: 16 <

(1

4

)log 15(x2−x+19)

.

Problema 129(ITA-2011) Determine todos os valores de m ∈ R tais que a equacao(2−m)x2 + 2mx+m+ 2 = 0 tenha duas raızes reais distintas e maioresque zero.

J. Bartasevicius 35

Page 36: bartasevicius.com.brbartasevicius.com.br/sites/default/files/2018-12/... · Sobre este trabalho. E permitido: • Compartilhar - copiar e redistribuir o material em qualquer meio

CAPITULO 1. PROBLEMAS

Problema 130(ITA-2012) Deseja-se trocar uma moeda de 25 centavos, usando-seapenas moedas de 1, 5 e 10 centavos. Entao, o numero de diferentesmaneiras em que a moeda de 25 centavos pode ser trocada e igual a

a) 6

b) 8

c) 10

d) 12

e) 14

Problema 131(ITA-2012) Dois atiradores acertam o alvo uma vez a cada tres disparos.Se os dois atiradores disparam simultaneamente, entao a probabilidadedo alvo ser atingido pelo menos uma vez e igual a

a) 29

b) 13

c) 49

d) 59

e) 23

Problema 132

(ITA-2012) Analise se f : R→ R, f(x) =

3 + x2, x > 0

3− x2, x < 0e bijetora e,

em caso afirmativo, encontre f−1 : R→ R.

Problema 133(ITA-2012) Determine os valores de θ ∈ [0, 2π] tais que logtan(θ) e

sin(θ) >0.

Problema 134(ITA-2013) A soma das raızes da equacao em C, z8 − 17z4 + 16 = 0,tais que z − |z| = 0, e

a) 1

b) 2

c) 3

d) 4

36 J. Bartasevicius

Page 37: bartasevicius.com.brbartasevicius.com.br/sites/default/files/2018-12/... · Sobre este trabalho. E permitido: • Compartilhar - copiar e redistribuir o material em qualquer meio

CAPITULO 1. PROBLEMAS

e) 5

Problema 135(ITA-2013) A soma de todos os numeros reais x que satisfazem a equacao

8√x+1 + 44

(2√x+1)

+ 64 = 19(

4√x+1)

e igual a

a) 8

b) 12

c) 16

d) 18

e) 20

Problema 136(ITA-2013) Se os numeros reais a e b satisfazem, simultaneamente, asequacoes √

a√b =

1

2e ln(a2 + b) + ln 8 = ln 5,

um possıvel valor de ab e

a)√

22

b) 1

c)√

2

d) 2

e) 3√

2

Problema 137

(ITA-2014) Se z =

(1 +√

3i

1−√

3i

)10

, entao o valor de 2 arcsin(Re(z)) +

5 arctan(2Im(z)) e igual a

a)5π

7

b)4π

5

c)3π

2

J. Bartasevicius 37

Page 38: bartasevicius.com.brbartasevicius.com.br/sites/default/files/2018-12/... · Sobre este trabalho. E permitido: • Compartilhar - copiar e redistribuir o material em qualquer meio

CAPITULO 1. PROBLEMAS

d)8π

3

e)9π

4

Problema 138

(ITA-2014) Seja p o polinomio dado por p(x) =15∑j=0

ajxj , com aj ∈

R, j = 0, 1, . . . , 15, e a15 6= 0. Sabendo-se que i e uma raiz de p e quep(2) = 1, entao o resto da divisao de p pelo polinomio q, dado porq(x) = x3 − 2x2 + x− 2, e igual a

a)1

5x2 − 1

5

b)1

5x2 +

1

5

c)2

5x2 +

2

5

d)3

5x2 − 3

5

e)3

5x2 +

1

5

Problema 139

(ITA-2014) Seja n um inteiro positivo tal que sinπ

2n=

√2−√

3

4.

a) Determine n.

b) Determine sinπ

24.

Problema 140(ITA-2014) Sabe-se que a equacao 3x2 + 5xy − 2y2 − 3x+ 8y − 6 = 0representa a reuniao de duas retas concorrentes, r e s, formando umangulo agudo θ. Determine a tangente de θ.

Problema 141(EN-2008) Consideremos a, x ∈ R∗+, x 6= 1 e a 6= 1. Denotemos por log xe loga x, os logaritmos nas bases 10 e a respectivamente. O produto das

raızes reais da equacao 2[1 + logx2(10)

]=

[1

log(x(−1))

]2

e

a) 10√

10

b)√

10

38 J. Bartasevicius

Page 39: bartasevicius.com.brbartasevicius.com.br/sites/default/files/2018-12/... · Sobre este trabalho. E permitido: • Compartilhar - copiar e redistribuir o material em qualquer meio

CAPITULO 1. PROBLEMAS

c)√

1010

d)√

10100

e) 100

Problema 142(EN-2008) Seja n o menor inteiro pertencente ao domınio da funcao

real de variavel real f(x) = ln 3

√ex+1

(2764)−(34)(x+1) . Podemos afirmar que

logn 3

√3√

3√

3 . . . e a raiz da equacao

a) x3 − 2x2 − 9 = 0

b) x3 + x− 1 = 0

c) x4 − 4x2 − x+ 2 = 0

d) x2 − 4x+ 3 = 0

e) x4 − 4x2 + x+ 1 = 0

Problema 143(EN-2008) O sistema linear

x+ 2y − 3z = 4

3x− y + 5z = 2

4x+ y + (a2 − 14)z = a+ 2

onde a ∈ R, pode ser impossıvel e tambem possıvel e indeterminado.Os valores de a que verificam a afirmacao anterior sao, respectivamente

a) 4 e −4

b) −4 e 4

c) 24 e −24

d) −24 e 24

e)√

12 e 12

Problema 144(EN-2009) Ao escrevermos x2

x4+1= Ax+B

a1x2+b1x+c1+ Cx+D

a2x2+b2x+c2onde

ai, bi, ci(1 6 i 6 2) e A,B,C e D sao constantes reais, podemos afirmarque A2 + C2 vale

a) 38

J. Bartasevicius 39

Page 40: bartasevicius.com.brbartasevicius.com.br/sites/default/files/2018-12/... · Sobre este trabalho. E permitido: • Compartilhar - copiar e redistribuir o material em qualquer meio

CAPITULO 1. PROBLEMAS

b) 12

c) 14

d) 18

e) 0

Problema 145(EN-2009) No sistema decimal, a quantidade de numeros ımpares posi-tivos menores que 1000, com todos os algarismos distintos e

a) 360

b) 365

c) 405

d) 454

e) 500

Problema 146(EN-2009) Considere x1, x2 e x3 ∈ R raızes da equacao 64x3 − 56x2 +14x− 1 = 0. Sabendo que x1, x2 e x3 sao termos consecutivos de umaPG e estao em ordem decrescente, podemos afirmar que o valor daexpressao sin [(x1 + x2)π] + tan [(4x1x3)π] vale

a) 0

b)√

22

c) 2−√

22

d) 1

e) 2+√

22

Problema 147(EN-2010) Considere a equacao x2 + bx + c = 0, onde c representa aquantidade de valores inteiros que satisfazem a inequacao |3x− 4| 6 2.Escolhendo-se o numero b, ao acaso, no conjunto −4, −3, −2, −1, 0,1, 2, 3, 4, 5, qual e a probabilidade da equacao acima ter raızes reais?

a) 0, 50

b) 0, 70

c) 0, 75

d) 0, 80

40 J. Bartasevicius

Page 41: bartasevicius.com.brbartasevicius.com.br/sites/default/files/2018-12/... · Sobre este trabalho. E permitido: • Compartilhar - copiar e redistribuir o material em qualquer meio

CAPITULO 1. PROBLEMAS

e) 1

Problema 148(EN-2010) A inequacao x2 − 6x 6 − x2 + px+ c tem como solucao ointervalo [0, 2], onde p, c ∈ R. Seja q a maior raiz da equacao

4|x+1| = 16.2|x+1| − 64. A representacao trigonometrica do numerocomplexo p+ iq e

a) 2√

3

(cos

3+ i

3

)b) 2√

2

(cos

4+ i

4

)c)√

2(

cosπ

6+ i

π

6

)d) 2√

3(

cosπ

3+ i

π

3

)e) 2√

2

(cos

4+ i

4

)Problema 149

(EN-2010) Uma progressao geometrica infinita tem o 4° termo igual a5. O logaritmo na base 5 do produto de seus 10 primeiros termos vale10− 15 log5 2. Se S e a soma desta progressao, entao o valor de log2 S e

a) 2 + 3 log2 5

b) 2 + log2 5

c) 4 + log2 5

d) 1 + log2 5

e) 4 + 2 log2 5

Problema 150(EN-2010) Sejam a, b, c as raızes da equacao 12x3 − 4x2 − 3x+ 1 = 0.

Qual o valor de√a3 + b3 + c3 + 1?

a)2√

21

9

b)2√

7

3

c)2√

7

9

J. Bartasevicius 41

Page 42: bartasevicius.com.brbartasevicius.com.br/sites/default/files/2018-12/... · Sobre este trabalho. E permitido: • Compartilhar - copiar e redistribuir o material em qualquer meio

CAPITULO 1. PROBLEMAS

d)

√21

9

e)

√21

3

Problema 151(EN-2010) Considere f uma funcao definida no conjunto dos numerosnaturais tal que f(n+ 2) = 3 + f(n),∀n ∈ N, f(0) = 10 e f(1) = 5.Qual o valor de

√f(81)− f(70)

a) 2√

2

b)√

10

c) 2√

3

d)√

15

e) 3√

2

Problema 152(EN-2011) Em que ponto da curva y2 = 2x3 a reta tangente e perpendi-cular a reta de equacao 4x− 3y + 2 = 0?

a)

(1

8,−1

16

)b)

(1

4,−√

2

16

)c)(1,−√

2)

d) (2,−4)

e)

(1

2,−1

2

)Problema 153

(EN-2011) Sendo x e y numeros reais, a soma de todos os valores de x

e de y, que satisfazem ao sistema

xy =

1

y2

yx =1√x

a)36

5

b)9

2

42 J. Bartasevicius

Page 43: bartasevicius.com.brbartasevicius.com.br/sites/default/files/2018-12/... · Sobre este trabalho. E permitido: • Compartilhar - copiar e redistribuir o material em qualquer meio

CAPITULO 1. PROBLEMAS

c)5

2

d)25

4

e)−1

2

Problema 154(EN-2011) Sendo i =

√−1, n ∈ N, z =

i8n−5 + i4n−8

3+ 2i e P (x) =

−2x3 + x2 − 5x + 11 um polinomio sobre o conjunto dos numeroscomplexos, entao P (z) vale

a) −167 + 4i

b) 41 + 0i

c) −167− 4i

d) 41 + 2i

e) 0 + 4i

Problema 155(EN-2011) Tres numeros inteiros estao em PG. A soma destes numerosvale 13 e a soma dos seus quadrados vale 91. Chamando de n o termodo meio desta PG, quantas comissoes de n elementos, a Escola Navalpode formar 28 professores do Centro Tecnico Cientıfico?

a) 2276

b) 3176

c) 3276

d) 19656

e) 19556

Problema 156(EN-2012) Uma esfera confeccionada em aco e usada em um rolamentode motor de um navio da Marinha do Brasil. Se o raio da esfera mede√

3

√5

√3√

5√

3 . . .cm, entao seu volume vale

a) 45.10−3π dm3

b) 0, 45.10−3π dm3

c) 60.10−3π dm3

J. Bartasevicius 43

Page 44: bartasevicius.com.brbartasevicius.com.br/sites/default/files/2018-12/... · Sobre este trabalho. E permitido: • Compartilhar - copiar e redistribuir o material em qualquer meio

CAPITULO 1. PROBLEMAS

d) 0, 15.103π dm3

e) 60.103π dm3

Problema 157(EN-2012) Considere a sequencia (a, b, 2) uma progressao aritmetica e asequencia (b, a, 2) uma progressao geometrica nao constante, a, b ∈ R.A equacao da reta que passa pelo ponto (a, b) e pelo vertice da curvay2 − 2y + x+ 3 = 0

a) 6y − x− 4 = 0

b) 2x− 4y − 1 = 0

c) 2x− 4y + 1 = 0

d) x+ 2y = 0

e) x− 2y = 0

Problema 158(EN-2012) Considere como espaco amostral (Ω), o cırculo no plano xyde centro na origem e raio igual a 2. Qual a probabilidade do eventoA = (x, y) ∈ Ω | |x|+ |y| < 1?

a)2

π

b) 4π

c)1

π

d)1

e) π

Problema 159(EN-2012) Seja m a menor raiz inteira da equacao[

(x− 1)(5x− 7)

3

]! = 1.

Pode-se afirmar que o termo medio do desenvolvimento de(√

y − z3)12m

e

a)12!

6!6!y18z

32

b)−12!

6!6!y3z18

44 J. Bartasevicius

Page 45: bartasevicius.com.brbartasevicius.com.br/sites/default/files/2018-12/... · Sobre este trabalho. E permitido: • Compartilhar - copiar e redistribuir o material em qualquer meio

CAPITULO 1. PROBLEMAS

c)30!

15!15!y152 z45

d)−30!

15!15!y152 z45

e)12!

6!6!y3z18

Problema 160(EN Masculino-2013) Considere f uma funcao real de variavel real talque:

(1) f(x+ y) = f(x)f(y)

(2) f(1) = 3

(3) f(√

2) = 2

Entao, f(2 + 3√

2) e igual a

a) 108

b) 72

c) 54

d) 36

e) 12

Problema 161(EN Masculino-2013) De um curso preparatorio de matematica parao concurso publico de ingresso a Marinha participaram menos de 150pessoas. Destas, o numero de mulheres estava para o numero de homensna razao de 2 para 5, respectivamente. Considerando que a quantidadede participantes foi a maior possıvel, de quantas unidades o numero dehomens excedia o de mulheres?

a) 50

b) 55

c) 57

d) 60

e) 63

Problema 162(EN Feminino-2013) Sejam F (x) = x3 + ax+ b e G(x) = 2x2 + 2x− 6dois polinomios na variavel real x, com a e b numeros reais. Qual o

valor de (a+ b) para que a divisaoF (x)

G(x)seja exata?

J. Bartasevicius 45

Page 46: bartasevicius.com.brbartasevicius.com.br/sites/default/files/2018-12/... · Sobre este trabalho. E permitido: • Compartilhar - copiar e redistribuir o material em qualquer meio

CAPITULO 1. PROBLEMAS

a) −2

b) −1

c) 0

d) 1

e) 2

Problema 163

(EN Feminino-2013) O coeficiente de x5 no desenvolvimento de

(2

x+ x3

)7

e

a) 30

b) 90

c) 120

d) 270

e) 560

Problema 164(EN Feminino-2013) Qual o menor valor de n, n inteiro maior que zero,para que (1 + i)n seja um numero real?

a) 2

b) 3

c) 4

d) 5

e) 6

Problema 165(EN Feminino-2013) Um aspirante da Escola Naval tem, em uma prate-leira de sua estante, 2 livros de Calculo, 3 livros de Historia e 4 livrosde Eletricidade. De quantas maneiras ele pode dispor estes livros naprateleira de forma que os livros de cada discipplina estejam semprejuntos?

a) 1728

b) 1280

c) 960

d) 864

46 J. Bartasevicius

Page 47: bartasevicius.com.brbartasevicius.com.br/sites/default/files/2018-12/... · Sobre este trabalho. E permitido: • Compartilhar - copiar e redistribuir o material em qualquer meio

CAPITULO 1. PROBLEMAS

e) 288

Problema 166(EN Feminino-2014) Sabendo que log x representa o logaritmo de xna base 10, qual e o domınio da funcao real de variavel real f(x) =

arccos3(

logx

10

)√

4x− x3?

a) ]0, 2[

b)

]1

2, 1

[c) ]0, 1]

d) [1, 2[

e)

[1

2, 2

[Problema 167

(EN Feminino-2014) Considere a sequencia x1 =1

2; x2 =

1 + 2

1 + 2; x3 =

1 + 2 + 3

1 + 2 + 4; x4 =

1 + 2 + 3 + 4

1 + 2 + 4 + 8; . . .. O valor de xn e

a)n+ 1

2

b)n(n− 1)

2n

c)n(n+ 1)

2n − 1

d)n(n+ 1)

2n

e)n(n+ 1)

2 (2n − 1)

Problema 168(EN Feminino-2014) Um restaurante a quilo vende 200 quilos de comidapor dia, a 40 reais o quilo. Uma pesquisa de opiniao revelou que, a cadaaumento de um real no preco do quilo, o restaurante perde 8 clientespor dia, com um consumo medio de 500 gramas cada. Qual deve sero valor do quilo de comida, em reais, para que o restaurante tenha amaior receita possıvel por dia?

J. Bartasevicius 47

Page 48: bartasevicius.com.brbartasevicius.com.br/sites/default/files/2018-12/... · Sobre este trabalho. E permitido: • Compartilhar - copiar e redistribuir o material em qualquer meio

CAPITULO 1. PROBLEMAS

a) 52

b) 51

c) 46

d) 45

e) 42

Problema 169

(EN Masculino-2014) Sabendo que z e o numero complexo z =1

2+

√3

2i,

o qual o menor inteiro positivo n, para o qual o produto z.z2.z3 . . . zn eum real positivo?

a) 1

b) 2

c) 3

d) 4

e) 5

Problema 170(Magisterio Marinha - 2007) Duas pessoas combinam de se encontrarem um determinado lugar entre meio dia e 13 horas, podendo cada umadelas escolher o horario aleatoriamente. O primeiro a chegar esperara15 min pelo outro, sendo que, se o outro nao chegar, esse primeirovoltara para casa, e o encontro nao acontecera. Alem disso, nenhumdeles continuara a esperar alem das 13 horas. Qual a probabilidade deo encontro acontecer?

a) 516

b) 38

c) 716

d) 12

e) 58

Problema 171(Magisterio Marinha - 2007) Considere as expressoes abaixo.

x =2, 00000000002

1, 000000000022 + 2, 00000000002

48 J. Bartasevicius

Page 49: bartasevicius.com.brbartasevicius.com.br/sites/default/files/2018-12/... · Sobre este trabalho. E permitido: • Compartilhar - copiar e redistribuir o material em qualquer meio

CAPITULO 1. PROBLEMAS

y =2, 00000000004

1, 000000000042 + 2, 00000000004e

z =20, 00000000002

19, 000000000022 + 20, 00000000002

Sobre elas, e correto afirmar que:

a) x > y > z

b) x > z > y

c) y > x > z

d) y > z > x

e) z > y > x

Problema 172(CN-2007) Se x+ y = 2 e (x2 + y2)/(x3 + y3) = 4, entao xy e igual a

a) 12/11

b) 13/11

c) 14/11

d) 15/11

e) 16/11

Problema 173(CN-2007) Em um numero natural N de 9 algarismos, tem-se: osalgarismos das unidades simples, unidades de milhar e unidades demilhao iguais a x; os algarismos das dezenas simples, dezenas de milhare dezenas de milhao iguais a y; e os algarismos das centenas simples,centenas de milhar e centenas de milhao iguais a z. Pode-se afirmarque N sempre sera divisıvel por

a) 333664

b) 333665

c) 333666

d) 333667

e) 333668

Problema 174(CN-2008) O numero a 6= 0 tem inverso igual a b. Sabendo-se quea+ b = 2, qual e o valor de (a3 + b3)(a4 − b4)?

J. Bartasevicius 49

Page 50: bartasevicius.com.brbartasevicius.com.br/sites/default/files/2018-12/... · Sobre este trabalho. E permitido: • Compartilhar - copiar e redistribuir o material em qualquer meio

CAPITULO 1. PROBLEMAS

a) 8

b) 6

c) 4

d) 2

e) 0

Problema 175(CN-2008) Qual e a soma dos quadrados das raızes da equacao 2

x−1 +3

x+1 = 1, com x real e x 6= ±1?

a) 16

b) 20

c) 23

d) 25

e) 30

Problema 176(CN-2008) A solucao de

√4x2 − 4x+ 1 =

3√−1 + 6x− 12x2 + 8x3 no

campo reais e

a) o conjunto vazio

b) 1/2c) -1/2,1/2d) [1/2,+∞[

e) ]−∞,+∞[

Problema 177(CN-2008) Num determinado jogo, o apostador recebe, toda vez queganha, o valor apostado inicialmente, mais 25% do mesmo; e recebe, todavez que perde, apenas 25% do valor apostado inicialmente. Sabendo-se que foi feita uma aposta inicial de uma quantia x e que foramrealizadas quatro jogadas, sempre sendo apostado o valor total obtidona jogada anterior, das quais ganhou-se duas e perdeu-se duas, qual e,aproximadamanete, o percentual de x obtido no final?

a) 3,7

b) 4,7

c) 5,7

50 J. Bartasevicius

Page 51: bartasevicius.com.brbartasevicius.com.br/sites/default/files/2018-12/... · Sobre este trabalho. E permitido: • Compartilhar - copiar e redistribuir o material em qualquer meio

CAPITULO 1. PROBLEMAS

d) 6,7

e) 9,8

Problema 178

(CN-2008) O valor de(3+2

√2)2008

(5√

2+7)1338+ 3− 2

√2 e um numero

a) multiplo de onze

b) multiplo de sete

c) multiplo de cinco

d) multiplo de tres

e) primo

Problema 179(CN-2009) A menor raiz da equacao ax2 +bx+c, com abc 6= 0, e a mediageometrica entre m e a maior raiz. A maior raiz e a media geometricaentre n e a menor raiz. Pode-se afirmar que m+ n e expresso por:

a) 3abc−b3a2c

b) 3abc+b3

a2c

c) 3abc−b3c2a

d) abc+b3

c2a

e) abc−b3a2c

Problema 180(CN-2009) O conjunto solucao de numeros reais, tal que o valor da

expressao(x−5)15(2x−1)10

(3x+1)8e maior do que, ou igual a zero, e:

a) [5,+∞[∪−13 ,

12

b) ]−∞, 12 ] ∪ [5,+∞[

c) ]−∞,+∞[

d) ]− 13 ,

12 ] ∪ [5,+∞[

e) 12 ∪ [5,+∞[

Problema 181(CN-2009) Quantos sao os numeros inteiros com os quais e possıvel, noconjunto dos reais, calcular o valor numerico da expressao algebrica√

103x− x2 − 300?

J. Bartasevicius 51

Page 52: bartasevicius.com.brbartasevicius.com.br/sites/default/files/2018-12/... · Sobre este trabalho. E permitido: • Compartilhar - copiar e redistribuir o material em qualquer meio

CAPITULO 1. PROBLEMAS

a) 100

b) 99

c) 98

d) 97

e) 96

Problema 182(CN-2010) No conjunto dos inteiros positivos sabe-se que a e primo comb quando mdc(a, b) = 1.Em relacao a este conjunto, analise as afirmativas a seguir.

I - A fatoracao em numeros primos e unica.

II - Existem 8 numeros primos com 24 e menores que 24.

III - Se (a+ b)2 = (a+ c)2 entao b = c.

IV - Se a < b, entao a.c < b.c.

Quantas das afirmativas acima sao verdadeiras?

a) 0

b) 1

c) 2

d) 3

e) 4

Problema 183(CN-2010) Estudando os quadrados dos numeros naturais, um alunoconseguiu determinar corretamente o numero de solucoes inteiras epositivas da equacao 5x2 + 11y2 = 876543. Qual foi o numero desolucoes que este aluno obteve?

a) 0

b) 1

c) 2

d) 3

e) 4

Problema 184(CN-2010) No conjunto dos numeros reais, o conjunto solucao da equacao4√

(2x− 1)4 = 3x+ 2

52 J. Bartasevicius

Page 53: bartasevicius.com.brbartasevicius.com.br/sites/default/files/2018-12/... · Sobre este trabalho. E permitido: • Compartilhar - copiar e redistribuir o material em qualquer meio

CAPITULO 1. PROBLEMAS

a) e vazio.

b) e unitario.

c) possui dois elementos.

d) possui tres elementos.

e) possui quatro elementos.

Problema 185

(CN-2010) No sistema

3x− y

√3 = 0

x2y−2 = 13

, a quantidade de solucoes

inteiras para x e y e:

a) 0

b) 1

c) 2

d) 3

e) infinita.

Problema 186(CN-2010) Considere o sistema abaixo nas variaveis reais x e y, sendo ae b reais.

375y2x− 125y3 − 375yx2 + 125x3 = 125b

y2 + x2 + 2yx = a2

Nessas condicoes, qual sera o valor de(x2 − y2

)6?

a) a3b6

b) a8b6

c) a6b2

d) a3b6

e) a4b6

Problema 187(CN-2011) A quantidade de solucoes reais e distintas da equacao 3x3 −√

33x3 + 97 = 5

a) 1

b) 2

J. Bartasevicius 53

Page 54: bartasevicius.com.brbartasevicius.com.br/sites/default/files/2018-12/... · Sobre este trabalho. E permitido: • Compartilhar - copiar e redistribuir o material em qualquer meio

CAPITULO 1. PROBLEMAS

c) 3

d) 5

e) 6

Problema 188

(CN-2011) O valor de

√90,5x0, 333 . . .+ 7

√4x√

0, 0625− (3,444...+4,555...)3√64

e

a) 0

b)√

2

c)√

3− 2

d)√

2− 2

e) 1

Problema 189(CN-2011) O numero real

3√

26− 15√

3 e igual a

a) 5−√

3

b)√

7− 4√

3

c) 3−√

2

d)√

13− 3√

3

e) 2

Problema 190(CN-2012) Os numeros (35041000)7, (11600)7 e (62350000)7 estao nabase 7. Esses numeros terminam, respectivamente, com 3, 2 e 4 zeros.Com quantos zeros terminara o numero de base decimal n = 212012, nabase 7?

a) 2012

b) 2013

c) 2014

d) 2015

e) 2016

Problema 191(CN-2012) Qual o menor valor positivo de 2160x+ 1680y, sabendo quex e y sao numeros inteiros?

54 J. Bartasevicius

Page 55: bartasevicius.com.brbartasevicius.com.br/sites/default/files/2018-12/... · Sobre este trabalho. E permitido: • Compartilhar - copiar e redistribuir o material em qualquer meio

CAPITULO 1. PROBLEMAS

a) 30

b) 60

c) 120

d) 240

e) 480

Problema 192(CN-2012) Na fabricacao de um produto e utilizado o ingrediente A ouB. Sabe-se que 10 quilogramas (kg) do ingrediente A produz o mesmoefeito que 100kg do ingrediente B. Se a soma de x kg do ingrediente Acom y kg do ingrediente B e igual a 44000 gramas, entao:

a) yx = 260

b)√x.y = 5

√10

c) 10√yx = 256

d) 4√xy = 20

e)√

yx = 2

√5

Problema 193(CN-2012) Sabendo que A = 3+

√6

5√

3−2√

12−√

32+√

50, qual e o valor de

A2

6√A7

?

a)5√

34

b)7√

36

c)8√

35

d)10√

37

e)12√

35

Problema 194(CN-2013) Sejam P =

(1 + 1

3

)(1 + 1

5

)(1 + 1

7

)(1 + 1

9

)(1 + 1

11

)e

Q =(

1− 15

)(1− 1

7

)(1− 1

9

)(1− 1

11

). Qual e o valor de

√PQ?

a)√

2

b) 2

c)√

5

J. Bartasevicius 55

Page 56: bartasevicius.com.brbartasevicius.com.br/sites/default/files/2018-12/... · Sobre este trabalho. E permitido: • Compartilhar - copiar e redistribuir o material em qualquer meio

CAPITULO 1. PROBLEMAS

d) 3

e) 5

Problema 195(CN-2013) Qual e o valor da expressao

(30,333...)27

+ 2217

− 5

√239 +

3

√448

7−(

3√

3)33

7√92

a) 0,3

b) 3√

3

c) 1

d) 0

e) -1

Problema 196(CN-2013) Seja a, b, x, y numeros naturais nao nulos. Se a.b = 5, k =

2(a+b)2

2(a−b)2e x2 − y2 = 5

√k, qual e o algarismo das unidades do numero

(yx − xy)?

a) 2

b) 3

c) 5

d) 7

e) 8

Problema 197(CN-2013) O maior inteiro n, tal que n2+37

n+5 tambem e inteiro, tem comosoma dos seus algarismos um valor igual a

a) 6

b) 8

c) 10

d) 12

e) 14

56 J. Bartasevicius

Page 57: bartasevicius.com.brbartasevicius.com.br/sites/default/files/2018-12/... · Sobre este trabalho. E permitido: • Compartilhar - copiar e redistribuir o material em qualquer meio

CAPITULO 1. PROBLEMAS

Problema 198(CN-2013) Dado que a e b sao numeros reais nao nulos, com b 6= 4a, e

que

1 + 2

ab = 55−2b2

4a−b = 4a+ b, qual e o valor de 16a4b2 − 8a3b3 + a2b4?

a) 4

b) 118

c) 112

d) 18

e) 14

Problema 199(CN-2014) A solucao real da equacao

√x+ 4 +

√x− 1 = 5 e:

a) multiplo de 3.

b) par e maior do que 17.

c) ımpar e nao primo.

d) um divisor de 130.

e) uma potencia de 2.

Problema 200(CN-2014) Considere a equacao do 2° grau 2014x2 − 2015x− 4029 = 0.Sabendo-se que a raiz nao inteira e dada por a

b , onde a e b sao primosentre si, a soma dos algarismos de a+ b e:

a) 7

b) 9

c) 11

d) 13

e) 15

J. Bartasevicius 57

Page 58: bartasevicius.com.brbartasevicius.com.br/sites/default/files/2018-12/... · Sobre este trabalho. E permitido: • Compartilhar - copiar e redistribuir o material em qualquer meio

Capıtulo 2

Solucoes

Solucao 1Substituindo y = ax em xy = yx, tem-se:

xax = (ax)x =⇒ (xa)x = (ax)x =⇒ xa = ax

Sendo x 6= 0, divide-se a equacao por x e tem-se:

xa−1 = a

∴ x = a−1√a

Solucao 2

tan a+ tan b = −p =⇒ sin a

cos a+

sin b

cos b= −p =⇒

sin a cos b+ sin b cos a

cos a cos b= −p =⇒

sin(a+ b)

cos a cos b= −p =⇒

sin(a+ b) = −p(cos a cos b) (A)

tan a. tan b = q =⇒ sin a

cos a× sin b

cos b= q =⇒

sin a sin b

cos a cos b= q =⇒ − sin a sin b

cos a cos b= −q =⇒ 1− sin a sin b

cos a cos b= 1−q =⇒

cos a cos b− sin a sin b

cos a cos b= 1− q =⇒ cos(a+ b)

cos a cos b= 1− q =⇒

58

Page 59: bartasevicius.com.brbartasevicius.com.br/sites/default/files/2018-12/... · Sobre este trabalho. E permitido: • Compartilhar - copiar e redistribuir o material em qualquer meio

CAPITULO 2. SOLUCOES

cos(a+ b) = (1− q)(cos a cos b) (B)

Fazendo (A) ÷ (B):

sin(a+ b)

cos(a+ b)=−p(cos a cos b)

(1− q)(cos a cos b)=⇒

tan(a+ b) =−p

1− q(C)

Calculando y:

y =

[sin(a+ b)

cos(a+ b)+ p+ q.

cos(a+ b)

sin(a+ b)

]. sin(a+ b) cos(a+ b) =⇒

y =

[tan(a+ b) + p+ q.

1

tan(a+ b)

]. [sin(a+ b) cos(a+ b)]

Substituindo sin(a+ b), cos(a+ b) e tan(a+ b) pelas expressoes em (A),(B) e (C):

y =

[−p

1− q+ p+ q.

1− q−p

]. [−p(cos a cos b)] [(1− q)(cos a cos b)] =⇒

y =

[p2 + p(1− q)(−p) + q(1− q)2

(1− q)(−p)

].[(−p)(1− q)(cos2 a cos2 b)

]=⇒

y =[p2 + (−p2 + p2q) + (q − 2q2 + q3)

].

[1

tan2 a+ 1

1

tan2 b+ 1)

]=⇒

y =[p2q + q − 2q2 + q3

].

[1

tan2 a tan2 b+ tan2 a+ tan2 b+ 1

]=⇒

y =[q3 − 2q2 + q + p2q

×

1

(tan a tan b)2 +(

(tan a+ tan b)2 − 2 tan a tan b)

+ 1

=⇒

y =[q3 − 2q2 + q + p2q

1

q2 +(

(−p)2 − 2q)

+ 1

=⇒

y =q3 − 2q2 + q + p2q

q2 + p2 − 2q + 1=q(q2 − 2q + 1 + p2

)q2 − 2q + 1 + p2

∴ y = q

J. Bartasevicius 59

Page 60: bartasevicius.com.brbartasevicius.com.br/sites/default/files/2018-12/... · Sobre este trabalho. E permitido: • Compartilhar - copiar e redistribuir o material em qualquer meio

CAPITULO 2. SOLUCOES

Solucao 3Nomeia-se P (x) = (cosϕ+ x sinϕ)n.Chamando o divisor d(x) =

(x2 + 1

), um polinomio de 2º grau, o resto

r(x) deve ser um polinomio de, no maximo, primeiro grau: r(x) = αx+β.Portanto, P (x) = d(x)q(x) + r(x).Ao utilizar x com o valor das raızes do divisor, tem-se que o valor dopolinomio e o mesmo valor do resto.As raızes de d(x) =

(x2 + 1

)sao x = ±i.

Logo:P (i) = d(i)︸︷︷︸

=0

q(i) + r(i) =⇒ P (i) = r(i) (A)

eP (−i) = d(−i)︸ ︷︷ ︸

=0

q(−i) + r(−i) =⇒ P (−i) = r(−i) (B)

Substituindo x por ±i em (A) e (B):(cosϕ+ i sinϕ)n = αi+ β

(cosϕ− i sinϕ)n = α(−i) + β

Transformando cosϕ± i sinϕ para coordenada polar:(1 ϕ)n = iα + β

(1 −ϕ)n = −iα + β=⇒

1 nϕ = iα + β

1 −nϕ = −iα + β

Voltando para representacao em coordenada retangular:cos(nϕ) + i sin(nϕ) = iα + β

cos(nϕ)− i sin(nϕ) = −iα + β=⇒

α = sin(nϕ)

β = cos(nϕ)

Substituindo α e β em r(x):

r(x) = αx+ β

∴ r(x) = sin(nϕ)x+ cos(nϕ)

Solucao 4

1− sinx =√

3 cosx =⇒

60 J. Bartasevicius

Page 61: bartasevicius.com.brbartasevicius.com.br/sites/default/files/2018-12/... · Sobre este trabalho. E permitido: • Compartilhar - copiar e redistribuir o material em qualquer meio

CAPITULO 2. SOLUCOES

Como −1 6 sinx 6 1, logo√

3 cosx > 0 =⇒ −π26 x 6

π

2.

1− sinx =√

3√

1− sin2 x =⇒

1− 2 sinx+ sin2 x = 3(

1− sin2 x)

=⇒

4 sin2 x− 2 sinx− 2 = 0 =⇒ (÷2)

2 sin2 x− sinx− 1 = 0 =⇒

sinx = 1

sinx = −1

2

Entao, x =π

2ou x ∈

−π

6,−5π

6

.

Como cosx > 0, sao somente solucoesπ

2e −π

6.

∴ x =π

2+ 2kπ ou x = −π

6+ 2kπ, k ∈ Z.

Solucao 5

αxγ+1 + βxγ + 1 | (x− 1)2

αxγ+1 + βxγ + 1 precisa ser dividido duas vezes por (x− 1).

Na primeira divisao de αxγ+1 + βxγ + 1, o quociente e αxγ + (α +β)xγ−1 + (α + β)xγ−2 + . . .+ (α + β) e o resto e α + β + 1.A divisao e exata, entao α + β + 1 = 0. Logo,

α + β = −1 (A)

Substituindo α+β = −1, o quociente sera αxγ−xγ−1−xγ−2 + . . .−1.

Dividindo este quociente novamente por (x− 1), sera realizada a divisaopor (x− 1)2.

αxγ − xγ−1 − xγ−2 + . . .− 1 dividido por (x− 1) resulta em quocienteαxγ−1 + (α− 1)xγ−2 + (α− 2)xγ−3 + (α− 3)xγ−4 + . . .+ (α− (γ− 1))e resto α− γ + 1.

A divisao e exata, entao α− γ + 1 = 0, e

α = γ − 1 (B)

J. Bartasevicius 61

Page 62: bartasevicius.com.brbartasevicius.com.br/sites/default/files/2018-12/... · Sobre este trabalho. E permitido: • Compartilhar - copiar e redistribuir o material em qualquer meio

CAPITULO 2. SOLUCOES

O quociente e 120 para x = 1. Substituindo no quociente, tem-se oresultado

α + (α− 1) + (α− 2) + (α− 3) + . . .+ (α− (γ − 1))︸ ︷︷ ︸γ termos

= 120

αγ − (1 + 2 + 3 + . . .+ (γ − 1)) = 120

αγ − (γ − 1)γ

2= 120

Substituindo α de B:

(γ − 1)γ − (γ − 1)γ

2= 120

γ2 − γ − 240 = 0 =⇒ γ = 16 ∨ γ = −15

γ = −15 nao e uma solucao possıvel, pois os expoentes de x no po-linomio sao inteiros positivos.

Com γ = 16, de (B) α = 15. E de (A), β = −14.

∴ γ = 16, α = 15 e β = −14.

Solucao 6

(2n+ 1) + (2n+ 3) + (2n+ 5) + . . .+ (2n+ (2a+ 1))︸ ︷︷ ︸a+1 termos

= 73

Portanto, a serie inicia em (2n+ 1) e termina em (2n+ (2a+ 1)).

(2n)(a+ 1) + (1 + 3 + 5 + . . .+ (2a+ 1)) = 73

(2n)(a+ 1) +(2a+ 2)(a+ 1)

2= 73

(2n)(a+ 1) + (a+ 1)(a+ 1) = 73

(a+ 1)(2n+ a+ 1) = 73 (A)

De (A) ha 4 possibilidades para a, n ∈ Z:

I. (a+ 1) = 70 e (2n+ a+ 1) = 73 =⇒ a = 0 e n = 168. Neste caso,existe somente um numero na serie: 343;

62 J. Bartasevicius

Page 63: bartasevicius.com.brbartasevicius.com.br/sites/default/files/2018-12/... · Sobre este trabalho. E permitido: • Compartilhar - copiar e redistribuir o material em qualquer meio

CAPITULO 2. SOLUCOES

II. (a+ 1) = 71 e (2n+ a+ 1) = 72 =⇒ a = 6 e n = 21. Neste caso,a serie se inicia em 43 e termina em 55;

III. (a + 1) = 72 e (2n + a + 1) = 71 =⇒ a = 48 e n = −21. Nestecaso, a serie se inicia em -41 e termina em 55;

IV. (a+ 1) = 73 e (2n+ a+ 1) = 70 =⇒ a = 342 e n = −171. Nestecaso, a serie se inicia em -341 e termina em 343;

Solucao 7

z2 + 2iz + (2− 4i) = 0

z =−2i±

√(2i)2 − 4(2− 4i)

2

z =−2i± 2

√−3 + 4i

2

z = −i±√−3 + 4i

(X + Y i)2 = −3 + 4i =⇒ X = 1 e Y = 2,∴√−3 + 4i = 1 + 2i

∴ z1 = 1 + i e z2 = −1− 3i

Localizando no plano complexo:

0 1 2 3-1-2-3

z1

z2

i

2i

3i

−i

−2i

−3i

C

R

J. Bartasevicius 63

Page 64: bartasevicius.com.brbartasevicius.com.br/sites/default/files/2018-12/... · Sobre este trabalho. E permitido: • Compartilhar - copiar e redistribuir o material em qualquer meio

CAPITULO 2. SOLUCOES

Solucao 8O artifıcio para solucionar este tipo de problema e transformar a matrizde modo a possuir zeros em seu triangulo superior ou inferior para queo determinante seja o produto da diagonal.Diminuindo os elementos das demais colunas do valor dos elementos daprimeira coluna:

D =

∣∣∣∣∣∣∣∣∣∣∣∣∣∣

1 1 1 1 1 1 11 3 1 1 1 1 11 1 5 1 1 1 11 1 1 7 1 1 11 1 1 1 9 1 11 1 1 1 1 11 11 1 1 1 1 1 13

∣∣∣∣∣∣∣∣∣∣∣∣∣∣=

∣∣∣∣∣∣∣∣∣∣∣∣∣∣

1 0 0 0 0 0 01 2 0 0 0 0 01 0 4 0 0 0 01 0 0 6 0 0 01 0 0 0 8 0 01 0 0 0 0 10 01 0 0 0 0 0 12

∣∣∣∣∣∣∣∣∣∣∣∣∣∣Com o aparecimento de zeros no “triangulo superior” da matriz, calculodo determinante e simplificado:

D = 1×

∣∣∣∣∣∣∣∣∣∣∣∣

2 0 0 0 0 00 4 0 0 0 00 0 6 0 0 00 0 0 8 0 00 0 0 0 10 00 0 0 0 0 12

∣∣∣∣∣∣∣∣∣∣∣∣= 1× 2×

∣∣∣∣∣∣∣∣∣∣4 0 0 0 00 6 0 0 00 0 8 0 00 0 0 10 00 0 0 0 12

∣∣∣∣∣∣∣∣∣∣=

= 1× 2× 4×

∣∣∣∣∣∣∣∣6 0 0 00 8 0 00 0 10 00 0 0 12

∣∣∣∣∣∣∣∣ = . . .

∴ D = 1× 2× 4× . . .× 12 = 46080

Solucao 9

n∑k=1

k2 = 12 + 22 + 32 + . . .+ n2 =⇒

=⇒n∑k=1

k2 =n−1∑k=1

k2 + n2 (A)

64 J. Bartasevicius

Page 65: bartasevicius.com.brbartasevicius.com.br/sites/default/files/2018-12/... · Sobre este trabalho. E permitido: • Compartilhar - copiar e redistribuir o material em qualquer meio

CAPITULO 2. SOLUCOES

Como o enunciado da questao diz que o polinomio possui, no maximo,4 termos, entao pode-se assumir o polinomio na forma

n∑k=1

k2 = an3 + bn2 + cn+ d (B)

Aplicando (B) em (A), tem-se:

an3 + bn2 + cn+ d = a(n− 1)3 + b(n− 1)2 + c(n− 1) + d+ n2 =⇒

an3 + bn2 + cn+ d = an3 + (−3a+ b+ 1)n2 + (3a− 2b+ c)n+ (d− c)Igualando os coeficientes tem-se:

a = a

b = −3a+ b+ 1 =⇒ a = 13

c = 3a− 2b+ c =⇒ 3a− 2b = 0

d = d− c+ b− a =⇒ a− b+ c = 0

Com a = 13 , 3.13 − 2b = 0 =⇒ b = 1

2 .

a− b+ c = 0 e ∴ c = 16 .

Para o calculo da cosntante d, deve-se substituir n por 1 emn∑k=1

k2:

12 =1

3.13 +

1

2.12 +

1

6.1 + d =⇒ d = 0

∴ O polinomio e 2n3+3n2+n6

Solucao 10Incialmente, deve-se separar cada uma das possibilidades por lancamento:Jogador I Jogador II Jogador III Numero de ocorrencias

P P P aP P B bP B P cP B B dB P P eB P B fB B P gB B B h

a+ b+ c+ d+ e+ f + g + h = 50

J. Bartasevicius 65

Page 66: bartasevicius.com.brbartasevicius.com.br/sites/default/files/2018-12/... · Sobre este trabalho. E permitido: • Compartilhar - copiar e redistribuir o material em qualquer meio

CAPITULO 2. SOLUCOES

Ocorrer faces pretas para pelo menos um jogador e o mesmo quedescartar todas as ocorrencias onde houve faces brancas para todos osjogadores. Portanto, o resultado deste problema e 50− h. Analisandoas afirmacoes:

a. 28 ocorrencias P** =⇒ a+ b+ c+d = 28 =⇒ e+ f + g+h = 22

b. 25 ocorrencias *B* =⇒ c+ d+ g + h = 25

c. 27 ocorrencias **B =⇒ b+ d+ f + h = 27

d. 8 ocorrencias PBP =⇒ c = 8

e. 7 ocorrencias PBB =⇒ d = 7

f. 4 ocorrencias PPP =⇒ a = 4

g. 11 ocorrencias *PP =⇒ a+ e = 11 =⇒ e = 7

Com a = 4, c = 8, d = 7 e a+ b+ c+ d = 28, entao b = 9. Com c = 8,d = 7 e c+ d+ g + h = 25, entao

g + h = 10 (A)

Com b = 9, d = 7 e b+ d+ f + h = 27, entao

f + h = 11 (B)

(A)+(B):f + g + 2h = 21 (C)

Somando e com C:

e+ f + g + 2h = 21 + 7 = 28 =⇒ e+ f + g + h+ h = 28 (D)

Substituindo e+ f + g + h = 22 em D tem-se 22 + h = 28 =⇒ h = 6.

∴ 50− h = 50− 6 = 44.

Solucao 11Alterando a ordem dos fatores:

(d− a)(d− b)(d− c)(a− b)(c− a)(c− b)

Para que o produto acima seja multiplo de 12, ele devera ser multiplode 4 e multiplo de 3.

66 J. Bartasevicius

Page 67: bartasevicius.com.brbartasevicius.com.br/sites/default/files/2018-12/... · Sobre este trabalho. E permitido: • Compartilhar - copiar e redistribuir o material em qualquer meio

CAPITULO 2. SOLUCOES

1. Multiplo de 4: Para o produto (d− a)(d− b)(d− c) existem treshipoteses:

(a) Todas as parcelas (d− a), (d− b) e (d− c) sao ımpares.Desta forma, a, b e c possuem o mesmo resto da divisao por2, o que implica as parcelas (a − b), (c − a) e (c − b) seremtodas multiplos de 2. Assim, o produto (a− b)(c− a)(c− b) emultiplo de 8 e, portanto, multilo de 4.

(b) Somente uma das parcelas (d− a), (d− b) ou (d− c) e par.Neste caso, a e b, ou a e c, ou b e c possuem o mesmo resto dadivisao por 2, implicando uma das parcelas (a− b), (c− a) ou(c− b) ser multiplo de 2. Ora se uma das parcelas com d e pare uma das demais parcelas tambem possui uma subtracao par,entao ha duas parcelas pares, confirmando a multiplicidadepor 4.

(c) Duas ou tres parcelas de (d− a), (d− b) ou (d− c) sao pares.Neste caso, havendo duas parcelas pares, ha um multiplo de4, sem a necessidade de analisar as parcelas (a− b), (c− a) e(c− b).

Portanto, em qualquer dos casos, (d− a)(d− b)(d− c)(a− b)(c−a)(c− b) e multiplo de 4.

2. Multiplo de 3: Para o produto (d− a)(d− b)(d− c) existem duashipoteses:

(a) Nenhuma das parcelas (d− a), (d− b) ou (d− c) e multiplode 3.Se uma parcela nao e multiplo de 3, ela so pode ter resto 1ou resto 2. Portanto, existem dois numeros entre a, b e c quepossuem o mesmo resto da divisao por tres. Assim, uma dasparcelas (a− b), (c− a) ou (c− b) e multiplo de 3.

(b) Uma das parcelas (d− a), (d− b) ou (d− c) e multiplo de 3.Neste caso, o produto (d − a)(d − b)(d − c) e multiplo de 3,sem a necessidade de analisar as demais parcelas.

Concluindo, como (d− a)(d− b)(d− c)(a− b)(c− a)(c− b) e multiplode 3 e 4, ∴ (d− a)(d− b)(d− c)(a− b)(c− a)(c− b) e multiplo de 12.

Solucao 12

2 loga x = log1/b y. log√a b

J. Bartasevicius 67

Page 68: bartasevicius.com.brbartasevicius.com.br/sites/default/files/2018-12/... · Sobre este trabalho. E permitido: • Compartilhar - copiar e redistribuir o material em qualquer meio

CAPITULO 2. SOLUCOES

Convertendo para a mesma base:

2log x

log a=

log y

− log b.

log b12 log a

=⇒

log x = − log y =⇒ x =1

y(A)

Substituindo (A) em axb1/y =√ab:

axbx = (ab)1/2 =⇒ (ab)x = (ab)1/2 =⇒ x =1

2

De (A), y = 2.

Solucao 13Para que existam 11 patrulhas nas condicoes apresentadas pelo enunci-ado, e preciso completar a primeira patrulha com elementos suficientesque possam ser alocados nas demais 10 patrulhas. Assim, coloca-se10 voluntarios na primeira patrulha e cada um deles sera alocado nasdemais patrulhas separadamente, conforme abaixo:

P1 P2 P3 P4 P5 P6 P7 P8 P9 P10 P111 1 2 3 4 5 6 7 8 9 102 11 11 12 . . .

3 12. . .

4...

. . .

5. . .

6. . .

7. . .

8. . .

9. . .

10. . .

68 J. Bartasevicius

Page 69: bartasevicius.com.brbartasevicius.com.br/sites/default/files/2018-12/... · Sobre este trabalho. E permitido: • Compartilhar - copiar e redistribuir o material em qualquer meio

CAPITULO 2. SOLUCOES

Na patrulha 2 sera necessario alocar mais voluntarios e utilizar domesmo metodo de distribuicao nas demais patrulhas.Assim, e possıvel observar que havera 10 voluntarios em cada patrulhae o numero total sera a soma de uma serie decrescente de 10 ate 1:

10 + 9 + 8 + . . .+ 1 = 55

∴ sao 55 voluntarios e 10 patrulhas.

Solucao 14Um numero terminado em k tem a forma 10n+ k, com n ∈ N.

Ao se elevar 10n+k a um potencia p, havera o resultadop∑i=0

Cpi .(10n)p−iki.

Das parcelas apresentas, a unica que gera algarismo na casa das unida-des e quando i = p, pois nao ha fator 10 de multiplicacao.Desta forma, e necessario analisar somente o ultimo algarismo daspotencias de k:

k k2 k3 k4 k5

0 0 0 0 01 1 1 1 12 4 8 6 23 9 7 1 34 6 4 6 45 5 5 5 56 6 6 6 67 9 3 1 78 4 2 6 89 1 9 1 9

Solucao 15O enunciado infere que sejam somados todos os multiplos de 6 e 14,exceto os multiplos de mmc(6, 14), i.e. 42. A operacao a ser realizada e:

1. Soma de todos os multiplos de 6 desde 204 ate 498

2. Soma de todos os multiplos de 14 desde 210 ate 490

J. Bartasevicius 69

Page 70: bartasevicius.com.brbartasevicius.com.br/sites/default/files/2018-12/... · Sobre este trabalho. E permitido: • Compartilhar - copiar e redistribuir o material em qualquer meio

CAPITULO 2. SOLUCOES

3. Subtracao dupla (uma para excluir no conjunto dos multiplos de 6e outra para excluir no conjunto dos multiplos de 14) de todos osmultiplos de 42 desde 210 ate 462

204 + 210 + 216 + . . .+ 498 =702.50

2= 17550 (A)

210 + 224 + 238 + . . .+ 490 =700.21

2= 7350 (B)

210 + 252 + 294 + . . .+ 462 =672.7

2= 2352 (C)

(A)+(B)-2.(C)=20196

Solucao 16Como as operacoes sao de soma e multiplicacao, com x, y, z ∈ R+ fazerx 6 y 6 z sera analogo para todas as demais combinacoes de ordementre x, y e z.Assim, assumem-se os seguintes valores, com δ1, δ2 ∈ R+:

y = x+ δ1 (A)

z = x+ δ1 + δ2 (B)

Se e necessario provar x+y+z3 > 3

√x.y.z, entao deve-se provar que(

x+ y + z

3

)3

> x.y.z (C)

Aplicando (A) e (B) em (C) chega-se as expressoes (D) e (E):(x+ y + z

3

)3

=

(x+

2δ1 + δ23

)3

=⇒

x3 + 3x2(

2δ1 + δ23

)+ 3x

(2δ1 + δ2

3

)2

+

(2δ1 + δ2

3

)3

(D)

e

x.y.z = x(x+ δ1)(x+ δ1 + δ2) =⇒

x3 + x2 (2δ1 + δ2) + x(δ21 + δ1δ2

)(E)

70 J. Bartasevicius

Page 71: bartasevicius.com.brbartasevicius.com.br/sites/default/files/2018-12/... · Sobre este trabalho. E permitido: • Compartilhar - copiar e redistribuir o material em qualquer meio

CAPITULO 2. SOLUCOES

Fazendo (D) − (E) resulta em:

∆ = x

(δ21 + δ1δ2 + δ2

2

3

)+

(8δ3

1 + 12δ21δ2 + 6δ1δ

22 + δ3

2

27

)(F)

Com x, δ1, δ2 ∈ R+, entao ∆, em (F), e maior ou igual a zero por haversomente operacoes de soma e multiplicacao.

∴x+ y + z

3> 3√x.y.z.

E por δ1 e δ2 estarem presentes em todos as parcelas, a unica possibili-dade de se verificar a igualdade e δ1 = δ2 = 0, ou seja, x = y = z.

Solucao 17 √5−√

5− x = x (A)

Substituindo x dentro da expressao do lado esquerdo em (A), tem-serecursivamente:√

5−

√5− (

√5−√

5− x) = x =⇒

√√√√√√√5−

√√√√5−

√5−

√5−

√5−√

5− . . .︸ ︷︷ ︸x

= x =⇒

√5− x = x =⇒

x2 + x− 5 = 0 =⇒

x1 =

√21− 1

2∧ x2 =

−√

21− 1

2

Como x nao pode ser negativo, entao x =√

21−12 .

J. Bartasevicius 71

Page 72: bartasevicius.com.brbartasevicius.com.br/sites/default/files/2018-12/... · Sobre este trabalho. E permitido: • Compartilhar - copiar e redistribuir o material em qualquer meio

CAPITULO 2. SOLUCOES

Solucao 18Em representacao polar, z = 1 ϕ.Na multiplicacao em notacao polar, os modulos se multiplicam, en-quanto que os angulos se somam. Assim, zn = 1 nϕ.Da mesma forma, na divisao em notacao polar, os modulos se multi-plicam, enquanto que os angulos se subtraem. Para demonstrar que adivisao e um numero real, entao, os argumentos de zn e 1 + z2n devemser iguais. Calculo do argumento de 1 + z2n:

1 + z2n = 1 + 1 2nϕ = 1 + cos 2nϕ+ i sin 2nϕ =⇒

1 + (cos2 nϕ− sin2 nϕ) + i(2 sinnϕ cosnϕ) =⇒

2 cos2 nϕ+ 2i(sinnϕ cosnϕ) =⇒

2 cosnϕ(cosnϕ+ i sinnϕ) (A)

Em (A), a razao entre a parte complexa e a parte real e sinnϕcosnϕ , ou seja

tannϕ.Isto significa que o argumento e nϕ.

∴ 1 + z2n = 2 cosnϕ nϕ

Finalmente,

zn

1 + z2n=

1 nϕ

2 cosnϕ nϕ=

1

2 cosnϕ0

Solucao 19Fazendo log(12x3 − 19x2 + 8x) = y, iguala-se |y| = y, que somente sesatisfaz se y > 0.

∴ log(12x3 − 19x2 + 8x) > 0 =⇒ 12x3 − 19x2 + 8x > 1

12x3 − 19x2 + 8x− 1 > 0 =⇒

12x3 − 3x2 − 16x2 + 8x− 1 > 0 =⇒

3x2(4x− 1)− (4x− 1)2 > 0 =⇒

(4x− 1)(3x2 − 4x+ 1) > 0 =⇒

(4x− 1)(3x2 − 3x− x+ 1) > 0 =⇒

(4x− 1) (3x(x− 1)− (x− 1)) > 0 =⇒

(4x− 1)(3x− 1)(x− 1) > 0 (A)

72 J. Bartasevicius

Page 73: bartasevicius.com.brbartasevicius.com.br/sites/default/files/2018-12/... · Sobre este trabalho. E permitido: • Compartilhar - copiar e redistribuir o material em qualquer meio

CAPITULO 2. SOLUCOES

Aplicando a regra de multiplicacao de sinais, produz-se a seguinte tabelaa partir dos fatores de (A):

1/4

1/3

1

1/4 1/3 1

(4x− 1)

(3x− 1)

(x− 1)

(4x− 1)(3x− 1)(x− 1)

+ + +

+ +

+− − −

− −

− −+ +

∴ S = x ∈ R | 1

46 x 6

1

3∪ x > 1

Solucao 20Para resolver este problema e necessario saber o seguinte produtonotavel:

(a− b)(a2 + ab+ b2) = a3 − b3 (A)

x =3√

20 + 14√

2 +3√

20− 14√

2 =⇒

x3 = (20 + 14√

2) + 33√

(20 + 14√

2)2(20− 14√

2)

+ 33√

(20 + 14√

2)(20− 14√

2)2 + (20− 14√

2) =⇒

x3 = (40) + 33√

8(20 + 14√

2) + 33√

8(20− 14√

2) =⇒

x3 = (40) + 6(3√

20 + 14√

2 +3√

20− 14√

2) =⇒

x3 = 40 + 6x =⇒x3 − 6x− 40 = 0 =⇒x3 − 64− 6x+ 24 = 0

Neste ponto deve ser aplicado o produto notavel em (A):

(x− 4)(x2 + 4x+ 16)− 6(x− 4) = 0 =⇒

(x− 4)(x2 + 4x+ 10) = 0

∴ ha raiz somente para x = 4.

J. Bartasevicius 73

Page 74: bartasevicius.com.brbartasevicius.com.br/sites/default/files/2018-12/... · Sobre este trabalho. E permitido: • Compartilhar - copiar e redistribuir o material em qualquer meio

CAPITULO 2. SOLUCOES

Solucao 21Aplicando as propriedades de raızes da equacao tem-se:

x1 + x2 + x3 = 0 =⇒ x1 = −(x2 + x3) (A)

ex1x2 + x1x3 + x2x3 = a =⇒ x1 = −(x2 + x3) (B)

Substituindo (A) em (B):

−(x2 + x3)x2 − (x2 + x3)x3 + x2x3 = a =⇒

−(x22 + x2x3 + x2

3) = a (C)

Nenhuma das raızes e nula, pois b 6= 0. Para a equacao em (C) ha duashipoteses:

a. x2, x3 > 0 ou x2, x3 < 0

Neste caso, x2x3 > 0 =⇒ −(x22 + x2x3 + x2

3) < 0 e, portantoa < 0.

b. (x2 > 0 ∧ x3 < 0) ou (x2 < 0 ∧ x3 > 0)

Neste caso, x2x3 < 0. Se |x2| > |x3|, entao x22 > |x2x3| =⇒

−(x22 + x2x3) < 0.

Analogamente, −(x23 + x2x3) < 0 quando |x3| > |x2|.

Como x22 e x2

3 sao sempre maiores que zero, conclui-se que −(x22 +

x2x3 + x23) < 0

∴ a < 0

Solucao 22Hipotese: 11 . . . 1︸ ︷︷ ︸

(n−1) vezes

222 . . . 2︸ ︷︷ ︸n vezes

5 = x2, com x ∈ N.

x2 e um quadrado perfeito, pois x ∈ N. Entao 32x2 tambem sera umquadrado perfeito.Se for demonstrado que 32x2 e um quadrado perfeito, entao a hipotesee verdadeira.Multiplicando 11 . . . 1︸ ︷︷ ︸

(n−1) vezes

222 . . . 2︸ ︷︷ ︸n vezes

5 por 9, tem-se:

1 000 . . . 01︸ ︷︷ ︸(n−1)algs

000 . . . 02︸ ︷︷ ︸n algs

5 = 32x2

74 J. Bartasevicius

Page 75: bartasevicius.com.brbartasevicius.com.br/sites/default/files/2018-12/... · Sobre este trabalho. E permitido: • Compartilhar - copiar e redistribuir o material em qualquer meio

CAPITULO 2. SOLUCOES

1000 . . . 0︸ ︷︷ ︸(n−1)algs

1000 . . . 0︸ ︷︷ ︸n algs

25 = 32x2

102n + 10n+1 + 25 = 102n + 2.5.10n + 52 = 32x2

(10n + 5)2 = 32x2 (Quadrado Perfeito!)

10n + 5 e multiplo de 3, pois a soma dos algarismos e 6. Isto implica xser um numero natural.∴ se 32x2 e um quadrado perfeito, entao a hipotese e verdadeira.

Solucao 23

f(x+ y) =156x+y + 156−(x+y)

2=⇒

f(x+ y) =156x.156y + 156−x.156−y

2(A)

f(x− y) =156x−y + 156−(x−y)

2=⇒

f(x− y) =156x.156−y + 156−x.156y

2(B)

Fazendo (A)+(B):

f(x+ y) + f(x− y) =

=156x.156y + 156−x.156−y + 156x.156−y + 156−x.156y

2=⇒

f(x+ y) + f(x− y) =

=156x.(156y + 156−y) + 156−x(156−y + 156y)

2=⇒

f(x+ y) + f(x− y) =(156x + 156−x).(156y + 156−y)

2=⇒

Mas(156x+156−x)

2 = f(x) e(156y+156−y)

2 = f(y).Portanto:

f(x+ y) + f(x− y) = f(x).f(y).2

J. Bartasevicius 75

Page 76: bartasevicius.com.brbartasevicius.com.br/sites/default/files/2018-12/... · Sobre este trabalho. E permitido: • Compartilhar - copiar e redistribuir o material em qualquer meio

CAPITULO 2. SOLUCOES

Solucao 24

detAn =

∣∣∣∣∣∣∣∣∣∣∣∣

2 −1 0 0 . . . 0 0−1 2 −1 0 . . . 0 00 −1 2 −1 . . . 0 0. . . . . . . . . . . . . . . . . . . . .0 0 0 0 . . . 2 −10 0 0 0 . . . −1 2

∣∣∣∣∣∣∣∣∣∣∣∣n×n

Multiplicando a segunda coluna por 2 e somando a 1ª coluna com a 2ª:

2× detAn =

∣∣∣∣∣∣∣∣∣∣∣∣

2 −2 0 0 . . . 0 0−1 4 −1 0 . . . 0 00 −2 2 −1 . . . 0 0. . . . . . . . . . . . . . . . . . . . .0 0 0 0 . . . 2 −10 0 0 0 . . . −1 2

∣∣∣∣∣∣∣∣∣∣∣∣n×n

=

=

∣∣∣∣∣∣∣∣∣∣∣∣

2 0 0 0 . . . 0 0−1 3 −1 0 . . . 0 00 −2 2 −1 . . . 0 0. . . . . . . . . . . . . . . . . . . . .0 0 0 0 . . . 2 −10 0 0 0 . . . −1 2

∣∣∣∣∣∣∣∣∣∣∣∣n×n

Multiplicando a terceira coluna por 3 e somando a 1ª coluna com a 3ª:

2× 3× detAn =

∣∣∣∣∣∣∣∣∣∣∣∣

2 0 0 0 . . . 0 0−1 3 −3 0 . . . 0 00 −2 6 −1 . . . 0 0. . . . . . . . . . . . . . . . . . . . .0 0 0 0 . . . 2 −10 0 0 0 . . . −1 2

∣∣∣∣∣∣∣∣∣∣∣∣n×n

=

=

∣∣∣∣∣∣∣∣∣∣∣∣

2 0 0 0 . . . 0 0−1 3 0 0 . . . 0 00 −2 4 −1 . . . 0 0. . . . . . . . . . . . . . . . . . . . .0 0 0 0 . . . 2 −10 0 0 0 . . . −1 2

∣∣∣∣∣∣∣∣∣∣∣∣n×n

=

76 J. Bartasevicius

Page 77: bartasevicius.com.brbartasevicius.com.br/sites/default/files/2018-12/... · Sobre este trabalho. E permitido: • Compartilhar - copiar e redistribuir o material em qualquer meio

CAPITULO 2. SOLUCOES

Fazendo continuamente ate o ultimo elemento n:

2× 3× . . .× n× detAn =

=

∣∣∣∣∣∣∣∣∣∣∣∣

2 0 0 0 . . . 0 0−1 3 0 0 . . . 0 00 −2 4 0 . . . 0 0. . . . . . . . . . . . . . . . . . . . .0 0 0 0 . . . n 00 0 0 0 . . . −(n− 1) n+ 1

∣∣∣∣∣∣∣∣∣∣∣∣n×n

Como a matriz esta com zeros em seu “triangulo superior”, o determi-nante e o produto de sua diagonal:

2× 3× . . .× n× detAn = 2× 3× . . .× n× (n+ 1) =⇒

detAn =2× 3× . . .× n× (n+ 1)

2× 3× . . .× n=⇒

detAn = n+ 1

Solucao 25Angulos em progressao aritmetica, com razao ρ: α− ρ, α e α + ρ.

(α− ρ) + (α) + (α + ρ) = π =⇒ α =π

3(sinx+ cosx)(1− sinx cosx) = 1 =⇒

(sin2 x+ 2 sinx cosx+ cos2 x)(1− 2 sinx cosx+ sin2 x cos2 x) = 1 =⇒(1 + 2 sinx cosx)(1− 2 sinx cosx+ sin2 x cos2 x) = 1

Fazendo sinx cosx = a

(1 + 2a)(1− 2a+ a2) = 1 =⇒

2a3 − 3a2 = 0 =⇒

a = 0 ∨a = 3

2(impossıvel, pois sinx cosx < 1)

sinx cosx = 0 =⇒

sinx = 0 =⇒ x = 0(impossıvel)∨cosx = 0 =⇒ x = π

2

∴ os angulos do triangulo sao uma PA com ρ = π6 :

π

6,π

3eπ

2.

J. Bartasevicius 77

Page 78: bartasevicius.com.brbartasevicius.com.br/sites/default/files/2018-12/... · Sobre este trabalho. E permitido: • Compartilhar - copiar e redistribuir o material em qualquer meio

CAPITULO 2. SOLUCOES

Solucao 26k e primo, portanto deve estar presente nos fatores de x ou y, dado quex ∧ y ∈ Z.Como so ha operacoes de soma e multiplicacao, pode-se escolher x ou ylivremente, e servir analogamente para o outro caso.Portanto, k primo =⇒ x = ak, com a ∈ Z.Substituindo:

k(ak + y) = aky (÷k)

ak + y = ay =⇒ y = ka

a− 1

As possibilidades de y ∈ Z sao:

a. a | (a− 1) =⇒ |a− 1| = 1 ∴ a = 2 ∨ a = 0 =⇒ (x = 2k ∧ y =2k) ∨ (a = 0 ∨ y = 0); ou

b. k = a− 1 =⇒ x = (k + 1)k ∧ y = k + 1

Portanto S = (0, 0), (2k, 2k), ((k + 1)k, k + 1) | k e primo.

Solucao 27

aK

bK

A = aKbK2

a1 = 30 cm e b1 = 42 cm =⇒ A1 = 630 cm2

AK+1 =aK+1bK+1

2=

23aK

45bK

2=

8

15AK

∴ Soma das areas e uma progressao geometrica de razao 815

S = 630 + 630

(8

15

)1

+ 630

(8

15

)2

+ 630

(8

15

)3

+ . . .

S

630= 1 +

(8

15

)1

+

(8

15

)2

+

(8

15

)3

+ . . .

S

630=

8

15

((8

15

)−1

+ 1 +

(8

15

)1

+

(8

15

)2

+ . . .

)

78 J. Bartasevicius

Page 79: bartasevicius.com.brbartasevicius.com.br/sites/default/files/2018-12/... · Sobre este trabalho. E permitido: • Compartilhar - copiar e redistribuir o material em qualquer meio

CAPITULO 2. SOLUCOES

S

630=

8

15

((8

15

)−1

+S

630

)S

630

7

15= 1

S = 1350 cm2

Solucao 28

log3 α3 +

log3 β

log3 9= log3 310 =⇒

log3 α3 + log3

√β = log3 310 =⇒

α3.√β = 310 =⇒

α6.β = 320 (A)

log3 α

log3 9− log3 β

2 = log3 310 =⇒

log3√α− log3 β

2 = log3 310 =⇒√α.β−2 = 310 =⇒

α.β−4 = 320 (B)

Fazendo α = 3a e β = 3b, tem-se das equacoes (A) e (B):6a+ b = 20

a− 4b = 20=⇒ a = 4 e b = −4

∴ P = αβ = 3a3b = 3a+b = 1

Solucao 29

logsinx+cosx (1 + sin 2x) = 2 =⇒ (sinx+ cosx)2 = 1 + sin 2x =⇒

sin2 x+ 2 sinx cosx+ cos2 x = 1 + 2 sinx cosx =⇒

1 + 2 sinx cosx = 1 + 2 sinx cosx

J. Bartasevicius 79

Page 80: bartasevicius.com.brbartasevicius.com.br/sites/default/files/2018-12/... · Sobre este trabalho. E permitido: • Compartilhar - copiar e redistribuir o material em qualquer meio

CAPITULO 2. SOLUCOES

Portanto a equacao e valida para qualquer valor de x que satisfaca oDomınio da equacao.Sao condicoes do logaritmo a base positiva e diferente de 1, e o logarit-mando maior que zero:

sinx+ cosx > 0 ∧ sinx+ cosx 6= 1

∧1 + sin 2x > 0

• Resolvendo sinx+ cosx > 0:

sinx > − cosx =⇒ −π4< x 6

π

2

• Resolvendo sinx+ cosx 6= 1:

sinx+ cosx 6= 1 =⇒

sinx 6= 1

∨cosx 6= 1

sinx 6= 1 =⇒ x 6= π

2cosx 6= 1 =⇒ x 6= 0

• Resolvendo 1 + sin 2x > 0:

sin 2x > −1 =⇒ 2x 6=−π

2,3π

2

=⇒ x 6=

−π

4,3π

4

∴ S =

x ∈ R | −π

4< x <

π

2∧ x 6= 0 ∧ x 6= 3π

4

Solucao 30Numa sequencia simetrica, a metade direita da sequencia deve espelhara metade esquerda. Por este motivo, m e n nao podem ser ambosımpares.Ha tres possibilidades:

a. m e n paresComo ambos sao pares, entao ha uma divisao exata entre as bolasbrancas e pretas.

80 J. Bartasevicius

Page 81: bartasevicius.com.brbartasevicius.com.br/sites/default/files/2018-12/... · Sobre este trabalho. E permitido: • Compartilhar - copiar e redistribuir o material em qualquer meio

CAPITULO 2. SOLUCOES

Somente uma das metades e independente para formar a sequencia.Portanto, o numero de sequencias com metade das bolas brancas epretas e:

C(m+n)/2m/2

b. m par e n ımparNeste caso, uma bola branca deve ficar localizada no centro paraque possa haver simetria, e em cada metade da sequencia haveram2 bolas pretas e n−1

2 bolas brancas.O numero de sequencias e:

C(m+n−1)/2(n−1)/2

c. m ımpar e n parAnalogamente ao caso anterior, uma bola preta deve se localizarao centro da sequencia.O numero de sequencias e:

C(m+n−1)/2(m−1)/2

Solucao 31

a+ b

c+ 1 =

b+ c

a+ 1 =

a+ c

b+ 1 =⇒

a+ b+ c

c=a+ b+ c

a=a+ b+ c

b=⇒

1

c=

1

a=

1

b=⇒

a = b = c

∴a+ b

c= 2

Solucao 32

2006∑k=0

f(k) = f(1) + f(2) + . . .+ f(2005) + f(2006) =⇒

J. Bartasevicius 81

Page 82: bartasevicius.com.brbartasevicius.com.br/sites/default/files/2018-12/... · Sobre este trabalho. E permitido: • Compartilhar - copiar e redistribuir o material em qualquer meio

CAPITULO 2. SOLUCOES

2005∑k=0

f(k) + f(2006) = 20082007

2008=⇒

f(2006) = 20082007

2008− 2008

2006

2007=⇒

f(2006) =1

2007

∴1

f(2006)= 2007

Solucao 33

(sinx+cosx)(sin2 x−sinx cosx+cos2 x) = (1+sin x cosx)(1−sinx cosx)

sin2 x+ cos2 x = 1, entao:

(sinx+ cosx)(1− sinx cosx) = (1 + sinx cosx)(1− sinx cosx) =⇒

sinx+ cosx = 1 + sinx cosx =⇒ (A)

sin2 x+ 2 sinx cosx+ cos2 x = 1 + 2 sinx cosx+ sin2 x cos2 x =⇒

Quando se elevou (A) ao quadrado, foram incluıdas solucoes que satisfi-zessem

− sinx− cosx = 1 + sinx cosx (B)

As solucoes que satisfazem as equacoes em (B) devem ser descartadas.

sin2 x cos2 x = 0 =⇒

cosx = 0 ∨ sinx = 0

Neste caso, ha as seguintes solucoes possıveis para 0 6 x 6 2π

x = 0, x = π ∨ x =π

2, x =

2

Mas x = π e x = 3π2 satisfazem a equacao em (B), portanto nao podem

ser solucoes.

∴ S = x | x = 2kπ ∪ x =π

2+ 2kπ, k ∈ Z

82 J. Bartasevicius

Page 83: bartasevicius.com.brbartasevicius.com.br/sites/default/files/2018-12/... · Sobre este trabalho. E permitido: • Compartilhar - copiar e redistribuir o material em qualquer meio

CAPITULO 2. SOLUCOES

Solucao 34(Q(x) + 2) |x4 e Q(x) e de 6º grau.Logo, Q(x) = x4

(ax2 + bx+ c

)− 2.

De acordo com o enunciado:

Q(x) + 1 = (x− 1)3 .P (x) =⇒ax6 + bx5 + cx4 − 2 + 1 = (x− 1)3 .P (x) (A)

Fazendo x = 1 na equacao em (A):

a16 + b15 + c14 − 1 = (1− 1)3︸ ︷︷ ︸=0

.P (1) =⇒

a+ b+ c = 1 (B)

Dividindo ax6 + bx5 + cx4 − 1 por x− 1, tem-se:

ax6 + bx5 + cx4 − 1

x− 1=[ax5 + (a+ b)x4 + (a+ b+ c)x3+

+ (a+ b+ c)x2 + (a+ b+ c)x+

+(a+ b+ c)] + [−(a+ b+ c)x+ 1] (C)

O polinomio em (C) e divisıvel por (x− 1)2. Portanto, fazendo x = 1 esubstituindo a igualdade em (B), tem-se:a15 + (a+ b)14︸ ︷︷ ︸

=2a+b

+ (1)13 + (1)12 + (1)1 + (1)︸ ︷︷ ︸=4

+ [−(1)1 + 1]︸ ︷︷ ︸=0

= 0 =⇒

2a+ b = −4 (D)

Substituindo (B) no polinomio em (C):[ax5 + (a+ b)x4 + (1)x3 + (1)x2 + (1)x+ (1)

]+ [−(1)x+ 1] =

= ax5 + (a+ b)x4 + x3 + x2 + x+ 1 (E)

Dividindo (E) por x− 1, tem-se:

ax5 + (a+ b)x4 + x3 + x2 + x+ 1

x− 1= ax4 + (2a+ b)x3+

+ (2a+ b+ 1)x2+

+ (2a+ b+ 2)x+ (2a+ b+ 3)(F)

J. Bartasevicius 83

Page 84: bartasevicius.com.brbartasevicius.com.br/sites/default/files/2018-12/... · Sobre este trabalho. E permitido: • Compartilhar - copiar e redistribuir o material em qualquer meio

CAPITULO 2. SOLUCOES

O polinomio em (F) e divisıvel por (x− 1). Portanto, fazendo x = 1 esubstituindo a igualdade em (D), tem-se:

a14 + (−4)13 + (−4 + 1)12 + (−4 + 2)1 + (−4 + 3) = 0 =⇒ a = 10

Substituindo a = 10 em (D):

2(10) + b = −4 =⇒ b = −24

Substituindo a e b em (B):

(10) + (−24) + c = 1 =⇒ c = 15

Substituindo os valores de a, b e c em (A):

10x6 + (−24)x5 + (15)x4 − 1 = (x− 1)3 .P (x) =⇒

P (x) =10x6 + (−24)x5 + (15)x4 − 1

(x− 1)3=⇒

P (x) = 10x3 + 6x2 + 3x+ 1

Solucao 35Inicialmente, a soma deve ser dividida em quatro somas menores, man-tendo os expoentes de i com o mesmo resto de divisao por 4:

a. im.4 = 1

b. im.4+1 = i

c. im.4+2 = −1

d. im.4+3 = −i

Somas dos expoentes m.4:

S0 = 1 + 5 + 9 + . . .+ (n+ 1) =(n+ 2)

2

(n4

+ 1)

=⇒

S0 =(n+ 2)(n+ 4)

8(A)

84 J. Bartasevicius

Page 85: bartasevicius.com.brbartasevicius.com.br/sites/default/files/2018-12/... · Sobre este trabalho. E permitido: • Compartilhar - copiar e redistribuir o material em qualquer meio

CAPITULO 2. SOLUCOES

Somas dos expoentes m.4+1:

S1 = 2i+ 6i+ 10i+ . . .+ (n− 2)i =(ni)

2

((n− 4)

4+ 1

)=⇒

S1 =n2

8i (B)

Somas dos expoentes m.4+2:

S2 = −3− 7− 11− . . .− (n− 1) = −(n+ 2)

2

((n− 4)

4+ 1

)=⇒

S2 = −(n+ 2)n

8(C)

Somas dos expoentes m.4+3:

S3 = −4i− 8i− 12i− . . .− (ni) = −(n+ 4)

2

((n− 4)

4+ 1

)=⇒

S3 = −(n+ 4)n

8i (D)

Somando (A)+(B)+(C)+(D):

S0 + S1 + S2 + S3 =2n+ 1

2− in

2

Solucao 36

x2 − 6x+ 9 + y2 − 6y + 9 = 4 =⇒

(x− 3)2 + (y − 3)2 = 22 (A)

A equacao acima representa uma circunferencia de centro em (3,3), comraio 2, conforme representado no plano xy:

J. Bartasevicius 85

Page 86: bartasevicius.com.brbartasevicius.com.br/sites/default/files/2018-12/... · Sobre este trabalho. E permitido: • Compartilhar - copiar e redistribuir o material em qualquer meio

CAPITULO 2. SOLUCOES

1

3

5

P

1 3 5O x

α

yx e tanα, onde α e o angulo POx.Para obter tanα maximo, entao α deve ser maximo.α maximo implica a reta y = kx tangenciar a circunferencia.Fazendo tanα = k, e substituindo y = kx em (A):

(x− 3)2 + (kx− 3)2 = 22 =⇒

(k2 + 1)x2 − (6 + 6k)x+ 14 = 0

Para tangenciar a circunferencia, a equacao deve produzir apenas umasolucao.Fazendo ∆ = 0:

(−(6 + 6k))2 − 4.(k2 + 1).14 = 0

5k2 − 18k + 5 = 0 =⇒

k =9± 2

√14

5

Mas k e maximo. Entao:

k =9 + 2

√14

5.

Solucao 37

2 + sin2 x+ 4 sinx− sinx cosx− 4√

2 sinx cosx+ 4 cosx− 2− 2√

2

2 sinx− 2√

2 sinx cosx+ 2 cosx−√

2> 2

86 J. Bartasevicius

Page 87: bartasevicius.com.brbartasevicius.com.br/sites/default/files/2018-12/... · Sobre este trabalho. E permitido: • Compartilhar - copiar e redistribuir o material em qualquer meio

CAPITULO 2. SOLUCOES

sin2 x− sinx cosx+ 2(2 sinx− 2√

2 sinx cosx+ 2 cosx−√

2)

2 sinx− 2√

2 sinx cosx+ 2 cosx−√

2> 2

sinx(sinx− cosx)

2 sinx− 2√

2 sinx cosx+ 2 cosx−√

2+ 2 > 2

sinx(sinx− cosx)

2 sinx− 2√

2 sinx cosx+ 2 cosx−√

2> 0

sinx(sinx− cosx)

2 sinx(1−√

2 cosx) +√

2(√

2 cosx− 1)> 0

sinx(sinx− cosx)

2(sinx− 1)(1−√

2 cosx)> 0

Aplicando a regra de sinais em 0 6 x 6 2π:+ + + + − − − −

− + + + + − − −

− + + − − − − −

− + + + + + + −

− + + − + − − +

sinx

sinx− cosx

2 sinx−√

2

1−√

2 cosxsinx(sinx−cosx)

2(sinx−1)(1−√

2 cosx)

0 π4

π2

3π4

π 5π4

3π2

7π4

S =

x ∈ R | π

46 x 6

4∪ π 6 x 6

4∪ 7π

46 x 6 2π

Solucao 38Sejam os dados A,B e C e os seguintes eventos:

• EVENTO 1: O dado C e a soma dos dados A e B:

2 6 DA +DB 6 6

• EVENTO 2: O dado B e a soma dos dados A e C:

2 6 DA +DC 6 6

J. Bartasevicius 87

Page 88: bartasevicius.com.brbartasevicius.com.br/sites/default/files/2018-12/... · Sobre este trabalho. E permitido: • Compartilhar - copiar e redistribuir o material em qualquer meio

CAPITULO 2. SOLUCOES

• EVENTO 3: O dado A e a soma dos dados B e C:

2 6 DB +DC 6 6

Assim, os eventos 1, 2 e 3 sao analogos. Como no dado nao ha ladozero, tambem sao mutuamente excludentes.Ao analisar a incidencia do evento 1, pode-se multiplicar por 3 paraobter o total de incidencias para os tres dados.EVENTO 1:

DC numero de incidencias de DA +DB = DC2 1 (DA = 1 e DB = 1)3 2 (DA = 1 e DB = 2 ou DA = 2 e DB = 1)4 3 (1,3 ou 2,2 ou 3,1)5 4 (1,4 ou 2,3 ou 3,2 ou 4,1)6 5 (1,5 ou 2,4 ou 3,3 ou 4,2 ou 5,1)

O numero total de incidencias dos eventos e 15× 3 = 45 e o numerototal de combinacoes de dados e 63 = 216.Portanto a probabilidade de a soma de dois dados ser o valor do terceiroe:

P =45

216=

5

24.

Solucao 39

pn = (q − 12)(q + 12)

Sendo p primo, entao:q − 12 = pa

q + 12 = pb, a+ b = n =⇒

q = pa + 12

q = pb − 12=⇒

pa + 12 = pb − 12 =⇒ pb − pa = 24

∴ pa(pb−a − 1) = 23.3 (A)

Se p e primo, existem duas hipoteses: p = 2 ou p e ımpar.

88 J. Bartasevicius

Page 89: bartasevicius.com.brbartasevicius.com.br/sites/default/files/2018-12/... · Sobre este trabalho. E permitido: • Compartilhar - copiar e redistribuir o material em qualquer meio

CAPITULO 2. SOLUCOES

1. p = 2:Se p = 2, entao, a partir de (A), pa = 23, pois pb−a − 1 deve serımpar e, portanto pb−a − 1 = 3.Chega-se aos valores p = 2, a = 3, b = 5 e q = 23 + 12 = 20.

2. p ımpar:Se p for ımpar, entao, a partir de (A), p = 3, a = 1, 3b−1 − 1 =23 =⇒ b = 3 e, portanto, q = 31 + 12 = 15.

Solucao 40

2x − x2 ≡ 0 mod 7

Montando a tabela de restos da divisao por 7, tem-se:

x 2x

0 mod 7 + 11 mod 7 + 22 mod 7 + 43 mod 7 + 14 mod 7 + 2...

...

x x2 −x2

0 mod 7 + 0 mod 7 + 01 mod 7 + 1 mod 7 + 62 mod 7 + 4 mod 7 + 33 mod 7 + 2 mod 7 + 54 mod 7 + 2 mod 7 + 55 mod 7 + 4 mod 7 + 36 mod 7 + 1 mod 7 + 67 mod 7 + 0 mod 7 + 0

Observa-se que o ciclo de restos na divisao de 7 de 2x ocorre em multiplosde 3 e o ciclo para x2 ocorre em multiplos de 7.Portanto, 2x − x2 e multiplo de 7 se:

a. x = m.3 ∧ (x = m.7 + 1 ∨ x = m.7 + 6)

b. x = m.3 + 1 ∧ (x = m.7 + 3 ∨ x = m.7 + 4)

c. x = m.3 + 2 ∧ (x = m.7 + 2 ∨ x = m.7 + 5)

J. Bartasevicius 89

Page 90: bartasevicius.com.brbartasevicius.com.br/sites/default/files/2018-12/... · Sobre este trabalho. E permitido: • Compartilhar - copiar e redistribuir o material em qualquer meio

CAPITULO 2. SOLUCOES

Do item a.:

I. x = m.3 ∧ x = m.7 + 1 =⇒ x = m.21 + 15

II. x = m.3 ∧ x = m.7 + 6 =⇒ x = m.21 + 6

Do item b.:

I. x = m.3 + 1 ∧ x = m.7 + 3 =⇒ x = m.21 + 10

II. x = m.3 + 1 ∧ x = m.7 + 4 =⇒ x = m.21 + 4

Do item c.:

I. x = m.3 + 2 ∧ x = m.7 + 2 =⇒ x = m.21 + 2

II. x = m.3 + 2 ∧ x = m.7 + 5 =⇒ x = m.21 + 5

Entao, 2x − x2 e multiplo de 7 se x = m.21 + 2, 4, 5, 6, 10, 15, o queimplica haver 6 numeros que satisfazem a condicao a cada 21 numeros.Sendo x 6 20000, o ultimo multiplo de 21 e 19992. Apos 19992 restamos numeros 19994, 19996, 19997 e 19998 que satisfazem a condicao.∴ o numero de possıveis solucoes de x e:

19992

21.6 + 4 = 5716

Solucao 41

x3 − ax2 +b

2x− ab = 0

Pelas propriedades das raızes da equacao tem-se:

x1x2x3 = ab

x1x2 + x1x3 + x2x3 =b

2x1 + x2 + x3 = a

Para calcular x21 + x2

2 + x23, e necessario transformar para:

x21 + x2

2 + x23 = (x1 + x2 + x3)2 − 2 (x1x2 + x1x3 + x2x3)

x21 + x2

2 + x23 = a2 − 2

b

2= a2 − b

90 J. Bartasevicius

Page 91: bartasevicius.com.brbartasevicius.com.br/sites/default/files/2018-12/... · Sobre este trabalho. E permitido: • Compartilhar - copiar e redistribuir o material em qualquer meio

CAPITULO 2. SOLUCOES

Calculando a solucao:

loga

[ab (a)a

2−b]b

= loga aba2 = ba2

Solucao 42Hipotese: 2r + 3s e multiplo de 17.Entao, 9× (2r + 3s) ≡ 0 mod 17.

18r + 27s = 17(r + s) + (r + 10s)

17(r + s) + (r + 10s) ≡ 0 mod 17 =⇒

r + 10s ≡ 0 mod 17

Portanto, 2r + 3s e multiplo de 17 se, e somente se r + 10s for multiplode 17.Ora, se r + 10s e multiplo de 17, entao 9× (r + 10s) e multiplo de 17tambem:

9r + 90s ≡ 0 mod 17 =⇒

5.17s+ 9r + 5s ≡ 0 mod 17 =⇒

9r + 5s ≡ 0 mod 17

Concluindo, a hipotese de 2r + 3s ser multiplo de 17 e valida se, esomente se 9r + 5s for multiplo de 17 tambem.

Solucao 43Raızes em progressao aritmetica: x1 − δ, x1 e x1 + δ.Pelas propriedades das raızes na equacao:

x1 − δ + x1 + x1 + δ = 18 =⇒ x = 6

log√ba2 = 4 =⇒ b2 = a2

Sendo a, b ∈ R∗+, entao a = b.Aplicando novamente as propriedades das raızes na equacao:

(6− δ)(6)(6 + δ) = logb(a)2m =⇒ 36− δ2 =m

3(A)

J. Bartasevicius 91

Page 92: bartasevicius.com.brbartasevicius.com.br/sites/default/files/2018-12/... · Sobre este trabalho. E permitido: • Compartilhar - copiar e redistribuir o material em qualquer meio

CAPITULO 2. SOLUCOES

(6− δ)(6) + (6− δ)(6 + δ) + (6)(6 + δ) = logb(ab)m + 8−m =⇒

36− 6δ + (36− δ2) + 36 + 6δ = 2m+ 8−m =⇒

=⇒ 72 + (36− δ2) = m+ 8 (B)

Substituindo (A) em (B):

72 +m

3= m+ 8 =⇒ m = 96

Solucao 44

−by + cz = 0 =⇒ by = cz

Substituindo em 3ax− 4by e dividindo o sistema, tem-se:ax+ cz = 2abc

3ax− 4cz = −abc=⇒ x = bc

3ax− 4by = −abc =⇒ 3a(bc)− 4by = −abc =⇒ y = ac

−b(ac) + cz = 0 =⇒ z = ab

Pelo enunciado, a, b, c ∈ Z, com 2 < a < b < c.Portanto, x, y, z ∈ N, com x > y > z.

xyz = 20132 =⇒ (abc)2 = (3.11.61)2 =⇒ abc = 3.11.61

Assim, a = 3, b = 11 e c = 61.

x = 671

y = 183

z = 33

Solucao 45

x2 + px+ q = 0 =⇒

Z1 =−p−√p2−4q

2 e

Z2 =−p+√p2−4q

2

(A)

A diferenca entre os argumentos de Z1 e Z2 e α, e ambos possuem omesmo modulo: Z1 = m ϕ e Z2 = m ϕ± α.

92 J. Bartasevicius

Page 93: bartasevicius.com.brbartasevicius.com.br/sites/default/files/2018-12/... · Sobre este trabalho. E permitido: • Compartilhar - copiar e redistribuir o material em qualquer meio

CAPITULO 2. SOLUCOES

Portanto, a divisao entre as representacoes polares de Z1 e Z2 resultanum valor de modulo 1, com argumento α.Como o interesse e determinar cos2

(α2

), entao e possıvel obter o valor

de cos(α

2

)ao calcular o valor da parte real de

√Z1Z2

:√Z1

Z2=

√m ϕ

m ϕ± α= 1 ∓α2 =⇒

√Z1

Z2= cos

(∓α

2

)︸ ︷︷ ︸parte real

+i sin(∓α

2

)︸ ︷︷ ︸

parte imaginaria

Na parte real, cos(α

2

)= cos

(−α2), portanto para determinar o resul-

tado, nao ha diferenca entre calcular Z1Z2

ou Z2Z1

.

Substituindo os valores de Z1 e Z2 em (A):

√Z1

Z2=

√√√√√ −p−√p2−4q

2

−p+√p2−4q

2

=

√−p−

√p2 − 4q

−p+√p2 − 4q

Racionalizando:

√Z1

Z2=

√√√√(−p+√p2 − 4q

)2

p2 − p2 + 4q=−p√

4q+

√p2 − 4q√

4q

A parte real e −p√4q

.

∴ cos2(α

2

)=

(−p√

4q

)2

=p2

4q

Solucao 46O polinomio possui 5 raızes. De acordo com o enunciado, sao 4 raızescomplexas e 1 real. Para que todos os coeficientes sejam reais, as raızesdevem ser conjugadas, alem de simetricas. Portanto, as raızes assumemas seguintes formas em coordenada polar:

• x1 = a ϕ

J. Bartasevicius 93

Page 94: bartasevicius.com.brbartasevicius.com.br/sites/default/files/2018-12/... · Sobre este trabalho. E permitido: • Compartilhar - copiar e redistribuir o material em qualquer meio

CAPITULO 2. SOLUCOES

• x2 = −x1 = a ϕ+ π

• x3 = x1 = a −ϕ• x4 = −x3 = a −ϕ− π• x5 = a

O produto das raızes e o valor negativo do termo independente dopolinomio:

x1x2x3x4x5 = a5 = 243 =⇒ a = 3

∴, x5 = 3.P (x)

x− 3= x4 + 10x2 + 81

Raızes de x4 + 10x2 + 81:

x4 + 10x2 + 81 = 0 =⇒ x2 = −5± 2√

14i

Logo:

x = ±√−5± 2

√14i (A)

Para simplificar a expressao, deve-se procurar fazer

−5± 2√

14i =(A±B

√14i)2

=⇒

AB = 1

A2 − 14B2 = −5=⇒

A = ±√

2 ou ±√

7

B = ±√

1

2ou B = ±

√1

7

Adotando A = +√

2 e B = +

√1

2, tem-se:

−5± 2√

14i =

(√

2±√

1

2

√14i

)2

=⇒

−5± 2√

14i =(√

2±√

7i)2

(B)

Substituindo (B) em (A):

x = ±(√

2±√

7i)

∴ S =

2,√

2 +√

7i,√

2−√

7i,−√

2 +√

7i,−√

2−√

7i

94 J. Bartasevicius

Page 95: bartasevicius.com.brbartasevicius.com.br/sites/default/files/2018-12/... · Sobre este trabalho. E permitido: • Compartilhar - copiar e redistribuir o material em qualquer meio

CAPITULO 2. SOLUCOES

Solucao 47

y−1∏z=0

(y − z) = (y − 0)(y − 1)(y − 2) . . . (y − (y − 1)) =⇒

y−1∏z=0

(y − z) = y!

Aplicando a equacao a ser resolvida:

x2 =x∑y=1

y! = 1! + 2! + 3! + . . .+ x!

O fator x! e crescente e possui ordem de grandeza comparavel a xx,de modo que as solucoes possıveis devem observar apenas as hipotesesenquanto 1! + 2! + 3! + . . .+ x! 6 x2.

• x = 112 = 1 e 1! = 1, ∴ x = 1 e solucao.

• x = 222 = 4 e 1! + 2! = 3, ∴ x = 2 nao e solucao.

• x = 332 = 9 e 1! + 2! + 3! = 9, ∴ x = 3 e solucao.

• x = 442 = 16 e 1! + 2! + 3! + 4! = 33, ∴ valores x > 4 nao sao solucao.

∴ x ∈ 1, 3.

Solucao 48

2 logcosx sinx.logcosx sinx

logcosx cos2 x= 4

2 logcosx sinx.logcosx sinx

2= 4

log2cosx sinx = 4 =⇒

logcosx sinx = 2

ou

logcosx sinx = −2

J. Bartasevicius 95

Page 96: bartasevicius.com.brbartasevicius.com.br/sites/default/files/2018-12/... · Sobre este trabalho. E permitido: • Compartilhar - copiar e redistribuir o material em qualquer meio

CAPITULO 2. SOLUCOES

• logcosx sinx = 2

logcosx sinx = 2 =⇒ cos2 x = sinx =⇒

1− sin2 x = sinx =⇒ sin2 x+ sinx− 1 = 0

sinx =−1±

√5

2

Por definicao, −1 > sinx > 1. Entao sinx = −1+√

52 .

• logcosx sinx = −2

logcosx sinx = −2 =⇒ cos−2 x = sinx =⇒

cos2 x =1

sinx

Por definicao, −1 > sinx > 1 e −1 > cosx > 1. Excetuando-seos valores em que sinx = ±1, os demais valores de x produzem

1sinx > 1, e consequentemente implica cosx > 1. Impossıvel,portanto.Quando sinx = ±1, cosx = 0. Portanto nao ha valor de x para aigualdade logcosx sinx = −2.

∴ x = arcsin

(√5− 1

2

)

Solucao 49

3

√37 037037 . . . 037︸ ︷︷ ︸

87 algs

− 11 . . . 1︸ ︷︷ ︸30 algs

00 . . . 0︸ ︷︷ ︸30 algs

(A)

37 037037 . . . 037︸ ︷︷ ︸87 algs

= 37× 1 001001 . . . 001︸ ︷︷ ︸87 algs

=

= 37×(

100 + 103 + 106 + . . .+ 1087)

=⇒

37 037037 . . . 037︸ ︷︷ ︸87 algs

= 37× 1090 − 1

33.37=

1090 − 1

33(B)

11 . . . 1︸ ︷︷ ︸30 algs

00 . . . 0︸ ︷︷ ︸30 algs

=(

100 + 103 + 106 + . . .+ 1029)× 1030 =⇒

96 J. Bartasevicius

Page 97: bartasevicius.com.brbartasevicius.com.br/sites/default/files/2018-12/... · Sobre este trabalho. E permitido: • Compartilhar - copiar e redistribuir o material em qualquer meio

CAPITULO 2. SOLUCOES

11 . . . 1︸ ︷︷ ︸30 algs

00 . . . 0︸ ︷︷ ︸30 algs

=1030 − 1

32× 1030 (C)

Aplicando (B) e (C) em (A):

3

√37 037037 . . . 037︸ ︷︷ ︸

87 algs

− 11 . . . 1︸ ︷︷ ︸30 algs

00 . . . 0︸ ︷︷ ︸30 algs

=3

√1090 − 1

33− 1030 − 1

32× 1030 =

=3

√1090 − 1

33− 1060 − 1030

32=

=3

√1090 − 3× 1060 + 3× 1030 − 1

33=

=3

√(1030 − 1

)333

=1030 − 1

3= 333 . . . 3︸ ︷︷ ︸

29 algs

Solucao 50Existem cinco possibilidades de agrupar os alunos: nenhum grupo de 2ou com 1, 2, 3 e 4 grupos de 2.Analisando as possibilidades de cada uma delas:

• nenhum grupo de 2:So e possıvel haver uma unica combinacao sem grupos de 2 alunos:C9,0 = 1

• 1 grupo de 2:Com 9 alunos e possıvel formar C9,2 grupos diferentes de 2 alunos.Para cada um desses grupos havera C2,2 combinacoes possıveis.Mas as combinacoes sao permutaveis e elas se repetem. Como haum grupo a permutar, divide-se pelo numero de permutacoes com1 elemento: 1!.∴ numero de combinacoes com 1 grupo de 2 e:

C9,2 × C2,2

1!=

(9!)(2!)

(2!7!)(2!0!)1!= 36

J. Bartasevicius 97

Page 98: bartasevicius.com.brbartasevicius.com.br/sites/default/files/2018-12/... · Sobre este trabalho. E permitido: • Compartilhar - copiar e redistribuir o material em qualquer meio

CAPITULO 2. SOLUCOES

• 2 grupos de 2:Com 9 alunos e possıvel formar C9,4 grupos diferentes de 4 alunos.Para cada um desses grupos havera C4,2 combinacoes possıveispara a primeira dupla e, com os dois alunos restantes, C2,2 para asegunda dupla.Mas as combinacoes sao permutaveis e elas se repetem. Como hadois grupos a permutar, divide-se pelo numero de permutacoescom 2 elementos: 2!.∴ numero de combinacoes com 2 grupos de 2 e:

C9,4 × C4,2 × C2,2

2!=

(9!)(4!)(2!)

(4!5!)(2!2!)(2!0!)2!= 378

• 3 grupos de 2:Com 9 alunos e possıvel formar C9,6 grupos diferentes de 6 alunos.Para cada um desses grupos havera C6,2 combinacoes possıveispara a primeira dupla. C4,2 para a segunda dupla, com os 4 alunosrestantes. E C2,2 duplas para os ultimos dois alunos.Mas as combinacoes sao permutaveis e elas se repetem. Como hatres grupos a permutar, divide-se pelo numero de permutacoescom 3 elementos: 3!.∴ numero de combinacoes com 3 grupos de 2 e:

C9,6 × C6,2 × C4,2 × C2,2

3!=

(9!)(6!)(4!)(2!)

(6!3!)(2!4!)(2!2!)(2!0!)3!= 1250

• 4 grupos de 2:Com 9 alunos e possıvel formar C9,8 grupos diferentes de 8 alunos.Para cada um desses grupos havera C8,2 combinacoes possıveispara a primeira dupla. C6,2 combinacoes para a segunda dupla,com os 6 alunos restantes. C4,2 combinacoes para a terceira dupla,com os 4 alunos restantes. E C2,2 duplas para os ultimos doisalunos.Mas as combinacoes sao permutaveis e elas se repetem. Como haquatro grupos a permutar, divide-se pelo numero de permutacoescom 4 elementos: 4!.∴ numero de combinacoes com 4 grupos de 2 e:

C9,8 × C8,2 × C6,2 × C4,2 × C2,2

4!=

=(9!)(8!)(6!)(4!)(2!)

(8!1!)(2!6!)(2!4!)(2!2!)(2!0!)4!= 945

98 J. Bartasevicius

Page 99: bartasevicius.com.brbartasevicius.com.br/sites/default/files/2018-12/... · Sobre este trabalho. E permitido: • Compartilhar - copiar e redistribuir o material em qualquer meio

CAPITULO 2. SOLUCOES

∴ o numero total de combinacoes e

1 + 36 + 378 + 1250 + 945 = 2610

Solucao 51

De√x−√y = log3

y

xobtem-se o Domınio:

(x, y) ∈ R2 | x > 0 ∧ y > 0

.

Nao e possıvel utilizar artifıcios que transformem as equacoes em po-linomios. Portanto deve-se analisar as condicoes entre x e y antes deresolver o sistema.Analise das hipoteses de

y

x> 1 e

y

x< 1:

•y

x> 1:

y

x> 1 =⇒ y > x =⇒

√x−√y < 0 (A)

Aplicando em log3y

x:

y

x> 1 =⇒ log3

y

x> 0 (B)

As conclusoes em (A) e (B) se contradizem.

•y

x< 1:

y

x< 1 =⇒ y < x =⇒

√x−√y > 0 (C)

Aplicando em log3y

x:

y

x< 1 =⇒ log3

y

x< 0 (D)

As conclusoes em (C) e (D) se contradizem.

Por absurdo, resta apenas a hipotese de x = y:

√x−√x = log3

x

x=⇒ 0 = 0 ∴ x = y

Substituindo y por x na segunda equacao:

2x+2 + 8x = 5.4x =⇒ 4.2x + 23x − 5.22x = 0

J. Bartasevicius 99

Page 100: bartasevicius.com.brbartasevicius.com.br/sites/default/files/2018-12/... · Sobre este trabalho. E permitido: • Compartilhar - copiar e redistribuir o material em qualquer meio

CAPITULO 2. SOLUCOES

Fazendo 2x = A:

A3 − 5A2 + 4A = 0 =⇒ A(A2 − 5A+ 4

)= 0 =⇒

A = 0

A = 1

A = 4

A = 0 nao pode ser solucao, pois nao existe x tal que 2x = 0.A = 1 nao pode ser a solucao, pois implica x = 0, que nao esta definidono Domınio.Resta apenas A = 4, implicando x = 2 e y = 2.

∴ S = (2, 2)

Solucao 52

4

2 log3 x− 2+

log3 3−2

log3 x> 1 =⇒

4

2 log3 x− 2− 2

log3 x> 1 =⇒

4 log3 x− 2 (2 log3 x− 2)− log3 x (2 log3 x− 2)

log3 x (2 log3 x− 2)> 0 =⇒

−2 log23 x+ 2 log3 x+ 4

log3 x (2 log3 x− 2)> 0

Fazendo y = log3 x:

−2y2 + 2y + 4

y (2y − 2)> 0 =⇒

−y2 + y + 2

y (y − 1)> 0

Analise de sinais, com y 6= 0, 1:

• −y2 + y + 2:

As raızes sao:

x = −1

x = 2.

Sinais da funcao:

100 J. Bartasevicius

Page 101: bartasevicius.com.brbartasevicius.com.br/sites/default/files/2018-12/... · Sobre este trabalho. E permitido: • Compartilhar - copiar e redistribuir o material em qualquer meio

CAPITULO 2. SOLUCOES

− + −−1 2

−y2 + y + 2

• y:Sinais da funcao:

− +

0y

• y − 1:Sinais da funcao:

− +

1y − 1

Analisando os sinais de−y2 + y + 2

y (y − 1):

−1

−1

0

0

1

1

2

2

−y2 + y + 2

y (y − 1)

y − 1

y

−y2 + y + 2− + + + −

− − + + +

− − − + +

− + − + −

A solucao em y e: −1 < y < 0 ∨ 1 < y < 2.Substituindo y por log3 x:

−1 < log3 x < 0 ∨ 1 < log3 x < 2 =⇒

1

3< x < 1 ∨ 3 < x < 9

∴ S =

x ∈ R | 1

3< x < 1 ∪ 3 < x < 9

J. Bartasevicius 101

Page 102: bartasevicius.com.brbartasevicius.com.br/sites/default/files/2018-12/... · Sobre este trabalho. E permitido: • Compartilhar - copiar e redistribuir o material em qualquer meio

CAPITULO 2. SOLUCOES

Solucao 53Definindo o Domınio da equacao:

4x− 4 > 0 =⇒ x > 1 (A)

x+ 3 > 0 =⇒ x > − 3 (B)

x−√

4x− 4 > 0 =⇒ x > 1 (C)

Domınio = (A) ∩ (B) ∩ (C):

∴ Domınio = x ∈ R | x > 1

Resolvendo a equacao, elevando ambos os termos ao quadrado:

x+√

4x− 4 + 2√x2 − (4x− 4) + x−

√4x− 4 = x+ 3 =⇒

x+ 2√

(x− 2)2 − 3 = 0 =⇒

x+ 2|x− 2| − 3 = 0 (D)

Com |x− 2|, (D) deve ser separada em duas hipoteses:

• x > 2:

x+ 2x− 4− 3 = 0 =⇒ x =7

3

• x < 2:x− 2x+ 4− 3 = 0 =⇒ x = 1

Sendo ambos os valores pertencentes ao Domınio da equacao:

∴ x ∈ 1, 7

3

Solucao 54Inicialmente deve ser calculada a hipoteses de o numero maior que 99ter, no maximo, 4 algarismos:

x = u+ 10d+ 100c+ 1000m, 0 6 u, d, c,m 6 9 ∧ u, d, c,m ∈ N

O valor de f(x), invertendo os algarismos das unidades e dezenas, e oda centena e milhar, sera:

f(x) = d+ 10u+ 100m+ 1000c

102 J. Bartasevicius

Page 103: bartasevicius.com.brbartasevicius.com.br/sites/default/files/2018-12/... · Sobre este trabalho. E permitido: • Compartilhar - copiar e redistribuir o material em qualquer meio

CAPITULO 2. SOLUCOES

Evoluindo a equacao dada no problema:

x2 − (f(x))2 = 9 (x+ f(x)) =⇒

(x− f(x)) (x+ f(x)) = 9 (x+ f(x)) =⇒x− f(x) = 9 (A)

Substituindo x e f(x) em (A):

u+ 10d+ 100c+ 1000m− (d+ 10u+ 100m+ 1000c) = 9 =⇒

−9u+ 9d− 900c+ 900m = 9 =⇒d− u+ 100(m− c) = 1 (B)

O valor mınimo de 100(m − c) e 0. Mas m e c nao podem valer 0simultaneamente, caso contrario o numero teria 2 algarismos ou menos.Devido ao valor maximo de d− u ser 9, o resultado de m− c deve ser 0.Se m− c 6= 0 nao ha valor de d− u que faca (B) ser 1.Portanto, o menor valor de m nestas condicoes e m = 1. Logo c = 1.Neste caso, d−u = 1, cujo menor valor de d e d = 1 e, consequentemente,u = 0.Finalmente, o menor numero maior que 99 que satisfaz a equacao doproblema e 1110.

Solucao 55a) Dividindo o problema em partes:a.1) Todos os coeficientes sao 0 ou 1. Assim, assumindo valores positivospara x, o polinomio P (x) sera maior do que zero.

Conclusao 1: Nao ha raızes positivas para P (x).

a.2) Se a0 = 0, entao e possıvel fatorar com x em evidencia. Sendo xum fator de P (x), entao x = 0 e raiz.

Conclusao 2: 0 e uma possıvel raiz de P (x), se a0 = 0.

a.3) Assumindo a0 = 1 e todos os demais coeficientes 0, P (x) = a2015+1.Neste caso, −1 e raiz do polinomio.Adicionalmente, a soma de termos de coeficiente 1 com expoente parcom outro de expoente ımpar, quando x = −1, da o valor zero. Istoimplica mostrar que o polinomio tera a raiz −1 sempre que houver umnumero exato de pares de termos cujos expoentes sejam par e ımpar.Demonstracao: Seja i, j ∈ N, com i > j:

x2i + x2j+1 = 0 =⇒

J. Bartasevicius 103

Page 104: bartasevicius.com.brbartasevicius.com.br/sites/default/files/2018-12/... · Sobre este trabalho. E permitido: • Compartilhar - copiar e redistribuir o material em qualquer meio

CAPITULO 2. SOLUCOES

x2j+1(x2i−2j−1 + 1

)= 0 =⇒

x2i−2j−1 + 1 = 0 =⇒

x2(i−j)

x+ 1 = 0 =⇒

(x2)i−jx

+ 1 = 0 =⇒ x = −1

Conclusao 3: −1 e uma possıvel raiz possıvel de P (x), se houver paresde termos de coeficientes 1 e expoentes pares e ımpares.

a.4) As demais raızes inteiras sao para x 6 − 2.Supor que o unico termo de expoente ımpar seja x2015 e todos os de-mais termos de expoente par de P (x) estejam presentes. Desta forma,havera apenas um termo produzindo valor negativo e todos os demaisproduzindo valor positivo. Esta condicao faz com que o valor negativoseja o menor possıvel, enquanto o valor positivo seja o maior possıvel.Quando x = −2, apenas o termo x2015 produzira valor negativo.Todos os demais termos de expoente par produzirao valor positivo.Resta saber se a soma de todos os termos de expoente par podem seigualar ou serem maiores que o modulo de x2015. Se forem iguais oumaiores, entao havera valores de x 6 − 2 que tambem serao possıveisraızes.Portanto, se a inequacao∣∣∣x2015

∣∣∣ 6 x2014 + x2012 + . . . + x2 + 1 (A)

for verdadeira, entao existe raiz menor ou igual a −2. Caso contrario,nao existem outras raızes, pois o modulo do termo negativo sera sempremenor que a soma dos termos positivos.

x2014 + x2012 + . . . + x2 + 1 e uma progressao geometrica de razao1

x2:

x2014 + x2012 + . . . + x2 + 1 =x2016 − 1

x2 − 1

Voltando a inequacao (A):∣∣∣x2015∣∣∣ 6 x2016 − 1

x2 − 1

Sendo x < 0,∣∣x2015

∣∣ = −x2015:

−x2015 − x2016 − 1

x2 − 16 0 =⇒

104 J. Bartasevicius

Page 105: bartasevicius.com.brbartasevicius.com.br/sites/default/files/2018-12/... · Sobre este trabalho. E permitido: • Compartilhar - copiar e redistribuir o material em qualquer meio

CAPITULO 2. SOLUCOES

−x2017 − x2016 + x2015 + 1

x2 − 16 0

Para x 6 − 2, o denominador (x2 − 1) e sempre positivo.Portanto, se existir x 6 − 2 tal que −x2017 − x2016 + x2015 + 1 6 0,existirao mais solucoes.Analisando:

−x2017 − x2016 + x2015 + 1 6 0 =⇒

−x2015(x2 + x− 1

)+ 1 6 0 =⇒

• −x2015 > 0, para x 6 − 2.

• x2 + x− 1 possui raızes−1±

√5

2, portanto x2 + x− 1 > 0 para

x 6 − 2.

• Sendo−x2015 e x2+x−1 ambos positivos, logo−x2015(x2 + x− 1

)+

1 > 0, contrariando a hipotese. Portanto nao ha mais solucoes.

Conclusao 4: Nao existem solucoes para x 6 − 2.

∴, as possıveis raızes sao somente 0 e −1.

b) Para que haja duas raızes inteiras distintas, 0 e −1 sao obriga-toriamente estas duas raızes.De acordo com a Conclusao 2, para existir a raiz 0, a0 = 0.E, de acordo com a Conclusao 3, e necessario existir pares de termoscom expoentes pares e ımpares para que −1 possa ser raiz.Sabendo que os termos vao de a1 a a2014, devem existir:

• n+ 1 coeficientes de termos com expoente par; e

• n coeficientes de termos com expoente ımpar.

Alem de x2015 e 0x0, existem 1007 termos de expoente ımpar e 1007termos de expoente par:

• Somente um termo de expoente par (exceto x0) de coeficiente 1 enenhum termo de expoente ımpar:Sao:

C1007,1︸ ︷︷ ︸possibilidades de um

coeficiente 1 em expoente par

× C1007,0︸ ︷︷ ︸possibilidades de nenhum

coeficiente 1 em expoente ımpar

J. Bartasevicius 105

Page 106: bartasevicius.com.brbartasevicius.com.br/sites/default/files/2018-12/... · Sobre este trabalho. E permitido: • Compartilhar - copiar e redistribuir o material em qualquer meio

CAPITULO 2. SOLUCOES

• Dois termos de expoente par de coeficiente 1 e um termo de expo-ente ımpar (exceto x2015):

C1007,2︸ ︷︷ ︸possibilidades de dois

coeficientes 1 em expoente par

× C1007,1︸ ︷︷ ︸possibilidades de um

coeficiente 1 em expoente ımpar

...

• Todos os termos de expoente par de coeficiente 1 (exceto a0) etodos, menos um, termo de expoente ımpar (exceto x2015):

C1007,1007︸ ︷︷ ︸possibilidades de todos

os coeficientes 1 em expoente par

× C1007,1006︸ ︷︷ ︸possibilidades de 1006

coeficientes 1 em expoente ımpar

Somando o numero de possibilidades dos polinomios tem-se:

C1007,1 × C1007,0 + C1007,2 × C1007,1 + . . . + C1007,1007 × C1007,1006

∴1006∑n=0

C1007,n × C1007,n+1

Solucao 56 ∣∣∣ln(sin2 x)∣∣∣ = ln

(sin2 x

)• ln

(sin2 x

)> 0 se sin2 x > 1 (impossıvel!)

• ln(sin2 x

)6 0 se sin2 x 6 1

∴ − ln(

sin2 x)

= ln(

sin2 x)

=⇒

ln(

sin2 x)

= 0 =⇒

sin2 x = 1 =⇒ sinx = ±1

Finalmente, x = π2 ou x = −π2 =⇒ x = π

2 + kπ, k ∈ Z.Alternativa a).

106 J. Bartasevicius

Page 107: bartasevicius.com.brbartasevicius.com.br/sites/default/files/2018-12/... · Sobre este trabalho. E permitido: • Compartilhar - copiar e redistribuir o material em qualquer meio

CAPITULO 2. SOLUCOES

Solucao 57Fazendo 3x = ∆, tem-se:

12∆3 − 19∆2 + 8∆− 1 = 0 (A)

Como 3x− 1 e fator de 12x3 − 19x2 + 8x− 1, entao 3∆− 1 e fator de(A): (

12∆3 − 19∆2 + 8∆− 1)÷ (3∆− 1) = 4∆2 − 5∆ + 1

Fatorando (A): (4∆2 − 5∆ + 1

)(3∆− 1) = 0

De onde se chega as raızes: 14 , 1 e 1

3 .Substituindo ∆:

3x =1

4=⇒ x = log3

1

4

3x = 1 =⇒ x = 0

3x =1

3=⇒ x = −1

∴ log31

4+ 0− 1 = log3

(1

4× 1

3

)= − log3 12

Alternativa a).

Solucao 58Os tres termos sao: x

e−2a , x e xe−2a.x

e−2a + x+ xe−2a = 7

xe−2a − xe−2a = 3

=⇒

xe4a + xe2a + x = 7e2a

x− xe4a = 3e2a

Fazendo e2a = y:xy2 + xy + x = 7y =⇒ x = 7y

y2+y+1

x− xy2 = 3y

Aplicando x na segunda equacao do sistema:

7y

y2 + y + 1− 7y

y2 + y + 1y2 = 3y =⇒

J. Bartasevicius 107

Page 108: bartasevicius.com.brbartasevicius.com.br/sites/default/files/2018-12/... · Sobre este trabalho. E permitido: • Compartilhar - copiar e redistribuir o material em qualquer meio

CAPITULO 2. SOLUCOES

7y − 7y3 − 3y3 − 3y2 − 3y

y2 + y + 1= 0 =⇒

y(−10y2 − 3y + 4

)= 0 =⇒

y = 0

y = −4

5

y =1

2

Como y = e2a, descartam-se as solucoes y = 0 e y = −45 .

e2a =1

2=⇒ a =

ln1

22

=− ln 2

2= − ln

√2

Alternativa d).

Solucao 59Ao calcular f (g(x)), primeiramente deve-se verificar as condicoes quefazem |g(x)| < 1 e |g(x)| > 1.Comecando por |g(x)| < 1:

|g(x)| < 1 =⇒ −1 < g(x) < 1

−1 <2x− 3

x− 1< 1 =⇒

2x− 3

x− 1> −1 ∧ 2x− 3

x− 1< 1

• 2x−3x−1 > −1

2x− 3 + (x− 1)

x− 1> 0 =⇒ 3x− 4

x− 1> 0 (A)

• 2x−3x−1 < 1

2x− 3− (x− 1)

x− 1< 0 =⇒ x− 2

x− 1< 0 (B)

108 J. Bartasevicius

Page 109: bartasevicius.com.brbartasevicius.com.br/sites/default/files/2018-12/... · Sobre este trabalho. E permitido: • Compartilhar - copiar e redistribuir o material em qualquer meio

CAPITULO 2. SOLUCOES

Solucao de (A), aplicando a regra de sinais:

− − +

− + +

+ − +

3x− 4

x− 1

3x−4x−1

1 43

Solucao de (B), aplicando a regra de sinais:

− − +

− + +

+ − +

x− 2

x− 1

x−2x−1

1 2

∴ a solucao de |g(x)| < 1 e (A) ∩ (B):

(A)

(B)

(A)∩(B)

1 43

2

∴ f(g(x)) = 1, se 43 < x < 2.

Alternativa c).

Solucao 60

x2 + y2 − 6√

2y + 18 = 18 =⇒ x2 +(y − 3

√2)2

=(

3√

2)2

Desenhando os graficos das equacoes da reta e da circunferencia:

J. Bartasevicius 109

Page 110: bartasevicius.com.brbartasevicius.com.br/sites/default/files/2018-12/... · Sobre este trabalho. E permitido: • Compartilhar - copiar e redistribuir o material em qualquer meio

CAPITULO 2. SOLUCOES

3√

2C

A

B

x

y

Do grafico, e necessario determinar a coordenada do ponto B paracalcular a distancia AB.De y =

√2x, tem-se x = y√

2e aplicando na equacao do cırculo:

y2

2+(y − 3

√2)2

=(

3√

2)2

=⇒ 3y2

2− 6√

2y = 0 =⇒

=⇒

y = 0 =⇒ x = 0 (ponto A)

y = 4√

2 =⇒ x = 4 (ponto B)AC = 3

√2

BC = 3√

2

AB =

√42 +

(4√

2)2

= 4√

3

=⇒ perımetro = 6√

2 + 4√

3

Alternativa e).

Solucao 61

y =ax − a−x

2=⇒ 2y = ax − 1

ax=⇒

a2x − 2yax − 1 = 0

Fazendo ax = Z:Z2 − 2yZ − 1 = 0 =⇒

Z =2y ±

√4y2 + 4

2= y ±

√y2 + 1

110 J. Bartasevicius

Page 111: bartasevicius.com.brbartasevicius.com.br/sites/default/files/2018-12/... · Sobre este trabalho. E permitido: • Compartilhar - copiar e redistribuir o material em qualquer meio

CAPITULO 2. SOLUCOES

a > 1 =⇒ ax > 0.√y2 + 1 > y, portanto a unica solucao para Z e

Z = y +√y2 + 1.

ax = y +

√y2 + 1 =⇒

x = loga

(y +

√y2 + 1

)Alternativa c).

Solucao 62Analisando as afirmacoes:

I. A afirmacao e VERDADEIRA.Calculo das raızes:

3x4−3−10x3+10x = 0 =⇒ 3(x4 − 1

)−10x

(x2 − 1

)= 0 =⇒

=⇒ 3(x2 − 1

)(x2 + 1

)− 10x

(x2 − 1

)= 0 =⇒

=⇒(x2 − 1

)(3x2 − 10x+ 3

)= 0(

x2 − 1)

= 0 =⇒ x = ±1

(3x2 − 10x+ 3

)= 0 =⇒

x = 3 ou

x =1

3

A equacao de 4° grau produz 4 raızes, e neste caso, todas as raızes

sao reais:

−1,

1

3, 1, 3

.

Portanto a afirmacao e verdadeira.

II. A afirmacao e FALSA.Equacao recıproca e aquela que cujas raızes possuem o seu inverso.

Por exemplo, em 3x2 − 10x+ 3 = 0, cujas raızes sao 3 e1

3. Para

isto e necessario um numero para de raızes, exceto quando umadas raızes for 1 ou −1, caso em que seu inverso e ele proprio.

J. Bartasevicius 111

Page 112: bartasevicius.com.brbartasevicius.com.br/sites/default/files/2018-12/... · Sobre este trabalho. E permitido: • Compartilhar - copiar e redistribuir o material em qualquer meio

CAPITULO 2. SOLUCOES

III. A afirmacao e VERDADEIRA.Calculo das raızes em x3 + 4x2 − 4x− 16 = 0:

x3 − 4x+ 4x2 − 16 = 0 =⇒ x(x2 − 4

)+ 4

(x2 − 4

)= 0 =⇒

=⇒ (x+ 4)(x2 − 4

)= 0 =⇒

x = ±2

x = −4

Calculo das raızes em x3 + 2x2 − x− 2 = 0:

x3 − x+ 2x2 − 2 = 0 =⇒ x(x2 − 1

)+ 2

(x2 − 1

)= 0 =⇒

=⇒ (x+ 2)(x2 − 1

)= 0

x = ±1

x = −2

Portanto as raızes de uma equacao sao exatamente o dobro daoutra.

Alternativa b).

Solucao 63Os 18 professores sao distribuıdos da seguinte maneira:

• 7 Matematica (M)

• 3 Fısica (F )

• 4 Quımica (Q)

• 4 Outras disciplinas (O)

A comissao composta de 12 professores precisa conter:

• 5 Matematica (M)

• 2, 3 Fısica (F )

• 0,1, 2 Quımica (Q)

• 0, 1, 2, 3, 4 Outras disciplinas (O)

O numero maximo de professores de Outras disciplinas e 4, portanto asoma de F e Q nao pode ser menor que 12− 5− 4 = 3, caso contrarionao havera professores suficientes de Outras disciplinas para completaro quadro de 12 professores. Entao, so nao podera haver a combinacaode 2 professores de Fısica com 0 (zero) de QuımicaAssim, havera 5 combinacoes entre as quantidades de professores deFısica, Quımica e Outras disciplinas:

112 J. Bartasevicius

Page 113: bartasevicius.com.brbartasevicius.com.br/sites/default/files/2018-12/... · Sobre este trabalho. E permitido: • Compartilhar - copiar e redistribuir o material em qualquer meio

CAPITULO 2. SOLUCOES

Disciplinas A B C D EM 5 5 5 5 5F 2 2 3 3 3Q 1 2 0 1 2O 4 3 4 3 2

Calculo do numero de possibilidades de comissoes diferentes:

• Possibilidade A:

C7,5 × C3,2 × C4,1 × C4,4 =7!

5!2!× 3!

2!1!× 4!

1!3!× 4!

4!0!=

= 21× 3× 4× 1 = 252

• Possibilidade B:

C7,5 × C3,2 × C4,2 × C4,3 =7!

5!2!× 3!

2!1!× 4!

2!2!× 4!

3!1!=

= 21× 3× 6× 4 = 1512

• Possibilidade C:

C7,5 × C3,3 × C4,0 × C4,4 =7!

5!2!× 3!

3!0!× 4!

0!4!× 4!

4!0!=

= 21× 1× 1× 1 = 21

• Possibilidade D:

C7,5 × C3,3 × C4,1 × C4,3 =7!

5!2!× 3!

3!0!× 4!

1!3!× 4!

3!1!=

= 21× 1× 4× 4 = 336

• Possibilidade E:

C7,5 × C3,3 × C4,2 × C4,2 =7!

5!2!× 3!

3!0!× 4!

2!2!× 4!

2!2!=

= 21× 1× 6× 6 = 756

∴ Solucao = 252 + 1512 + 21 + 336 + 756 = 2877Alternativa d).

J. Bartasevicius 113

Page 114: bartasevicius.com.brbartasevicius.com.br/sites/default/files/2018-12/... · Sobre este trabalho. E permitido: • Compartilhar - copiar e redistribuir o material em qualquer meio

CAPITULO 2. SOLUCOES

Solucao 64

A = Cn,030 + Cn,131 + Cn,232 + . . .+ Cn,n3n = (3 + 1)n

B = Cn−1,0110 + Cn−1,1111 + Cn−1,2112 + . . .+ Cn−1,n−111n−1 =

= (11 + 1)n−1

lnB − lnA = ln6561

4=⇒ ln

B

A= ln

38

22=⇒

(11 + 1)n−1

(3 + 1)n=

38

22=⇒

(12)n−1

(4)n−1.1

4=

38

22=⇒

(12

4

)n−1

= 38 =⇒ 3n−1 = 38 =⇒ n− 1 = 8

∴ n = 9

Alternativa e).

Solucao 65

a1an = 243 =⇒ a21qn−1 = 35 (A)

q20 = a1a1qa1q2 . . . a1q

n−1 = an1q1+2+...+n−1 = an1q

(n−1)n2 (B)

q6 = a1qn−1 (C)

Modificando (B):

q20 =

(a1q

(n−1)2

)n=

(a1q

(n−1)

q(n−1)

2

)n(D)

Substituindo (C) em (D):

q20 =

(q6

q(n−1)

2

)n=

(q6− (n−1)

2

)n=⇒

114 J. Bartasevicius

Page 115: bartasevicius.com.brbartasevicius.com.br/sites/default/files/2018-12/... · Sobre este trabalho. E permitido: • Compartilhar - copiar e redistribuir o material em qualquer meio

CAPITULO 2. SOLUCOES

q20 = qn(13−n2 ) =⇒ 20 =

13n− n2

2

=⇒ n2 − 13n− 40 = 0 =⇒

n = 8 ou

n = 5

Verificando as hipoteses de n:

• n = 8:De (B) tem-se:

q20 = a81q

28 =⇒ a1 = q−1 (E)

Aplicando (E) em (A):

a21qn−1 = 35 =⇒ q−2q7 = 35

∴ q = 3

• n = 5:De (B) tem-se:

q20 = a51q

10 =⇒ a1 = q2 (F)

Aplicando (F) em (A):

a21qn−1 = 35 =⇒ q4q4 = 35 =⇒ q8 = 35

Mas q ∈ N, ∴ n = 5 nao e solucao.

Substituindo (C) em (A):

a1(a1qn−1) = 35 =⇒ a1q

6 = 35 (G)

Para q = 3, a1 = 13 .

Por fim, solicita-se calcular 3−1 + 30 + 31 + . . .+ 36:

3−1 + 30 + 31 + . . .+ 36 = S =⇒ (×3)

30 + 31 + 32 + . . .+ 37 = 3S =⇒

S − 3−1 + 37 = 3S =⇒

S =38 − 1

6

Alternativa c).

J. Bartasevicius 115

Page 116: bartasevicius.com.brbartasevicius.com.br/sites/default/files/2018-12/... · Sobre este trabalho. E permitido: • Compartilhar - copiar e redistribuir o material em qualquer meio

CAPITULO 2. SOLUCOES

Solucao 662° termo=C6

5x5y1 = 6x5y1.

O termo de maior coeficiente implica ser o termo do meio: C63x

3y3 =

20x3y3.Para obter a soma de todos os coeficientes, deve-se fazer os valores dexiy6−i serem iguais a 1, para i ∈ N, 1 6 i 6 6. Assim, a unica maneirae adotar x = y = 1. Portanto a soma dos coeficientes e 26.

Substituindo x = (2)z+1 e y =(

14

)z−12:

6(

(2)z+1)5((

1

4

)z−12

)1

+1

10.20(

(2)z+1)3((

1

4

)z−12

)3

= 8.26

6(2)5z+5(2)−2z+1 + 2.(2)3z+3(2)−6z+3 = 29

3(2)3z+7 + (2)−3z+7 = 29 (÷27)

3(2)3z + (2)−3z = 22

Fazendo 23z = ∆:

3∆ +1

∆= 22

3∆2 − 4∆ + 1 = 0 =⇒

∆ = 1 =⇒ z = 0 ou

∆ = 13 =⇒ z =

− log2 33

Alternativa c).

Solucao 67Sendo ϕ o argumento de 2 + 2

√3, e z o seu modulo, tem-se que:(

2 + 2√

3)10

= z10 10ϕ

Portanto, a = z10:

a =

(√22 +

(2√

3)2)10

116 J. Bartasevicius

Page 117: bartasevicius.com.brbartasevicius.com.br/sites/default/files/2018-12/... · Sobre este trabalho. E permitido: • Compartilhar - copiar e redistribuir o material em qualquer meio

CAPITULO 2. SOLUCOES

∴ a = 410.Assim,

(4.410

)x= 410

411x = 410 =⇒ x =10

11

Alternativa a).

Solucao 68Calculando o determinante com os coeficientes de x, y e z:∣∣∣∣∣∣

3 −2 11 1 −12 1 −2

∣∣∣∣∣∣ = −4

Portanto, o sistema e possıvel e determinado.Calculo de x, y e z. Multiplicando a segunda equacao por −2 e somandocom a terceira, tem-se:

−2x− 2y + 2z = 0

2x+ y − 2z = −1=⇒ −y = −1 =⇒ y = 1

Somando a primeira com a segunda equacao:3x− 2y + z = 7

x+ y − z = 0=⇒ 4x− y = 7 =⇒ 4x = 8 =⇒ x = 2

Com x = 2 e y = 1 obtem-se z da segunda equacao:

x+ y − z = 0 =⇒ z = 3

∴ xyz = 6.Alternativa c).

Solucao 69B e a populacao de Votuporanga. B

65 presenciaram o acidente, em t = 0horas:

B

65=

B

1 + C=⇒ C = 64

J. Bartasevicius 117

Page 118: bartasevicius.com.brbartasevicius.com.br/sites/default/files/2018-12/... · Sobre este trabalho. E permitido: • Compartilhar - copiar e redistribuir o material em qualquer meio

CAPITULO 2. SOLUCOES

Quando t = 3 horas, B9 sabem do acidente:

B

9=

B

1 + 64e−k3=⇒ 9 = 1 + 64e−3k =⇒

e−3k = 2−3 =⇒ e−k = 2−1

Portanto, o valor de t quando o resultado da formula for B5 sera:

B

5=

B

1 + 64(2−1

)t =⇒

5 = 1 + 64.2−t =⇒ 2−t =1

16∴ t = 4 horas.Alternativa a).

Solucao 70As retas (r1) e (r2) sao retas perpendiculares, que podem ser verificadospela multiplicacao dos coeficientes de x, cujo resultado e −1.

Portanto, a circunferencia que tangencia ambas as retas, passando peloponto P : (0, 0) obrigatoriamente tangencia a reta (r1) neste ponto, poisP e um ponto da reta.

Entao, deve-se definir o centro da circunferencia pelo encontro das retasx+ y = 0 (perpendicular a reta (r1) em P ) e da reta perpendicular a(r2) em x = 2, conforme figura abaixo:

y

x

C

2

P

(r1)(r2)

x+ y = 0 x− y − 2 = 0

118 J. Bartasevicius

Page 119: bartasevicius.com.brbartasevicius.com.br/sites/default/files/2018-12/... · Sobre este trabalho. E permitido: • Compartilhar - copiar e redistribuir o material em qualquer meio

CAPITULO 2. SOLUCOES

Assim, o centro C da circunferencia e o encontro das retas x+ y = 0 ex− y − 2 = 0, definindo no ponto (1,−1).A distancia do ponto C ao ponto P e

√2.

∴ a equacao da circunferencia e (x− 1)2 + (y + 1)2 =(√

2)2

.Alternativa b).

Solucao 71

x3 + 1 = (x+ 1)(x2 − x+ 1)

Aplicando o mmc nos denominadores:

x3 + 4

x3 + 1=x3 + 1

x3 + 1+a(x2 − x+ 1)

x3 + 1+

(bx+ c)(x+ 1)

x3 + 1=⇒

x3 + 4 = x3 + 1 + ax2 − ax+ a+ bx2 + bx+ cx+ c =⇒

0 = (a+ b)x2 + (−a+ b+ c)x+ (a+ c− 3) =⇒

a+ b = 0

−a+ b+ c = 0

a+ c− 3 = 0

∴ a = 1, b = −1 e c = 2 =⇒ a+ b+ c = 2Alternativa d).

Solucao 72Para o fator a16m2, e necessario que a2 seja elevado a oitava potencia em a segunda potencia. Portanto o fator de a16m2 e:

C108

(3a2

2

)8(2m

3

)2

=9.5.36

26a16m2 (A)

Para o fator a14m3, e necessario que a2 seja elevado a 7a potencia e ma 3a potencia. Portanto o fator de a14m3 e:

C107

(3a2

2

)7(2m

3

)3

=8.3.5.34

24a14m3 (B)

Calculando a razao entre(A) e (B):

9.5.36

26a16m2

8.3.5.34

24a14m3

=9

16=⇒

J. Bartasevicius 119

Page 120: bartasevicius.com.brbartasevicius.com.br/sites/default/files/2018-12/... · Sobre este trabalho. E permitido: • Compartilhar - copiar e redistribuir o material em qualquer meio

CAPITULO 2. SOLUCOES

3a2

2m= 1 =⇒ 3a2 = 2m

Fazendo(

3a2

2 + 2m3

)10=(m2 + 4

)5e substituindo 3a2 por 2m:(

2m

2+

2m

3

)2

= m2 + 4 =⇒

25

9m2 = m2 + 4

Pelo enunciado, a e m sao positivos. Entao m = 32 e a = 1.

∴ a+m = 52 .

Alternativa c).

Solucao 73

a1a2a3 . . . an = 225 =⇒ an1 .q1+2+3+...+(n−1) = 225

an1 .qn(n−1)

2 = 225 ou

(a1.q

(n−1)2

)n= 225 (A)

n e ımpar =⇒ n = 2x + 1, x ∈ N. Portanto, o elemento do meio eax+1 = a1.q

x.

a1.qx = 25 (B)

Fazendo x = n−12 e substituindo (B) em (A) tem-se:(

25)n

= 225 =⇒ 5n = 25 ∴ n = 5

A soma dos 4 primeiros termos e 2(1 + q)(1 + q2):

a1 + a1.q + a1q2 + a1.q

3 = 2(q3 + q2 + q + 1) =⇒

a1

(1 + q + q2 + q3

)= 2(q3 + q2 + q + 1) =⇒ a1 = 2

Com n = 5 e a1 = 2, de (B) tem-se:

2.q2 = 25 =⇒ q = 4

∴ a1 + q + n = 2 + 4 + 5 = 11.Alternativa e).

120 J. Bartasevicius

Page 121: bartasevicius.com.brbartasevicius.com.br/sites/default/files/2018-12/... · Sobre este trabalho. E permitido: • Compartilhar - copiar e redistribuir o material em qualquer meio

CAPITULO 2. SOLUCOES

Solucao 74

PG de razao a =⇒

b = a2

c = a3

d = a4

e = a5

a+a2 +a3 +a4 +a5 = 13a+12 =⇒ a2 +a3 +a4 +a5 = 12a+12 =⇒

a2(

1 + a+ a2 + a3)

= 12(a+ 1) =⇒

a2(

1 + a+ a2 (1 + a))

= 12(a+ 1) =⇒

a2 (1 + a)(

1 + a2)

= 12(a+ 1) =⇒ a2(

1 + a2)

= 12 =⇒

a4 + a2 − 12 = 0 =⇒

a2 = 3 =⇒ a = ±

√3 ou

a2 = −4(a /∈ R)

a e base do logaritmo, consequentemente a > 0. ∴ a =√

3.Calculando a soma logarıtmica, convertendo todos os termos para abase a:

1

loga x+

1

loga2 x+

1

loga3 x+

1

loga4 x+

1

loga5 x=

5

2=⇒

loga a

loga x+

2 loga a

loga x+

3 loga a

loga x+

4 loga a

loga x+

5 loga a

loga x=

5

2=⇒

15

loga x=

5

2=⇒ 6 = log√3 x =⇒ x =

(√3)6

∴ x = 33.Alternativa a).

Solucao 75

xy =1

y2=⇒ x = y

− 2y (A)

log x+ log y = log1√x

=⇒ log xy = log1√x

=⇒ xy =1√x

(B)

J. Bartasevicius 121

Page 122: bartasevicius.com.brbartasevicius.com.br/sites/default/files/2018-12/... · Sobre este trabalho. E permitido: • Compartilhar - copiar e redistribuir o material em qualquer meio

CAPITULO 2. SOLUCOES

Substituindo (A) em (B):

y− 2

y y =1√y− 2

y

=⇒ y− 2

y+1− 1y = 1

Sendo x, y 6= 1, entao o expoente de y deve ser 0 para o resultado ser 1:

−2

y+ 1− 1

y= 0 =⇒ y = 3

Substituindo y = 3 em (A): x = 3−23 = 3

√19 .

Alternativa b).

Solucao 76Se 4 + i

√2 e√

5 sao raızes de um polinomio que possui todos oscoeficientes inteiros, entao as outras duas raızes sao, respectivamente,seu conjugado e seu valor negativo, ou seja 4− i

√2 e −

√5.

Calculando o valor da 5a raiz:

−540

2=(

4 + i√

2)(

4− i√

2)(√

5)(−√

5).r5

270 = (16 + 2) (5) .r5 =⇒ r5 = 3

∴ a soma dos quadrados das raızes reais e:(√5)2

+(−√

5)2

+ 32 = 19

Alternativa b).

Solucao 77Fazendo z3 = y:

z1y2 + z2y − 8 = 0 (A)

Se uma das raızes e z = 1, entao y = 1 e raiz da equacao acima:

z1 (y − 1) (y − y2) = 0 =⇒

z1y2 − yz1 (1 + y2) + z1y2 = 0 (B)

122 J. Bartasevicius

Page 123: bartasevicius.com.brbartasevicius.com.br/sites/default/files/2018-12/... · Sobre este trabalho. E permitido: • Compartilhar - copiar e redistribuir o material em qualquer meio

CAPITULO 2. SOLUCOES

Comparando (A) com (B) tem-se as relacoes:z1y2 = −8

−z1 (1 + y2) = z2 =⇒ −z1 − z1y2 = z2 =⇒ −z1 + 8 = z2

De z1 + z2 = 8, onde z1 e z2 possuem mesmo modulo. Portanto, a somade seus valores so pode resultar num numero real se forem conjugados.

z1 = a+ ib

z2 = a− iba2 + b2 = 4

=⇒ z1 + z2 = 2a = 8 =⇒ a = 4

Entao, b = 0. Assim, a equacao original e:

4z6 + 4z3 − 8 = 0 =⇒ z6 + z3 − 2 = 0 =⇒

z3 = 1 ou

z3 = −2

Portanto, as raızes reais sao 1 e 3√−2, cuja soma e 1− 21/3.

Alternativa c).

Solucao 78

P (z) = z5(z+2)+6z3(z+2)+8z(z+2) =(z5 + 6z3 + 8z

)(z+2) =⇒

P (z) = z(z + 2)(z4 + 6z2 + 8

)=⇒

Alem de z = 0 e z = −2, tambem sao raızes de P (z) as raızes daequacao bi-quadrada z4 + 6z2 + 8:

z2 = −2 =⇒ z = ±i√

2

z2 = −4 =⇒ z = ±2i

∴ apenas duas raızes sao reais e distintas.Alternativa b).

J. Bartasevicius 123

Page 124: bartasevicius.com.brbartasevicius.com.br/sites/default/files/2018-12/... · Sobre este trabalho. E permitido: • Compartilhar - copiar e redistribuir o material em qualquer meio

CAPITULO 2. SOLUCOES

Solucao 79

1 + i =√

2 π4

√2

1 + i=

√2 0√2 π

4

= 1 −π4

(1 −π4 )93 = 1 −93π4

1 −93π4 = 1 −11.2π − 5π

4 = 1 −5π4

( √2

1 + i

)93

= 1 −5π4 = −

√2

2+ i

√2

2

Alternativa a).

Solucao 80a1, a2, a3, a4 em PG: a1, a1q, a1q

2, a1q3.

Aplicando no sistema de equacoes:a1x+ a1q

2y = 1

a1a1qx+ a1a1q3y = a1q(÷a1q) =⇒ a1x+ a1q

2y = 1

Apos a simplificacao, a segunda equacao e igual a primeira. Portanto,o sistema e possıvel e indeterminado.Alternativa c).

Solucao 81x, y, z em PA: y − r, y, y + r.

3ay−r + 2ay + ay+r = 0(÷ay)

3

ar+ 2 + ar = 0

124 J. Bartasevicius

Page 125: bartasevicius.com.brbartasevicius.com.br/sites/default/files/2018-12/... · Sobre este trabalho. E permitido: • Compartilhar - copiar e redistribuir o material em qualquer meio

CAPITULO 2. SOLUCOES

Fazendo ar = ε tem-se:

3 + 2ε+ ε2 = 0 =⇒

ε = 3 ou

ε = −1(impossıvel, pois a > 0)

Entao, ar = 3. ∴ r = loga 3.Alternativa e).

Solucao 82

z = x+ iy =⇒ z3 = x3 − 3xy2︸ ︷︷ ︸1

+i(3x2y − y3︸ ︷︷ ︸1

)

∴ z3 = 1 + i =√

2 π4

z =3√√

2 π4 =

6√

2 π12

Finalmente, z3 = 1 + i e | z |= 6√

2.Alternativa b).

Solucao 83Alterando todos os logaritmos para a base y:

2 logy 7 =logy 7

2 logy y+

logy 7

logy 2 + logy y=⇒

2 logy 7 =logy 7

2+

logy 7

logy 2 + 1=⇒ (÷ logy 7)

2 =1

2+

1

logy 2 + 1=⇒

4(logy 2 + 1

)=(logy 2 + 1

)+ 2 =⇒ logy 2 = −1

3

∴ y−13 = 2 =⇒ y =

1

8

Alternativa d).

J. Bartasevicius 125

Page 126: bartasevicius.com.brbartasevicius.com.br/sites/default/files/2018-12/... · Sobre este trabalho. E permitido: • Compartilhar - copiar e redistribuir o material em qualquer meio

CAPITULO 2. SOLUCOES

Solucao 84O calculo torna-se mais facil ao identificar o numero de arranjos com ascinco vogais juntas e subtrair do total.A palavra VESTIBULANDO possui 12 letras, todas distintas, com 5vogais e 7 consoantes. Assim, o numero total de arranjos e 12!Para se manter as cinco vogais juntas, elas podem ocupar as posicoesde 1 a 8:A︸︷︷︸1

E︸︷︷︸2

I︸︷︷︸3

O︸︷︷︸4

U︸︷︷︸5︸︷︷︸

6︸︷︷︸

7

A︸︷︷︸8

E︸︷︷︸9

I︸︷︷︸10

O︸︷︷︸11

U︸︷︷︸12

As vogais podem se arranjar em 5! possibilidades.E para cada arranjo de vogais, existem 7! arranjos de consoantes.Assim, o numero de arranjos com todas as vogais juntas e: 8× 5!× 7! =5!× 8!.Portanto, o numero de anagramas que nao possuem as 5 vogais juntase 12!− 8!5!.Alternativa c).

Solucao 85Sendo a > 1, para que a inequacao seja verdadeira, os expoentes tambemdevem obedecer a relacao:

2x(1− x) > x− 1 =⇒ 2x2 − x− 1 < 0

As raızes sao x = 1 e x = −12 .

A parabola 2x2 − x− 1 tem o coeficiente de x2 positivo, entao o valorda funcao e negativo quando os valores de x estiver entre as raızes.Portanto, −1

2 < x < 1.Alternativa c).

Solucao 86

4sin4 x

cos4 x=

1

cos4 x+ 4 =⇒ 4

(sin4 x− cos4 x

)= 1 =⇒

4(

sin2 x− cos2 x)

︸ ︷︷ ︸− cos 2x

(sin2 x+ cos2 x

)︸ ︷︷ ︸

1

= 1 =⇒

cos 2x = −1

4

126 J. Bartasevicius

Page 127: bartasevicius.com.brbartasevicius.com.br/sites/default/files/2018-12/... · Sobre este trabalho. E permitido: • Compartilhar - copiar e redistribuir o material em qualquer meio

CAPITULO 2. SOLUCOES

x ∈[0, π2

[=⇒ 2x ∈ [0, π[. Portanto o valor de sin 2x e positivo:

sin 2x =

√1−

(−1

4

)2

=⇒ sin 2x =

√15

4

Calculo de sin 4x:

sin 4x = 2 sin 2x cos 2x = 2

√15

4

(−1

4

)= −√

15

8

∴ sin 2x+ sin 4x =

√15

4+

(−√

15

8

)=

√15

8.

Alternativa b).

Solucao 87

C : x2 + 2x+ 1 + y2 + 2y + 1 = 1 =⇒ (x+ 1)2 + (y + 1)2 = 12

E : x2 − 4x+ 4 + 4y2 + 8y + 4 = 4 =⇒(x− 2

2

)2

+ (y + 1)2 = 1

Elementos da circunferencia C: centro em (−1,−1) e raio de tamanho1.Elementos da circunferencia E : centro em (2,−1), distancia do semi-eixox = 2 e distancia do semi-eixo y = 1.Esbocando o grafico:

y

−1

−1

2x

O

C E

Portanto, a circunferencia e a elipse sao tangentes exteriormente.Alternativa c).

J. Bartasevicius 127

Page 128: bartasevicius.com.brbartasevicius.com.br/sites/default/files/2018-12/... · Sobre este trabalho. E permitido: • Compartilhar - copiar e redistribuir o material em qualquer meio

CAPITULO 2. SOLUCOES

Solucao 88︸︷︷︸a︸︷︷︸b︸︷︷︸c︸︷︷︸d︸︷︷︸e︸︷︷︸f

Analise das hipoteses:

• Fixando os numeros 3 e 4 nas posicoes “a” e “b”, 1 e 2 podemocupar 3 posicoes: “c” e “e”, “c” e “f”, e “d” e “f”. Neste caso,podendo inverter tanto as posicoes de 3 e 4, quanto 1 e 2, tem-se2× 3× 2 = 12 possibilidades.

• Fixando os numeros 3 e 4 nas posicoes “b” e “c”, 1 e 2 podemocupar 4 posicoes: “a” e “d”, “a” e “e”, “a” e “f”, e “d” e “f”.Neste caso, podendo inverter tanto as posicoes de 3 e 4, quanto 1e 2, tem-se 2× 4× 2 = 16 possibilidades.

• Fixando os numeros 3 e 4 nas posicoes “c” e “d”, 1 e 2 podemocupar 4 posicoes: “a” e “e”, “a” e “f”, “b” e “e”, e “b” e “f’.Neste caso, podendo inverter tanto as posicoes de 3 e 4, quanto 1e 2, tem-se 2× 4× 2 = 16 possibilidades.

• Fixando os numeros 3 e 4 nas posicoes “d” e “e”, por simetria, asituacao se torna analoga a quando se fixa nas posicoes “b” e “c”.Neste caso, existem 16 possibilidades.

• Fixando os numeros 3 e 4 nas posicoes “e” e “f”, por simetria, asituacao se torna analoga a quando se fixa nas posicoes “a” e “b”.Neste caso, existem 12 possibilidades.

Para cada uma das hipoteses listadas acima, sobram os numeros 5 e 6,que tambem podem ter suas posicoes invertidas.Portanto, a quantidade de numeros que podem ser formados de acordocom a formulacao do problema e:

(12 + 16 + 16 + 16 + 12)× 2 = 144

Alternativa a).

Solucao 89

Fazendo 2x2

= y:

y2 − 5y + 4 = 0 =⇒

y = 4 ou

y = 1

128 J. Bartasevicius

Page 129: bartasevicius.com.brbartasevicius.com.br/sites/default/files/2018-12/... · Sobre este trabalho. E permitido: • Compartilhar - copiar e redistribuir o material em qualquer meio

CAPITULO 2. SOLUCOES

2x2

= 22 =⇒ x2 = 2 =⇒ x = ±√

2

e2x

2= 20 =⇒ x2 = 0 =⇒ x = 0

Apenas√

2 e raiz real e positiva.∴ a soma das raızes reais positivas e

√2.

Alternativa c).

Solucao 90Sendo a sequencia uma PA, entao a diferenca entre o 3o e o 2o termosdevem ter o mesmo valor da diferenca entre o 2o e o 1o termos, que e ovalor da razao:

1− 4n− (−5n) = −5n− (2 + 3n) =⇒ 9n = −3

∴ n = −13 .

Alternativa b).

Solucao 91A soma dos coeficientes do desenvolvimento do binomio pode ser calcu-lada fazendo x = y = 1, pois todos os termos xiym−i valerao 1 e seraosomados apenas os coeficientes.

(1 + 1)m = 1024 =⇒ 2m = 210 =⇒ m = 10

O numero de arranjos de m tomados 2 a 2 e:

10!

(10− 2)!=

10!

8!= 90

Alternativa b).

Solucao 92Condicoes a serem atendidadas:

f(x) > 0 (condicao I) e

∃√x2 + (2m+ 1)x+ (m2 + 2) 6= 0,∀x ∈ R (condicao II)

J. Bartasevicius 129

Page 130: bartasevicius.com.brbartasevicius.com.br/sites/default/files/2018-12/... · Sobre este trabalho. E permitido: • Compartilhar - copiar e redistribuir o material em qualquer meio

CAPITULO 2. SOLUCOES

Para satisfazer a condicao II, e necessario que x2+(2m+1)x+(m2+2) >0.Condicao II: como o coeficiente de x2 e positivo, a equacao nao poderater raızes reais para que seu valor seja maior que 0, qualquer que seja ovalor de x.

x2 + (2m+ 1)x+ (m2 + 2) > 0 =⇒ ((2m+ 1))2 − 4(m2 + 2) < 0

4m < 7 =⇒ m <7

4(A)

Condicao I: como a raiz quadrada produz resultado positivo, a condicaoI sera satisfeita se o numerador de f(x) > 0, ou seja, x2 + (2m+ 3)x+(m2 + 3) > 0. Pelo coeficiente de x2 ser maior do que 0, a equacao naopodera ter raızes reais ou apenas raızes duplas para que a condicao sejamantida.

x2 + (2m+ 3)x+ (m2 + 3) > 0 =⇒ (2m+ 3)2 − 4(m2 + 3) 6 0

12m 6 3 =⇒ m 61

4(B)

Fazendo (A) ∩ (B):

condicao II

condicao I

(condicao II) ∩ (condicao I)

74

14

14

∴ m 6 14 .

Alternativa d).

Solucao 93Transformando (1 + cos 2x) + i sin 2x = z em notacao polar.Modulo:

| z |=√

(1 + cos 2x)2 + sin2 2x =√

1 + 2 cos 2x+ cos2 2x+ sin2 2x

| z |=√

2√

1 + cos 2x

Fase:

z∠ = arctansin 2x

1 + cos 2x

130 J. Bartasevicius

Page 131: bartasevicius.com.brbartasevicius.com.br/sites/default/files/2018-12/... · Sobre este trabalho. E permitido: • Compartilhar - copiar e redistribuir o material em qualquer meio

CAPITULO 2. SOLUCOES

Portanto, zk =(√

2√

1 + cos 2x)k

k arctan sin 2x1+cos 2x .

Entao, a parte imaginaria e(√

2√

1 + cos 2x)k

sin k arctan sin 2x1+cos 2x .

Simplificacao de arctan sin 2x1+cos 2x :

arctansin 2x

1 + cos 2x= arctan

2 sinx cosx

1 + cos2 x− sin2 x= arctan

2 sinx cosx

2 cos2 x

= arctansinx

cosx= arctan(tanx)

∴ arctansin 2x

1 + cos 2x= x

Simplificacao de√

2√

1 + cos 2x:

√2√

1 + cos 2x =√

2√

1 + cos2 x− sin2 x =√

2√

2 cos2 x

∴| z |= 2 cos x

Finalmente, zk = 2k cosk x sin kx.Alternativa c).

Solucao 94

5− ||2x− 1| − 6| > 0 =⇒ ||2x− 1| − 6| 6 5

−5 6 |2x− 1| − 6 6 5 =⇒ (+6)

1 6 |2x− 1| 6 11

Separando as inequacoes: |2x− 1| > 1 e

|2x− 1| 6 11

|2x− 1| > 1 =⇒

2x− 1 > 1 =⇒ x > 1 ou

2x− 1 6 −1 =⇒ x 6 0(A)

|2x− 1| 6 11 =⇒ −11 6 2x− 1 6 11 =⇒ −5 6 x 6 6 (B)

J. Bartasevicius 131

Page 132: bartasevicius.com.brbartasevicius.com.br/sites/default/files/2018-12/... · Sobre este trabalho. E permitido: • Compartilhar - copiar e redistribuir o material em qualquer meio

CAPITULO 2. SOLUCOES

Fazendo (A) ∩ (B):

(A)

(B)

(A) ∩ (B)

10

−5 6

10−5 6

∴ S = x ∈ R | −5 6 x 6 0 ∪ 1 6 x 6 6.Alternativa e).

Solucao 95Em f(x), por

√2 > 1, entao o valor mınimo de

(√2)a

implica a sermınimo. Neste caso, o menor valor de 3 sinx− 1 e quando sinx = −1.

Portanto o valor mınimo de f(x) e(√

2)3(−1)−1

= 14 .

Em g(x), por 12 < 1, entao o valor mınimo de

(12

)aimplica a ser

maximo. Neste caso, o maior valor de 3 sin2 x− 1 e quando sinx = ±1.

Portanto o valor mınimo de g(x) e(

12

)3(1)2−1= 1

4 .

Finalmente, fmın(x) + gmın(x) = 14 + 1

4 = 12 .

Alternativa d).

Solucao 96

x3 − px2 + 0x− qm = 0

Da equacao tem-se as seguintes relacoes:abc = qm

ab+ ac+ bc = 0

a+ b+ c = p

Das relacoes acima, a2 + b2 + c2 e:

a2 + b2 + c2 = (a+ b+ c)2 − 2ab− 2ac− 2bc =⇒

a2 + b2 + c2 = (a+ b+ c)2 − 2(ab+ ac+ bc) =⇒a2 + b2 + c2 = p2

132 J. Bartasevicius

Page 133: bartasevicius.com.brbartasevicius.com.br/sites/default/files/2018-12/... · Sobre este trabalho. E permitido: • Compartilhar - copiar e redistribuir o material em qualquer meio

CAPITULO 2. SOLUCOES

Calculando logq

[abc(a2 + b2 + c2

)a+b+c]:

logq

[qm(p2)p]

= logq qm + logq p

2p

∴ m+ 2p logq p

Alternativa b).

Solucao 97Fazendo x

y = z:

(z + 2 +

1

z

)4

=

(z2 + 2z + 1

z

)4

=

((z + 1)2

z

)4

=

(z + 1)8

z4=

8∑i=0

C8,izi

z4

Os termos sao da forma C8,izi−4, onde os exponetes de z variam de −4

a 4.Quando i = 4 o termo sera C8,4z

0.Sendo x, y > 0, entao z > 0. Consequentemente, todos os demais termosda sequencia sao maiores do que zero, implicando a soma das potenciasde z ser maior que C8,4.

∴(xy + 2 + y

x

)8> C8,4.

Solucao 9817640 = 23.32.5.72

Os numeros que sao divisores de 17640 e multiplos de 3 devem possuirpelo menos um fator 3 em sua composicao.Assim, os divisores devem assumir a forma 2a.3b.5c.7d, onde os valoresa, b, c e d podem ser:

• a: 0, 1, 2 ou 3;

• b: 1 ou 2 (um fator 3 deve existir obrigatoriamente);

• c: 0 ou 1; e

J. Bartasevicius 133

Page 134: bartasevicius.com.brbartasevicius.com.br/sites/default/files/2018-12/... · Sobre este trabalho. E permitido: • Compartilhar - copiar e redistribuir o material em qualquer meio

CAPITULO 2. SOLUCOES

• d: 0, 1 ou 2.

Assim, sao quatro possibilidades para o fator 2, duas para o fator 3,duas para o fator 5, e tres para o fator 7:∴ 4× 2× 2× 3 = 48.Alternativa c).

Solucao 99

d1dtotal

A T

Aquiles corre d1 em t1 = d1vA

.

Neste mesmo perıodo de tempo t1, a tartaruga corre, d2 =

d1vAvT

= vTv2Ad1.

Portanto, e possıvel estabelecer a seguinte relacao:

• Aquiles percorre a distancia dn no tempo tn = dnvA

; e

• a tartaruga percorre, no tempo tn, a distancia dn+1 = vTtn.Analogamente, para n > 2, dn = vTtn−1

Isolando dn em ambas as relacoes tem-se:

tnvA = vTtn−1 =⇒ tntn−1

=vT

vA

Ou seja, a razao entre os tempos consecutivos e constante e igual avTvA

, o que define uma progressao geometrica. Como a velocidade datartaruga e menor, tem-se uma progressao com razao menor que 1.Sendo uma progressao geometrica com razao menor que 1, a soma dosseus infinitos termos e convergente:

ttotal =∞∑n=1

t1.

(vT

vA

)n−1

=∞∑n=1

(d1

vA

).

(vT

vA

)n−1

ttotal =

(d1

vA

)+

(d1

vA

).

(vT

vA

)1

+

(d1

vA

).

(vT

vA

)2

+

(d1

vA

).

(vT

vA

)3

+. . .

ttotal =

(vT

vA

)(d1

vA

).

(vA

vT

)+

(d1

vA

)+

(d1

vA

).

(vT

vA

)1

+ . . .︸ ︷︷ ︸ttotal

134 J. Bartasevicius

Page 135: bartasevicius.com.brbartasevicius.com.br/sites/default/files/2018-12/... · Sobre este trabalho. E permitido: • Compartilhar - copiar e redistribuir o material em qualquer meio

CAPITULO 2. SOLUCOES(vA

vT

)ttotal =

[(d1

vT

)+ ttotal

](vA − vT

vT

)ttotal =

d1

vT

ttotal =d1

vA − vT

Sendo a soma infinita de termos ate possuir o tempo de deslocamentoparcial tendendo a zero, isto e, quando a distancia parcial a percorrertambem tender a zero, significa que ttotal representa o tempo que levapara Aquiles alcancar a tartaruga.

De outra forma:

• tempo para Aquiles percorrer a distancia total ate o ponto de

encontro: dtotalvA

; e

• tempo para a tartaruga percorrer a distancia total ate o ponto de

encontro: dtotal−d1vT

.

Como ambos se encontram no ponto, entao os tempos se igualam:

dtotal

vA=dtotal − d1

vT

dtotal =vAd1

vA − vT

O tempo total para a distancia a ser percorrida por Aquiles e

vAd1vA−vTvA

=d1

vA−vT , ∴ igual a ttotal.

Solucao 100 √3 3√x

5x=

√3

5

√3

√x

x3=

√3

5

3√x−1

3

√5x

3√x

=3

√5

3

3

√√x2

x=

3

√5

36√x

J. Bartasevicius 135

Page 136: bartasevicius.com.brbartasevicius.com.br/sites/default/files/2018-12/... · Sobre este trabalho. E permitido: • Compartilhar - copiar e redistribuir o material em qualquer meio

CAPITULO 2. SOLUCOES

No calculo do termo independente, o expoente de x deve ser nulo e assomas das potencias dos membros deve ser 12:(

−13

)m+

(16

)n = 0

m+ n = 12=⇒

m = 4

n = 8

O termo independente e:

C12,4

(√3

5

3√x−1

)4(3

√5

36√x

)8

= 495× 9

25× 25 3

√25

9 3√

9= (×

3√33√3

)

495×3√

753√

27= 165× 3

√75

Alternativa e).

Solucao 101

5 verdes

3 azuis

Sdados < 4

BRANCA

3 verdes

2 azuis

Sdados > 4

PRETA

Total de combinacoes de dois dados: 6× 6 = 36.Dados cuja soma e menor que 4: (1,1), (1,2) e (2,1).

PSdados<4 =3

36=

1

12

PSdados>4 =33

36=

11

12

Probabilidades de retirar bola verde:

• Caixa BRANCA: PBRANCA = 58

• Caixa PRETA:PPRETA = 35

136 J. Bartasevicius

Page 137: bartasevicius.com.brbartasevicius.com.br/sites/default/files/2018-12/... · Sobre este trabalho. E permitido: • Compartilhar - copiar e redistribuir o material em qualquer meio

CAPITULO 2. SOLUCOES

Calculo da probabilidade total de se retirar uma bola verde:

PBRANCA × PSdados<4 + PPRETA × PSdados>4 =

5

8× 1

12+

3

5× 11

12=

289

480

Solucao 102Raiz dupla implica a equacao assumir a forma fatorada de:

(x− r1)(x− r1)(x− r2)

Expandindo:

x3 + (−2r1 − r2)x2 + (r21 + 2r1r2)x− r2

1r2

Equiparando os coeficientes de x2 e x tem-se:3 = −2r1 − r2 =⇒ r2 = −3− 2r1

−2 = r21 + 2r1r2

Substituindo r2 na segunda equacao:

−2 = r21+2r1(−3−2r1) =⇒ 3r2

1+6r1−2 = 0 =⇒

r1 = −1 +

√606

r1 = −1−√

606

Como r1 ∈]0, 1[, entao r1 = −1 +√

606 .

r2 = −3− 2

(−1 +

√60

6

)= −1 +

√60

3

Pela equacao expandida, d = −r21r2:

d = −

(−1 +

√60

6

)2(−1 +

√60

3

)=

84− 11√

60

9

J. Bartasevicius 137

Page 138: bartasevicius.com.brbartasevicius.com.br/sites/default/files/2018-12/... · Sobre este trabalho. E permitido: • Compartilhar - copiar e redistribuir o material em qualquer meio

CAPITULO 2. SOLUCOES

Solucao 103

7− 4√

3 = (A−B√

3)2 =⇒

A2 + 3B2 = 7

2AB = 4 =⇒ A = 2B

3B4 − 7B2 + 4 = 0 =⇒

B = ±

√43

B = ±1

Prefere-se utilizar A,B ∈ Q∗+, necessariamente com A − B√

3 > 0.Entao B = 1. Logo A = 2.

7− 4√

3 = (2−√

3)2

∴ x =

√(2−

√3)2 +

√3 = 2 ∈ Q

Alternativa b).

Solucao 104Sendo 0 e −1 raızes de p(x), entao 5

√p(0) = 0 e 5

√p(−1) = 0.

Entao, ax2 − 2bx+ c+ 1 = 0 para x = 0 e x = −1.Resolvendo para x = 0:

a.02 − 2b.0 + c+ 1 = 0 =⇒ c = −1

Resolvendo para x = −1, substituindo c por −1:

a.(−1)2 − 2b.(−1) + (−1) + 1 = 0 =⇒ a = −2b

Decompondo p(x):

p(x) =(ax2 + ax

)5= a5x5 (x+ 1)5

A decomposicao de (x+ 1)5 e que produzira os coeficientes dos termos

do polinomio. A soma dos coeficientes sera5∑

n=0

5!

n!(5− n)!xn. Por se

tratar de um binomio de Newton, o resultado e 25 = 32.Assim, a soma final dos coeficientes do polinomio p(x) e: 32a5 = 32=⇒ a = 1.

a = 1 =⇒ b = −1

2

138 J. Bartasevicius

Page 139: bartasevicius.com.brbartasevicius.com.br/sites/default/files/2018-12/... · Sobre este trabalho. E permitido: • Compartilhar - copiar e redistribuir o material em qualquer meio

CAPITULO 2. SOLUCOES

∴ a+ b+ c = 1 +

(−1

2

)+ (−1) = −1

2.

Alternativa a).

Solucao 105

√n−√n− 1 < 0.01 =⇒

√n− 0.01 <

√n− 1

Elevando ao quadrado:

n− 2.10−2√n+ 10−4 < n− 1 =⇒√n >

1 + 10−4

2.10−2=⇒

√n > 50 + 0.005

∴ n > 2500 + δ, onde δ < 1. Entao o primeiro inteiro positivo n e 2501.Alternativa b).

Solucao 106(a) Se α e solucao da equacao, entao α3 + 3α− 4 = 0.Desenvolvendo α:

α3 = 2 +√

5 + 3

(3√

2 +√

5

)2(3√

2−√

5

)+ 3

(3√

2 +√

5

)(3√

2−√

5

)2

+ 2−√

5 =⇒

α3 = 4 + 3

(3√

2 +√

5

)(3√

2−√

5

)(3√

2 +√

5 +3√

2−√

5

)=⇒

α3 = 4 + 3 (−1)

3√

2 +√

5 +3√

2−√

5︸ ︷︷ ︸α

α3 = 4− 3α =⇒ α3 + 3α− 4 = 0

∴ α e raız de x3 + 3x− 4.(b) α e um numero positivo.Fatorando x3 + 3x− 4:

x3 − x+ 4x− 4 = x(x2 − 1) + 4(x− 1) = x(x+ 1)(x− 1) + 4(x− 1) =

J. Bartasevicius 139

Page 140: bartasevicius.com.brbartasevicius.com.br/sites/default/files/2018-12/... · Sobre este trabalho. E permitido: • Compartilhar - copiar e redistribuir o material em qualquer meio

CAPITULO 2. SOLUCOES

(x− 1) (x(x+ 1) + 4) = (x− 1)(x2 + x+ 4)

Uma das raızes e 1. Ao calcular as raızes do fator de 2o grau tem-se:

∆ = 12 − 16 = −15

Uma equacao de 3o grau, deve fornecer, no maximo, 3 raızes reais. Ofator de 2o grau, que fornece duas raızes, nao possui raızes reais. Pelafatoracao do polinomio, a unica raiz real e 1. A unica raiz real conhecidae α.∴ 3√

2 +√

5 +3√

2−√

5 = 1.

Solucao 107(a)

x =√

1 +mx−√

1−mx

x2 = 1 +mx− 2√

1−m2x2 + 1−mx

x2 − 2 = −2√

1−m2x2 =⇒ x4 + x2(−4 + 4m2) = 0 =⇒x = 0 ou

x = ±2√

1−m2

(b) Para admitir solucao nao nula, 1−m2 > 0.∴ −1 < m < 1.

Solucao 108

ax − 1ax

ax + 1ax

= m =⇒ a2x − 1

a2x + 1= m =⇒

a2x − 1 = (a2x + 1)m =⇒ a2x =1 +m

1−mCom a > 0, 1+m

1−m > 0:

− + +

+ + −

− + −

1 +m

1−m

1+m1−m−1 1

140 J. Bartasevicius

Page 141: bartasevicius.com.brbartasevicius.com.br/sites/default/files/2018-12/... · Sobre este trabalho. E permitido: • Compartilhar - copiar e redistribuir o material em qualquer meio

CAPITULO 2. SOLUCOES

∴ m ∈]− 1, 1[.Alternativa c).

Solucao 109Acertar somente 7 questoes implica errar 3 questoes.Cada questao possui 5 alternativas, portanto e possıvel errar umaquestao de 4 maneiras diferentes.Num conjunto de 10 questoes, podem ser formados

(103

)grupos diferentes

de 3 questoes.Assim, o numero de maneiras diferentes para acertar exatamente 7questoes e: (

10

3

)× 43 = 120× 43 = 30× 44

Alternativa a).

Solucao 110

log8

(4k√

2)

=log2

(22k+1

2

)log2 23

=2k + 1

2

3=

4k + 1

6

Portanto,101∑k=0

4k+16 e uma progressao aritmetica com termo inicial 1

6 e

razao 23 .

S =102

2

(1

6+

405

6

)= 3451

Portanto:

• afirmativa I e falsa, pois nao e uma progressao geometrica;

• afirmativa II e verdadeira;

• afirmativa III e verdadeira; e

• afirmativa IV e falsa, pois a soma e 3451, maior que 3434+log8

√2.

Alternativa b).

J. Bartasevicius 141

Page 142: bartasevicius.com.brbartasevicius.com.br/sites/default/files/2018-12/... · Sobre este trabalho. E permitido: • Compartilhar - copiar e redistribuir o material em qualquer meio

CAPITULO 2. SOLUCOES

Solucao 111Calculo do determinante da matriz das variaveis:∣∣∣∣a− b −(a+ b)

a+ b a− b

∣∣∣∣ = (a− b)2 − (−(a+ b)(a+ b)) = 2(a2 + b2)

I. se a = b = 0, tera as igualdades 0 = 1 em ambas as equacoes dosistema, tornando o sistema impossıvel. FALSO.

II. se a e b nao forem simultaneamente nulos, entao 2(a2 + b2) 6= 0.Portanto o sistema sera possıvel e determinado. VERDADEIRO.

III.

x =

∣∣∣∣1 −(a+ b)1 a− b

∣∣∣∣∣∣∣∣a− b −(a+ b)a+ b a− b

∣∣∣∣ =2a

2(a2 + b2)=

a

a2 + b2

y =

∣∣∣∣a− b 1a+ b 1

∣∣∣∣∣∣∣∣a− b −(a+ b)a+ b a− b

∣∣∣∣ =−2b

2(a2 + b2)=

−ba2 + b2

x2 + y2 =

(a

a2 + b2

)2

+

(−b

a2 + b2

)2

x2 + y2 =a2 + b2(a2 + b2

)2 =1

a2 + b2

VERDADEIRO.

Alternativa e).

Solucao 112

(1 + x+ x2)9 =

(1 + x)︸ ︷︷ ︸A

+ x2︸︷︷︸B

9

142 J. Bartasevicius

Page 143: bartasevicius.com.brbartasevicius.com.br/sites/default/files/2018-12/... · Sobre este trabalho. E permitido: • Compartilhar - copiar e redistribuir o material em qualquer meio

CAPITULO 2. SOLUCOES

Deve-se combinar as potencias de x nos termos A e B, a fim de obterx4.B podera ter as potencias 0, 1 e 2.

• B = 0 =⇒ potencia de A = 9.Deve-se calcular o coeficiente do termo x4 de (1 + x)9:

C9,0x0 × (1 + x)9 = C9,0x

0 × C9,4x4 = 1× 126x4 = 126x4

• B = 1 =⇒ potencia de A = 8.Deve-se calcular o coeficiente do termo x2 de (1 + x)8:

C9,1(x2)1 × (1 + x)8 = C9,1x2 × C8,2x

2 = 9× 28x4 = 252x4

• B = 2 =⇒ potencia de A = 7.Deve-se calcular o coeficiente do termo x0 de (1 + x)7:

C9,2(x2)2 × (1 + x)7 = C9,2x4 × C7,0x

0 = 36× 1x4 = 36x4

∴ (126 + 252 + 36)x4 = 414x4.

Solucao 113Numero de elementos de A = 14.Numero de elementos de B = 6.Como B e sub-conjunto de A, entao todos os elementos de B pertencema A. Portanto, o numero de elementos de A disjuntos de B e 14−6 = 8.Numero total de subconjuntos do conjunto A− B: 28.Numero de subconjuntos do conjunto A−B com 7 elementos: C8,7 = 8.Numero de subconjuntos do conjunto A−B com 8 elementos: C8,8 = 1.∴ numero de subconjuntos de A− B com 6 ou menos elementos:

28 − 8− 1 = 28 − 9.

Alternativa a).

Solucao 114

logk(xy) = 49 =⇒ logk x+ logk y = 49 (A)

J. Bartasevicius 143

Page 144: bartasevicius.com.brbartasevicius.com.br/sites/default/files/2018-12/... · Sobre este trabalho. E permitido: • Compartilhar - copiar e redistribuir o material em qualquer meio

CAPITULO 2. SOLUCOES

logk(x/z) = 44 =⇒ logk x− logk z = 44 (B)

logk x, logk y e logk z sao primos e de acordo com (A) e (B):

44 < logk x < 49 =⇒ logk x = 47

Logo, logk y = 49− 47 = 2 e logk z = 47− 44 = 3.

logk(xyz) = logk x+ logk y + logk z = 47 + 2 + 3 = 52

Alternativa a).

Solucao 115

ex, ey,ex − 2

√5

4− ey√

5∈ Q

∴ ex =A

Be ey =

C

D

Substituindo em ex−2√

54−ey

√5:

AB − 2

√5

4− CD

√5

Racionalizando: (AB − 2

√5)(

4 + CD

√5)

16− 5C2

D2

Numerador:4A

B− 8√

5 +AC

BD

√5− 10C

D

O denominador e um numero racional, pois CD e racional. Para que

ex−2√

54−ey

√5

seja racional, o numerador deve ser racional. Entao, todos os

termos√

5 devem ser anulados:

∴AC

BD= 8

Calculo de x+ y:

ex =A

Be ey =

C

D=⇒ exey =

AC

BD=⇒ ex+y =

AC

BD

144 J. Bartasevicius

Page 145: bartasevicius.com.brbartasevicius.com.br/sites/default/files/2018-12/... · Sobre este trabalho. E permitido: • Compartilhar - copiar e redistribuir o material em qualquer meio

CAPITULO 2. SOLUCOES

Mas ACBD = 8, entao:

ex+y = 8 =⇒ (x+ y) ln e = ln 23 =⇒ x+ y = 3 ln 2

Alternativa e).

Solucao 116log(x+ 2y)− log(w − 3z) = 0 =⇒ x+ 2y = w − 3z

2x+3z = 2y−3z+w+3 =⇒ x+ 3z = y − 3z + w + 3

2x+ y + 6z − 2w = 23 =⇒ 2x+ y + 6z − 2w = 8x+ 2y + 3z − w = 0

x− y + 6z − w = 3

2x+ y + 6z − 2w = 8

(A)

De log[(x+ 2y) (w − 3z)−1

], define-se o domınio:

x+ 2y

w − 3z> 0 ∧ w − 3z 6= 0 (B)

Multiplicando a ultima equacao do sistema em (A) por (−1) e depoissomando todas as equacoes:

x+ 2y + 3z − w = 0

x− y + 6z − w = 3

−2x− y − 6z + 2w = −8

3z = −5 =⇒ z = −53

Substituindo z no sistema (A):x+ 2y + 3

(−5

3

)− w = 0

x− y + 6(−5

3

)− w = 3

2x+ y + 6(−5

3

)− 2w = 8

=⇒

x+ 2y − w = 5

x− y − w = 13

2x+ y − 2w = 18

(C)

Da primeira equacao do sistema (C), tem-se w = x+ 2y − 5.Substituindo w nas demais equacoes:

x− y − (x+ 2y − 5) = 13

2x+ y − 2(x+ 2y − 5) = 18=⇒

−3y = 8

−3y = 8=⇒ y = −8

3

J. Bartasevicius 145

Page 146: bartasevicius.com.brbartasevicius.com.br/sites/default/files/2018-12/... · Sobre este trabalho. E permitido: • Compartilhar - copiar e redistribuir o material em qualquer meio

CAPITULO 2. SOLUCOES

Substituindo y em w = x+ 2y − 5, tem-se:

w = x+ 2

(−8

3

)− 5 =⇒ x− w =

31

3

Da relacao de domınio em (B):

x+ 2−83

w − 3−53

> 0 =⇒x− 16

3

w + 5> 0

∴ x 6= 16

3∧ w 6= −5

Finalmente, a solucao do sistema e:

S =

(x, y, w, z) ∈ R4 |

y = −8

3∧ z = −5

3∧ x− w =

31

3∧ x 6= −16

3∧ w 6= −5

Solucao 117Se x > 0, entao |5x − 1| = 5x − 1.Calculo das raızes de

∣∣53x − 52x+1 + 4.5x∣∣:

53x − 52x+1 + 4.5x = 0 =⇒ 5x(

52x − 5.5x + 4)

= 0

5x(

52x − 5.5x + 4)

= 0 =⇒

5x = 4 =⇒ x = log5 4 ou

5x = 1 =⇒ x = 0

Portanto, ha 3 hipoteses:

• x < 0:|5x − 1| = −5x + 1 e

∣∣53x − 52x+1 + 4.5x∣∣ = 53x − 52x+1 + 4.5x:

−5x + 1 = 53x − 52x+1 + 4.5x =⇒ 53x − 5.52x + 5.5x − 1 = 0

Substituindo y = 5x:

y3 − 5y2 + 5y − 1 = 0

Como x = 0 e raiz de 5x − 1 e de 53x − 52x+1 + 4.5x, entao y = 1e uma raiz.

(y−1)(y2−4y+1) = 0 =⇒

y = 2 +

√3 =⇒ x = log5

(2 +√

3)

y = 2−√

3 =⇒ x = log5

(2−√

3)

log5

(2 +√

3)> 0, entao este valor nao e raiz.

2−√

3 < 1 =⇒ log5

(2−√

3), ∴ log5

(2−√

3)

e raiz.

146 J. Bartasevicius

Page 147: bartasevicius.com.brbartasevicius.com.br/sites/default/files/2018-12/... · Sobre este trabalho. E permitido: • Compartilhar - copiar e redistribuir o material em qualquer meio

CAPITULO 2. SOLUCOES

• 0 6 x < log5 4:|5x − 1| = 5x − 1 e

∣∣53x − 52x+1 + 4.5x∣∣ = −53x + 52x+1 − 4.5x:

5x − 1 = −53x + 52x+1 − 4.5x =⇒ 53x − 5.52x + 5.5x − 1 = 0

E a mesma equacao do item anterior. As possıveis raızes sao:x = log5

(2 +√

3)

x = log5

(2−√

3)

0 < log5

(2 +√

3)< log5 4, entao este valor e raiz.

log5

(2−√

3)< 0, portanto este valor nao e raiz.

• x > log5 4:|5x − 1| = 5x − 1 e

∣∣53x − 52x+1 + 4.5x∣∣ = 53x − 52x+1 + 4.5x:

5x − 1 = 53x − 52x+1 + 4.5x =⇒ 53x − 5.52x + 3.5x + 1 = 0

Substituindo y = 5x:

y3 − 5y2 + 3y + 1 = 0

Como x = 0 e raiz de 5x − 1 e de 53x − 52x+1 + 4.5x, entao y = 1e uma raiz.

(y−1)(y2−4y−1) = 0 =⇒

y = 2 +

√5 =⇒ x = log5

(2 +√

5)

y = 2−√

5 =⇒ x = log5

(2−√

5)

2 +√

5 > 4 =⇒ log5

(2 +√

5)> log5 4, ∴ este valor e raiz.

2−√

5 < 4 =⇒ log5

(2 +√

5)< log5 4, portanto este valor nao

e raiz.

∴ as solucoes sao: 0, log5

(2−√

3), log5

(2 +√

3)

e log5

(2 +√

5).

Alternativa d).

Solucao 118Existem g carros GNV.Existem f carros “flex”.

J. Bartasevicius 147

Page 148: bartasevicius.com.brbartasevicius.com.br/sites/default/files/2018-12/... · Sobre este trabalho. E permitido: • Compartilhar - copiar e redistribuir o material em qualquer meio

CAPITULO 2. SOLUCOES

GNVg

“flex”bicombustıvel

f

bicombustıvel tricombustıvel

36% 36%

Apos as conversoes, tem os seguintes resultados:

• g − 0.36g automoveis movidos a GNV;

• 0.36g + (f − 0.36f) automoveis bi-combustıveis; e

• 0.36f automoveis tri-combustıveis.

O seguinte sistema deve ser resolvido:g + f = 1000

0.36g + (f − 0.36f) = 556=⇒

g + f = 1000

0.36g + 0.64f = 556

Entao, g = 300 e f = 700.O numero de veıculos tri-combustıveis e 0.36f . ∴ 0.36× 700 = 252.Alternativa b).

Solucao 119As raızes formam uma progressao geometrica:

x1 = x2q

x2 = x2

x3 = x2q

148 J. Bartasevicius

Page 149: bartasevicius.com.brbartasevicius.com.br/sites/default/files/2018-12/... · Sobre este trabalho. E permitido: • Compartilhar - copiar e redistribuir o material em qualquer meio

CAPITULO 2. SOLUCOES

Da equacao:

x1x2x3 = −54

2=⇒ x2qx2

x2

q= −27 =⇒ x3

2 = −27

∴ x2 = −3

Ainda da equacao:

x1 + x2 + x3 = −−a2

=⇒ x2q + x2 +x2

q=a

2=⇒

−3q − 3− 3

q=a

2=⇒ −6(q2 + q + 1) = aq =⇒ a =

−6(q2 + q + 1)

q

E, tambem da equacao:

x1x2 + x1x3 + x2x3 =b

2=⇒ x2qx2 + x2q

x2

q+ x2

x2

q=b

2=⇒

9q + 9 +9

q=b

2=⇒ 18(q2 + q + 1) = bq =⇒ b =

18(q2 + q + 1)

q

Finalmente:

a

b=

−6(q2+q+1)q

18(q2+q+1)q

= −1

3

Alternativa b).

Solucao 120

sin

(x+

3

)= − cosx

Substituindo na expressao:

2[− cosx+ cos2 x

sin2 x

]sin x

2cos x

2

1 +sin2 x

2cos2 x

2

=2[− cosx+ cos2 x

sin2 x

]sin x

2

1cos x

2

=

= 2 sinx

2cos

x

2︸ ︷︷ ︸sinx

[− cosx sin2 x+ cos2 x

sin2 x

]=

= sinx cosx

[− sin2 x+ cosx

sin2 x

]=

J. Bartasevicius 149

Page 150: bartasevicius.com.brbartasevicius.com.br/sites/default/files/2018-12/... · Sobre este trabalho. E permitido: • Compartilhar - copiar e redistribuir o material em qualquer meio

CAPITULO 2. SOLUCOES

=cosx

sinx

(cosx− sin2 x

)= tan−1 x

(cosx− sin2 x

)Alternativa a).

Solucao 121Seja xn uma das 11 raızes da equacao algebrica.

xn = β + iγn

Considerando a raiz do meio x6 = β + iγ6, sendo uma PA, as demaisraızes serao da forma:

xn = β + i[γ6 + (n− 6)γ]

Analise das afirmativas:

I. β = 0→ a0 = 0−a0 e o produto das raızes. Neste caso, as raızes serao da formaxn = i[γ6 + (n− 6)γ].

−a0 =11∏n=1

i[γ6 + (n− 6)γ]

−a0 = i11γ6(γ26−γ

2)(γ26−4γ2)(γ2

6−9γ2)(γ26−16γ2)(γ2

6−25γ2) =⇒

−a0 = −iγ6(γ26 − γ

2)(γ26 − 4γ2)(γ2

6 − 9γ2)(γ26 − 16γ2)(γ2

6 − 25γ2)

Para que a0 = 0, um dos fatores do produto acima deve ser zero.Desta maneira, β = 0 nao e condicao suficiente para que a0 = 0.∴ β = 0→ a0 = 0 e FALSA.

II. a10 = 0→ β = 0

a10 = −11∑n=1

[β + i [γ6 + (n− 6)γ]]

a10 = −11β − 11iγ6

Para que a10 = 0, β = 0 e condicao necessaria, apesar de nao sersuficiente.∴ a10 = 0→ β = 0 e VERDADEIRA.

150 J. Bartasevicius

Page 151: bartasevicius.com.brbartasevicius.com.br/sites/default/files/2018-12/... · Sobre este trabalho. E permitido: • Compartilhar - copiar e redistribuir o material em qualquer meio

CAPITULO 2. SOLUCOES

III. β = 0→ a1 = 0

a1 =11∑n=1

11∏j=1

xj

xn=

11∑n=1

−a0

xn= −a0

11∑n=1

1

xn

Conforme visto em I., nao se pode afirmar que a0 6= 0. Entao,

resta analisar se11∑n=1

1xn

= 0 para satisfazer a afirmacao:

a1 = −a0

(1

i(γ6 − 5γ)+

1

i(γ6 − 4γ)+ . . .+

+1

i(γ6 + 4γ)+

1

i(γ6 + 5γ)

)

a1 = ia0

(1

γ6 − 5γ+

1

γ6 − 4γ+ . . .+

1

γ6 + 4γ+

1

γ6 + 5γ

)

a1 = ia0

(2γ6

γ26 − 25γ2

+2γ6

γ26 − 16γ2

+2γ6

γ26 − 9γ

+2γ6

γ26 − 4γ2

+

+2γ6

γ26 − γ2

+1

γ6

)

Para que 2γ6γ26−25γ2

+ 2γ6γ26−16γ2

+ 2γ6γ26−9γ

+ 2γ6γ26−4γ2

+ 2γ6γ26−γ2

+ 1γ6

= 0, e

necessario que existam outras relacoes entre γ e γ6, alem de β = 0.Entao, β = 0 nao e condicao suficiente para que a1 = 0.∴ β = 0→ a1 = 0 e FALSA.

Solucao 122

x5 − y5 = (x− y)(x4 + x3y + x2y2 + xy3 + y4) =

= (x− y)(x4 + y4 + xy

(x2 + xy + y2

))(A)

Fazendo x = 2√

3 +√

5 e y = 2√

3−√

5:

x2 = 17 + 4√

15 (B)

y2 = 17− 4√

15 (C)

J. Bartasevicius 151

Page 152: bartasevicius.com.brbartasevicius.com.br/sites/default/files/2018-12/... · Sobre este trabalho. E permitido: • Compartilhar - copiar e redistribuir o material em qualquer meio

CAPITULO 2. SOLUCOES

x4 =(x2)2

= 529 + 136√

15 (D)

y4 =(y2)2

= 529− 136√

15 (E)

xy =(

2√

3 +√

5)(

2√

3−√

5)

= 12− 5 = 7 (F)

Substituindo (B), (C), (D), (E) e (F) em (A), tem-se:(2√

3 +√

5−(

2√

3−√

5))(

529 + 136√

15 + 529− 136√

15+

+7(

17 + 4√

15 + 7 + 17− 4√

15))

=

(2√

5)

(1058 + 7 (41)) = 2690√

5

Alternativa b).

Solucao 123Sao 6 refletores.Probabilidade do refletor aceso: Paceso = 2

3 .

Probabilidade do refletor apagado: Papagado = 1− 23 = 1

3 .Probabilidade de 4 refletores acesos:

P4 acesos = C6,4

(2

3

)4(1

3

)2

Probabilidade de 5 refletores acesos:

P5 acesos = C6,5

(2

3

)5(1

3

)1

Probabilidade de 4 ou 5 refletores acesos: Ptotal = P4 acesos +P5 acesos:

Ptotal =6!

4!2!× 24

34× 12

32+

6!

5!1!× 25

35× 1

3=

16

27

Alternativa a).

152 J. Bartasevicius

Page 153: bartasevicius.com.brbartasevicius.com.br/sites/default/files/2018-12/... · Sobre este trabalho. E permitido: • Compartilhar - copiar e redistribuir o material em qualquer meio

CAPITULO 2. SOLUCOES

Solucao 124Domınio de f(x):

f(x) =3x − 1

3x

2=⇒ f(x) =

32x − 1

2.3x

3x 6= 0 ∀x ∴ Domınio = R

Imagem de f(x):

f(x) =32x − 1

2.3x=

(3x + 1) (3x − 1)

2.3x

Se

x < 0 =⇒ f(x) < 0

x = 0 =⇒ f(x) = 0

x > 0 =⇒ f(x) > 0

e, se

x→ −∞ =⇒ f(x)→ −∞x→ +∞ =⇒ f(x)→ +∞

.

Portanto, Imagem = R.Assim, para cada valor do domınio em x existe um unico valor de f(x)correspondente. Da mesma forma, para cada valor da imagem f(x)existe um unico valor de x correspondete. Logo, a funcao e bijetora.Calculo de f−1:

(3x + 1) (3x − 1)

2.3x= y =⇒ 32x − 2y3x − 1 = 0

3x =

y +

√y2 + 1 ou

y −√y2 + 1

y −√y2 + 1 < 0, portanto nao e uma raiz valida para 3x.

Assim, x = log3

(y +

√y2 + 1

).

∴ f−1 = log3

(x+√x2 + 1

).

Solucao 125

• Numero de bolas multiplos de 5: 90/5 = 18.

• Numero de bolas multiplos de 6: 90/6 = 15.

• Numero de bolas multiplos de 5 e 6 simultaneamente (multiplo de30): 90/30 = 3.

J. Bartasevicius 153

Page 154: bartasevicius.com.brbartasevicius.com.br/sites/default/files/2018-12/... · Sobre este trabalho. E permitido: • Compartilhar - copiar e redistribuir o material em qualquer meio

CAPITULO 2. SOLUCOES

a) Numero de bolas multiplos de 5 ou 6: 18 + 15− 3 = 30.Calculo da probabilidade: P = 30

90 = 13 .

b) Ha duas possibilidades para o sorteio da primeira bola: a primeirabola ser um multiplo de 6, ou a bola nao ser um multiplo de 6.b.1) Primeira bola e multiplo de 6:A probabilidade de a primeira bola ser um multiplo de 6 e: P (bola 1 =m.6) = 15

90 = 16 .

A probabilidade de a segunda bola nao ser um multiplo de 6, nesta

condicao, e: P (bola 2 6= m.6) =89−(15−1)

90−1 = 7589 .

b.2) Primeira bola nao e multiplo de 6:A probabilidade de a primeira bola nao ser um multiplo de 6 e: P (bola 1 6=m.6) = 75

90 = 56 .

A probabilidade de a segunda bola nao ser um multiplo de 6, nestacondicao, e: P (bola 2 6= m.6) = 89−15

90−1 = 7489 .

Calculo da probabilidade de a segunda bola nao ser um multiplo de 6:

P =1

6× 75

89+

5

6× 74

89=

445

534

Solucao 126Calculo do determinante do sistema:∣∣∣∣∣∣

1 2 30 1 23 −1 −5c

∣∣∣∣∣∣ = 5− 5c

Para que o sistema nao seja possıvel e determinado, 5− 5c = 0, ou seja,c = 1. Quando c 6= 1,o sistema e possıvel e determinado.Na hipotese de c = 1, o sistema podera ser impossıvel, ou possıvel eindeterminado, dependendo dos valores de a e b:

Se c = 1, entao

x+ 2y + 3z = a

y + 2z = b

3x− y − 5z = 0

Da 2a equacao, y = b− 2z. Substituindo nas 1a e 3a equacoes:x+ 2(b− 2z) + 3z = a =⇒ x− z = a− 2b

3x− (b− 2z)− 5z = 0 =⇒ 3(x− z) = b =⇒ x− z = b3

154 J. Bartasevicius

Page 155: bartasevicius.com.brbartasevicius.com.br/sites/default/files/2018-12/... · Sobre este trabalho. E permitido: • Compartilhar - copiar e redistribuir o material em qualquer meio

CAPITULO 2. SOLUCOES

Se a− 2b = b3 , o sistema sera possıvel e indeterminado. Caso contrario,

o sistema sera impossıvel.

o sistema sera possıvel e determinado se c 6= 1 e

o sistema sera possıvel e indeterminado se a = 7b3

Alternativa b).

Solucao 127

Substituindo

ex = X e

ey = Y:

4X2 + 9Y 2 − 16X − 54Y + 61 = 0 =⇒4X2 − 16X + 16 + 9Y 2 − 54Y + 81 = 97− 61 =⇒

4(X − 2)2 + 9(Y − 3)2 = 36 =⇒(X − 2)2

32+

(Y − 3)2

22= 1

A equacao acima e a representacao da elipse, com centro em (2, 3), comeixo de comprimento horizontal 6 e eixo de comprimento vertical 4.

Y

X

Como os eixos representados sao X e Y , a escala e logarıtmica, comX = ex e

Y = ey.

ex > 0 e ey > 0, portanto somente os pontos localizados no primeiroquadrante podem ser considerados. Considerando que existem infinitospontos no primeiro quadrante, logo ha infinitas solucoes.Alternativa d).

J. Bartasevicius 155

Page 156: bartasevicius.com.brbartasevicius.com.br/sites/default/files/2018-12/... · Sobre este trabalho. E permitido: • Compartilhar - copiar e redistribuir o material em qualquer meio

CAPITULO 2. SOLUCOES

Solucao 128

24 < 2−2

(log 1

5(x2−x+19)

)

Como a base e maior que 1, os expoentes devem manter a ordem parasatisfazer a desigualdade:

4 < −2(

log 15

(x2 − x+ 19

))=⇒

2 < −

(log5

(x2 − x+ 19

)log5

15

)=⇒

2 < log5

(x2 − x+ 19

)=⇒ x2 − x+ 19 > 52 =⇒

x2 − x− 6 > 0

−2 3

y

x+ +

∴ S = x ∈ R | x < −2 ∪ x > 3.

Solucao 129As raızes sao:

x =−2m±

√8m2 − 16

2(2−m)=⇒ x =

−m±√

2m2 − 4

2−m

156 J. Bartasevicius

Page 157: bartasevicius.com.brbartasevicius.com.br/sites/default/files/2018-12/... · Sobre este trabalho. E permitido: • Compartilhar - copiar e redistribuir o material em qualquer meio

CAPITULO 2. SOLUCOES

Existirem raızes distintas e reais implica 2m2 − 4 > 0 e 2−m 6= 0.Portanto as condicoes de m sao:

Domınio =m ∈ R | m 6= 2 ∧

(m < −

√2 ∨m >

√2)

(A)

Ambas as raızes devem ser maiores que zero. Portanto:

−m+√

2m2 − 4

2−m> 0 ∧ −m−

√2m2 − 4

2−m> 0

Para as inequacoes, m deve ser dividido em dois grupos - que fazem onumerador ser maior ou menor que zero:

a. 2−m > 0 (ou m < 2):Quando m < 2 devem ser satisfeitas duas condicoes:

−m+√

2m2 − 4 > 0 =⇒√

2m2 − 4 > m (I) e

−m−√

2m2 − 4 > 0 =⇒ −√

2m2 − 4 > m (II)

Resolvendo a inequacao (I):√2m2 − 4 > m

A expressao acima e valida para quaisquer valores de m < 0.Para valores de 0 < m < 2, deve-se retirar o radical:

2m2 − 4 > m2 =⇒ m2 > 4 =⇒ m > 2 ∧m < −2

Neste caso, nao ha solucao para 0 < m < 2.A interseccao dos valores de Domınio em (A) com m < 0 e:

m < −√

2 (B)

Resolvendo a inequacao (II):

m < −√

2m2 − 4

A inequacao acima e falsa para qualquer m > 0.Para valores de m < 0, deve-se retirar o radical. Mas sabendoque m < 0, a inequacao acima tera ambos os termos direito eesquerdo menores que zero. Neste caso, ao elevar ambos os ladosao quadrado, deve-se levar em consideracao de que m deve sermaior que

√2m2 − 4:

m2 > 2m2 − 4 =⇒ m2 < 4 =⇒ −2 < m < 2

J. Bartasevicius 157

Page 158: bartasevicius.com.brbartasevicius.com.br/sites/default/files/2018-12/... · Sobre este trabalho. E permitido: • Compartilhar - copiar e redistribuir o material em qualquer meio

CAPITULO 2. SOLUCOES

Se m < 0, entao a solucao para a inequacao (II) e:

−2 < m < 0 (C)

Ambas as inequacoes (I) e (II) devem ser satisfeitas para quandom < 2. Portanto deve-se fazer a interseccao entre (B) e (C):(

m < −√

2)∩ (−2 < m < 0) = −2 < m < −

√2 (D)

b. 2−m < 0 (ou m > 2):Quando m > 2 devem ser satisfeitas duas condicoes:

−m+√

2m2 − 4 < 0 =⇒√

2m2 − 4 < m (I) e

−m−√

2m2 − 4 < 0 =⇒√

2m2 − 4 > −m (II)

Na resolucao da inequacao (I) tem-se que m >√

2m2 − 4, portantoso ha solucao se m > 0.Sabendo-se desta restricao, pode-se retirar o radical:

m2 > 2m2 − 4 =⇒ m2 < 4 =⇒ −2 < m < 2

Esta solucao, levando em consideracao que m > 2 produz umconjunto vazio como solucao.Se a solucao da inequacao (I) e um conjunto vazio, a interseccaocom a solucao da inequacao (II) tambem sera um conjunto vazio.Logo, nao ha solucao para m > 2.

Por fim, a solucao esta em (D): S =m ∈ R | − 2 < m < −

√2

.

Solucao 130

x+ 5y + 10z = 25, x, y, z ∈ R

Existem 3 valores para z:

a. z = 0:

x+ 5y = 25

Sao 255 + 1 = 6 possibilidades de valores para y: 0,1,2,3,4,5.

158 J. Bartasevicius

Page 159: bartasevicius.com.brbartasevicius.com.br/sites/default/files/2018-12/... · Sobre este trabalho. E permitido: • Compartilhar - copiar e redistribuir o material em qualquer meio

CAPITULO 2. SOLUCOES

b. z = 1:

x+ 5y + 10 = 25 =⇒ x+ 5y = 15

Sao 155 + 1 = 4 possibilidades de valores para y: 0,1,2,3.

c. z = 2:

x+ 5y + 20 = 25 =⇒ x+ 5y = 5

Sao 55 + 1 = 2 possibilidades de valores para y: 0,1.

Total de possibilidades: 6 + 4 + 2 = 12.Alternativa d).

Solucao 131Probabilidade de acerto do tiro: 1

3 .

Probabilidade de erro do tiro: 1− 13 = 2

3 .

A probabilidade de o pote nao ser atingido nenhuma vez e(

23

)2= 4

9 .

∴ a probabilidade de ser atingido ao menos uma vez e 1− 49 = 5

9 .Alternativa d).

Solucao 132Definicao do Domınio:nao ha restricoes para x e a funcao f(x) esta definida para todo x ∈ R.∴ Domınio=R.

Calculo da Imagem:

• x < 03− x2 e uma funcao decrescente para x < 0.Neste caso f(x) < 3− 02 =⇒ f(x) < 3.Para x→ −∞, f(x)→ −∞.

• x > 03 + x2 e uma funcao crescente para x > 0.Neste caso f(x) > 3 + 02 =⇒ f(x) > 3.Para x→ +∞, f(x)→ +∞.

J. Bartasevicius 159

Page 160: bartasevicius.com.brbartasevicius.com.br/sites/default/files/2018-12/... · Sobre este trabalho. E permitido: • Compartilhar - copiar e redistribuir o material em qualquer meio

CAPITULO 2. SOLUCOES

Imagem =]−∞, 3[∪[3,+∞[ =⇒ Imagem = R.Para todo valor do domınio ha um unico valor correspondente para aimagem e vice-versa, ∴ f(x) e bijetora.Calculo de f−1:

• Quando x < 0

3− x2 = y =⇒ x = ±√

3− y

Neste caso, como x < 0, entao x = −√

3− y, para y < 3.

• Quando x > 0

3 + x2 = y =⇒ x = ±√y − 3

Neste caso, como x > 0, entao x = +√y − 3, para y > 3.

∴ f−1 =

−√

3− x, x < 3√x− 3, x > 3

Solucao 133

sin θ logtan θ e > 0

Restricoes do domınio para a base do logaritmo:

tan θ > 0 ∧ tan θ 6= 1

(0 < θ <

π

2∨ π < θ <

2

)∧(θ 6∈

π

4,5π

4

)(A)

Simplificando a inequacao:

sin θ logtan θ e > 0 =⇒

sin θ

ln tan θ> 0

160 J. Bartasevicius

Page 161: bartasevicius.com.brbartasevicius.com.br/sites/default/files/2018-12/... · Sobre este trabalho. E permitido: • Compartilhar - copiar e redistribuir o material em qualquer meio

CAPITULO 2. SOLUCOES

Estudo de sinais:sin θ:

0 π 2π

+ −

ln tan θ:

Se

tan θ > 1, ln tan θ > 0

0 < tan θ < 1, ln tan θ < 0:

0 π4

π2

π 5π4

3π2

− + − +nao

definidonao

definido

Analise de sinais de logtan θ esin θ:

0 π 2π

+ −sin(θ)

0 π4

π2

π 5π4

3π2

− + − +nao

definidonao

definidoln tan θ

0 π4

π2

π 5π4

+ +logtan θ e

sin θ

Interseccao com o domınio:

0 π2

π 3π2

2πDomınio

0 π4

π2

π 5π4

2πlogtan θ e

sin θ

0 π4

π2

π 5π4

2πResultado

∴ S = θ ∈ R, 0 6 θ 6 2π | π4 < θ < π2 ∪ π < θ < 5π

4

Solucao 134

J. Bartasevicius 161

Page 162: bartasevicius.com.brbartasevicius.com.br/sites/default/files/2018-12/... · Sobre este trabalho. E permitido: • Compartilhar - copiar e redistribuir o material em qualquer meio

CAPITULO 2. SOLUCOES

Fazendo z4 = X:

X2 − 17X + 16 = 0 =⇒

X = 16 ou

X = 1

z4 = 16 =⇒

z = ±2 ou

z = ±2i

z4 = 1 =⇒

z = ±1 ou

z = ±iCondicao:

z − |z| = 0

2− |2| = 0 =⇒ 2 e solucao− 2− | − 2| = −4 =⇒ −2 nao e solucao2i− |2i| = 2i− 2 =⇒ 2i nao e solucao− 2i− | − 2i| = −2i− 2 =⇒ −2i nao e solucao

1− |1| = 0 =⇒ 1 e solucao− 1− | − 1| = −2 =⇒ −1 nao e solucaoi− |i| = i− 1 =⇒ i nao e solucao− i− | − i| = −i− 1 =⇒ −i nao e solucao

∴ as solucoes sao 2 e 1, cuja soma e 3.Alternativa c).

Solucao 135

23√x+1 + 44

(2√x+1)

+ 64 = 19(

22√x+1)

=⇒

23√x+1 − 19

(22√x+1)

+ 44(

2√x+1)

+ 64 = 0

Fazendo 2√x+1 = z:

z3−19z2 +44z+64 = 0 =⇒ z3−4z2−15z2 +60z−16z+64 = 0 =⇒

z2(z−4)−15z(z−4)−16z(z−4) = 0 =⇒ (z − 4)(z2 − 15z − 16

)= 0

As solucoes para z sao:

z = 4

z = 16 e

z = −1Resolvendo x:

2√x+1 = 4 =⇒

√x+ 1 = 2 =⇒ x = 3

162 J. Bartasevicius

Page 163: bartasevicius.com.brbartasevicius.com.br/sites/default/files/2018-12/... · Sobre este trabalho. E permitido: • Compartilhar - copiar e redistribuir o material em qualquer meio

CAPITULO 2. SOLUCOES

2√x+1 = 16 =⇒

√x+ 1 = 4 =⇒ x = 15

2√x+1 = −1 =⇒ 6 ∃x

∴ soma das solucoes e 15 + 3 = 18.Alternativa d).

Solucao 136

De√a√b =

1

2, a condicao e a, b > 0.√a√b =

1

2=⇒ a2b =

1

16

ln(a2 + b

)+ ln 8 = ln 5 =⇒ a2 + b =

5

8

Da primeira equacao: a2 =1

16b.

Substituindo na segunda equacao:

1

16b+ b =

5

8=⇒ 1 + 16b2 = 10b =⇒

16b2 − 10b+ 1 = =⇒

b =

1

2ou

b =1

8

Solucoes para a, lembrando da condicao a > 0:b =

1

2=⇒ a2 =

2

16=⇒ a =

1

2√

2

b =1

8=⇒ a2 =

8

16=⇒ a =

1√2

Para os dois pares ordenados de solucao (a, b),a

bvale:

a

b=

12√

212

=1√2

ou

1√2

18

=8√2

= 4√

2

Alternativa a).

J. Bartasevicius 163

Page 164: bartasevicius.com.brbartasevicius.com.br/sites/default/files/2018-12/... · Sobre este trabalho. E permitido: • Compartilhar - copiar e redistribuir o material em qualquer meio

CAPITULO 2. SOLUCOES

Solucao 137Convertendo para coordenada polar:

1 +√

3i = 2π

3

1−√

3i = 2 −π3

Calculando z:

z =

(2 π/3

2 −π/3

)10

=

(1

3

)10

= 120π

3= 1

3

Transformando z para coordenadas retangulares:

z = 12π

3=⇒

Re(z) = 1 cos

3= −1

2

Im(z) = 1 sin2π

3=

√3

2

Calculo de 2 arcsin (Re(z)) + 5 arctan (2Im(z)):

2 arcsin

(−1

2

)+ 5 arctan

(2

√3

2

)= 2

(−π

6

)+ 5

(π3

)=

3

Alternativa d).

Solucao 138A divisao de p(x) por q(x) resulta em quociente v(x) e resto r(x).Portanto:

p(x) = q(x)v(x) + r(x) (A)

Fatorando q(x):

q(x) = x3 − 2x2 + x− 2 = x2(x− 2) + (x− 2) =(x2 + 1

)(x− 2)

∴ q(x) = (x+ i) (x− i) (x− 2) (B)

Todos os valores aj ∈ R, para 0 6 j 6 15. Isto significa que, se i e raizde p(x), entao seu conjugado tambem deve ser raiz de p(x) para quetodos os coeficientes sejam reais.Substituindo (B) em (A):

p(x) = (x+ i) (x− i) (x− 2)v(x) + r(x) (C)

164 J. Bartasevicius

Page 165: bartasevicius.com.brbartasevicius.com.br/sites/default/files/2018-12/... · Sobre este trabalho. E permitido: • Compartilhar - copiar e redistribuir o material em qualquer meio

CAPITULO 2. SOLUCOES

Aplicando os valores de x = i,−i, 2, tem-se os valores de p(x) iguais a0, 0 e 1, respectivamente.q(x) e um polinomio de 3º grau, portanto r(x) sera, no maximo, umpolinomio de 2º grau.Fazendo r(x) = ax2 + bx + c, pode-se montar o seguinte sistema,substituindo os valores de x e p(x) em (C):

p(i) = (i+ i) (i− i)︸ ︷︷ ︸0

(i− 2)v(i) + a(i)2 + b(i) + c

p(−i) = (−i+ i)︸ ︷︷ ︸0

(−i− i) (−i− 2)v(i) + a(−i)2 + b(−i) + c

p(2) = (2 + i) (2− i) (2− 2)︸ ︷︷ ︸0

v(i) + a(2)2 + b(2) + c

Evoluindo para:

0 = −a+ bi+ c

0 = −a− bi+ c

1 = 4a+ 2b+ c

=⇒

a =

1

5b = 0

c =1

5

∴ r(x) =1

5x2 + 0x+

1

5=

1

5x2 +

1

5.

Alternativa b).

Solucao 139a) Calculo de n:

sinπ

n= 2 sin

π

2ncos

π

2n= 2 sin

π

2n

√1− sin2 π

2n=⇒

sinπ

n= 2

√2−√

3

4

√1− 2−

√3

4=⇒

sinπ

n= 2

√2−√

3

4

√2 +√

3

4=⇒

sinπ

n= 2

√1

42=

1

2=⇒

J. Bartasevicius 165

Page 166: bartasevicius.com.brbartasevicius.com.br/sites/default/files/2018-12/... · Sobre este trabalho. E permitido: • Compartilhar - copiar e redistribuir o material em qualquer meio

CAPITULO 2. SOLUCOES

π

n=π

6=⇒ ∴ n = 6

b) Calculo de sinπ

24:

n = 6 =⇒ sinπ

12=

√2−√

3

4.

sinπ

12= 2 sin

π

24

√1− sin2 π

24=⇒

2−√

3

4= 4 sin2 π

24

(1− sin2 π

24

)=⇒

2−√

3

4= 4 sin2 π

24− 4 sin4 π

24=⇒

4 sin4 π

24− 4 sin2 π

24+

2−√

3

4= 0 =⇒

sin2 π

24=

4±√

16− 4(2−√

3)

8=

4±√

8 + 4√

3

8=⇒

sin2 π

24=

4± 2√

2 +√

3

8=

2±√

2 +√

3

4

Fazendo 2 +√

3 =(A+B

√3)2

:

A2 + 3B2 = 2

2AB = 1=⇒

B =

√2

2=⇒ A =

√2

2ou

B =

√6

6=⇒ A =

√6

2

=⇒

Assumindo A =

√2

2e B =

√2

2, tem-se que:

2 +√

3 =

(√2

2+

√2

2

√3

)2

=⇒

√2 +√

3 =

√2 +√

6

2

Voltando ao calculo de sinπ

24:

sin2 π

24=

(√2 +√

6

2

)4

=4±

(√2 +√

6)

8

166 J. Bartasevicius

Page 167: bartasevicius.com.brbartasevicius.com.br/sites/default/files/2018-12/... · Sobre este trabalho. E permitido: • Compartilhar - copiar e redistribuir o material em qualquer meio

CAPITULO 2. SOLUCOES

Como 0 < sinπ

24< sin

π

12, entao sin

π

24=

√4−

(√2 +√

6)

8:

sinπ

24=

√2(4−√

2−√

6)

16=

√8− 2

√2− 2

√6

4

Solucao 140Se a equacao representa duas retas concorrentes, entao ela pode serfatorada no produto de duas equacoes lineares, na forma:

(Arx+Bry + Cr)︸ ︷︷ ︸reta r

(Asx+Bsy + Cs)︸ ︷︷ ︸reta s

= 0

Para descobrir os valores de A, B e C, e necessario encontrar 4 pontosda equacao.Fazendo x = 0, tem-se os seguintes valores de y:

−2y2 + 8y − 6 = 0 =⇒ y2 − 4y + 3 = 0 =⇒

y = 1

y = 3

Fazendo y = 0, tem-se os seguintes valores de x:

3x2 − 3x− 6 = 0 =⇒ x2 − x− 2 = 0 =⇒

x = −1

x = 2

Assim, os 4 pontos da equacao sao: (0, 1), (0, 3), (−1, 0) e (2, 0).Sabes-se que as retas sao concorrentes e nao podem ser os proprios eixosdas ordenadas e abcissas. Restam duas possibilidades:

y y

x x

(I) (II)

3 3

1 1

2 2−1 −1

r

s

r

s

J. Bartasevicius 167

Page 168: bartasevicius.com.brbartasevicius.com.br/sites/default/files/2018-12/... · Sobre este trabalho. E permitido: • Compartilhar - copiar e redistribuir o material em qualquer meio

CAPITULO 2. SOLUCOES

Para a opcao (I), a equacao da reta r e x+ 1 = y.Se a divisao da equacao original por x − y + 1 fosse exata, ter-se-iafatorado a equacao. Mas o resto e 6xy:(

3x2 + 5xy − 2y2 − 3x+ 8y − 6)

= (x− y + 1) (3x+ 2y − 6) + 6xy

Para a opcao (II), a equacao da reta r e 3x+ 3 = y.A divisao da equacao original por 3x− y + 3 fornece a equacao da retas : x+ 2y − 2 = 0, numa divisao exata.Portanto, o grafico da opcao (II) e o que representa a equacao fornecidano problema.(

3x2 + 5xy − 2y2 − 3x+ 8y − 6)

= (3x− y + 3) (x+ 2y − 2)

Para a equacao r, y = 3x+ 3, cujo coeficiente angular e cr = 3.

Para a equacao s, y = 1− x

2, cujo coeficiente angular e cs = −1

2.

O calculo da tangente do angulo das retas concorrentes e tan θ =∣∣∣∣ cr − cs1 + cr.cs

∣∣∣∣:tan θ =

∣∣∣∣∣∣∣∣3−

(−1

2

)1 + 3

(−1

2

)∣∣∣∣∣∣∣∣

∴ tan θ = 7

Solucao 141Convertendo os logaritmos:

2 + 2

(log 10

log x2

)=

(1

− log x

)2

=⇒

2 +2

2 log x=

1

log2 x=⇒

2 +1

log x− 1

log2 x= 0 =⇒

Fazendo1

log x= y:

2 + y − y2 = 0 =⇒ y2 − y − 2 = 0

y = 2 ou

y = −1

168 J. Bartasevicius

Page 169: bartasevicius.com.brbartasevicius.com.br/sites/default/files/2018-12/... · Sobre este trabalho. E permitido: • Compartilhar - copiar e redistribuir o material em qualquer meio

CAPITULO 2. SOLUCOES

Para y = 2,1

log x= 2 =⇒ log x =

1

2.

Portanto, quando y = 2, x =√

10.

Para y = −1,1

log x= −1 =⇒ log x = −1.

Portanto, quando y = −1, x =1

10.

O produto de todas as raızes reais e:√

10× 1

10=

√10

10.

Alternativa c).

Solucao 142

3

√√√√ ex + 1(2764

)−(

34

)(x+1)> 0 =⇒

ex + 1(33

26

)−(

34

)(x+1)> 0

Analisando os fatores da fracao acima, tem-se que ex + 1 > 0∀x ∈ R.Entao: (

33

26

)−(

3

4

)(x+1)

> 0 =⇒(

3

22

)3

>

(3

4

)x+1

Como3

4< 1, entao a desigualdade e mantida se 3 < x+ 1 =⇒ x > 2.

∴, o menor numero inteiro e n = 3.

Calculando 3

√3√

3√

3√. . .:

3

√√√√√3

√3√

3√. . .︸ ︷︷ ︸

Z

= Z =⇒

3√Z = Z =⇒ Z2 − 9Z = 0

Como Z 6= 0, entao Z = 9.

logn 3

√3

√3√

3√. . . = log3 9 = 2

J. Bartasevicius 169

Page 170: bartasevicius.com.brbartasevicius.com.br/sites/default/files/2018-12/... · Sobre este trabalho. E permitido: • Compartilhar - copiar e redistribuir o material em qualquer meio

CAPITULO 2. SOLUCOES

Analise das alternativas para verificar qual possui 2 como raiz:

a) 23 − 22 − 9 = −5

b) 23 + 2− 1 = 9

c) 24 − 4.22 − 2 + 2 = 0 (raiz!)

d) 22 − 4.2 + 3 = −1

e) 24 − 4.22 + 2 + 1 = 3

Alternativa c).

Solucao 143Para o sistema ser impossıvel ou possıvel e indeterminado, a matrizcaracterıstica deve ser nula:∣∣∣∣∣∣

1 2 −33 −1 5

4 1 (a2 − 14)

∣∣∣∣∣∣ = 0 =⇒

16− a2 = 0 =⇒ a = ±4

Se a = 4, entao a terceira equacao e 4x + y + 2z = 6, que e a somadas primeira e segunda equacoes do sistema. Neste caso, o sistema epossıvel e indeterminado.Se a = −4, entao a terceira equacao e 4x+ y+ 2z = −2, contrariando asoma das primeira e segunda equacoes do sistema, que e 4x+y+ 2z = 6.Neste caso, o sistema e impossıvel.∴, o sistema e impossıvel quando a = −4 e e possıvel e indeterminadoquando a = 4.Alternativa b).

Solucao 144Primeiramente, deve-se fatorar x4 + 1 em dois polinomios de 2º grau:

x4 + 1 =(x2 − i

)(x2 + i

)Mas o enunciado limita os valores de ai, bi, ci ∈ R, portanto nao epossıvel utilizar esta fatoracao.Assim, x4 + 1 podera ser fatorado da seguinte forma:

x4 + 1 =(x2 + 1 +

√2x)(

x2 + 1−√

2x)

170 J. Bartasevicius

Page 171: bartasevicius.com.brbartasevicius.com.br/sites/default/files/2018-12/... · Sobre este trabalho. E permitido: • Compartilhar - copiar e redistribuir o material em qualquer meio

CAPITULO 2. SOLUCOES

Separando a soma:

x2

x4 + 1=

Ax+B

x2 +√

2x+ 1+

Cx+D

x2 −√

2x+ 1=⇒

x2 = Ax3 − A√

2x2 + Ax+Bx2 −B√

2x+B+

+ Cx3 + C√

2x2 + Cx+Dx2 +D√

2x+D =⇒

x2 = (A+ C)x3 +(−A√

2 + C√

2 +B +D)x2+

+(A+ C −B

√2 +D

√2)x+ (B +D) =⇒

A+ C = 0

−A√

2 + C√

2 +B +D︸ ︷︷ ︸=0

= 1

A+ C︸ ︷︷ ︸=0

−B√

2 +D√

2 = 0

B +D = 0

=⇒

A+ C = 0

−A+ C =

√2

2−B√

2 +D√

2 = 0

B +D = 0

Elevando ao quadrado as primeira e segunda equacoes e somando:A2 + 2AC + C2 = 0

A2 − 2AC + C2 =1

2

=⇒ 2A2 + 2C2 =1

2

∴ A2 + C2 =1

4.

Alternativa c).

Solucao 145Por ser menor que 1000, o numero possui 3 algarismos.Para um numero ser ımpar, o algarismo das unidades deve ser 1, 3, 5, 7ou 9.Os demais algarismos devem ser distintos:

• O algarismo das dezenas pode ser qualquer um, exceto o dasunidades. Portanto ha 9 possibilidades.

• O algarismo das centenas pode ser qualquer um, exceto o dasdezenas e das unidades. Portanto ha 8 possibilidades.

J. Bartasevicius 171

Page 172: bartasevicius.com.brbartasevicius.com.br/sites/default/files/2018-12/... · Sobre este trabalho. E permitido: • Compartilhar - copiar e redistribuir o material em qualquer meio

CAPITULO 2. SOLUCOES

Quantidade de numeros nestas condicoes: 5× 9× 8 = 360.Porem, todos os numeros foram considerados com 3 algarismos.Por este motivo, os numeros 1, 3, 5, 7 e 9 foram considerados como 001,003, 005, 007 e 009.Logo, eles foram excluıdos do calculo por possuırem o numero zerorepetido nas casas da dezena e da centena.Portanto, o numero total de numeros ımpares entre 1 e 999 com todosos algarismos distintos e:

360 + 5 = 365

Alternativa b).

Solucao 146

Se x1, x2 e x3 formam uma PG decrescente, entao x1 = x2r e x3 =x2

r,

com r > 1.O produto das raızes deve ser igual ao valor negativo da divisao dotermo independente pelo coeficiente do termo de maior expoente.

x1x2x3 = x2r.x2.x2

r= x3

2 = −−1

64=⇒ x2 =

1

4

A soma das raızes deve ser igual ao valor negativo da divisao do coefici-ente de x2 pelo coeficiente de x3.

r

4+

1

4+

1

4r= −−56

64=⇒

2r2 + 2r + 2 = 7r =⇒ 2r2 − 5r + 2 = 0 =⇒

r = 2 ou

r =1

2

Como a condicao definida para uma PG decrescente e r > 1, portantor = 2.

As raızes sao:

x1 =

1

2

x2 =1

4

x3 =1

8Calculo da expressao:

sin

[(1

2+

1

4

]+ tan

[(4

1

2

1

8

]=

172 J. Bartasevicius

Page 173: bartasevicius.com.brbartasevicius.com.br/sites/default/files/2018-12/... · Sobre este trabalho. E permitido: • Compartilhar - copiar e redistribuir o material em qualquer meio

CAPITULO 2. SOLUCOES

= sin3π

4+ tan

π

4=

√2

2+ 1 =

√2 + 2

2

Alternativa e).

Solucao 147

| 3x− 4 |6 2, x ∈ Z

3x− 4 = 0 =⇒ x =4

3

• Se x <4

3, entao | 3x− 4 |= 4− 3x:

4− 3x 6 2 =⇒ x >2

3

∴2

36 x <

4

3=⇒ x = 1 somente.

• Se x >4

3, entao | 3x− 4 |= 3x− 4:

3x− 4 6 2 =⇒ x 6 2

∴4

36 x 6 2 =⇒ x = 2 somente.

Portanto, c ∈ 1, 2.Para que a equacao tenha raızes reais, b2 − 4c > 0.Entao, b2 > 4c.

• Se c = 1:

b2 > 4 =⇒ b ∈ −4,−3,−2, 2, 3, 4, 5

• Se c = 2:

b2 > 8 =⇒ b ∈ −4,−3, 3, 4, 5

J. Bartasevicius 173

Page 174: bartasevicius.com.brbartasevicius.com.br/sites/default/files/2018-12/... · Sobre este trabalho. E permitido: • Compartilhar - copiar e redistribuir o material em qualquer meio

CAPITULO 2. SOLUCOES

O conjunto mais restritivo e para quando c = 2, pois os valores de bproduzem raızes reais mesmo quando c = 1.Desta forma, sao 5 valores de b para o universo de 10 possibilidades.

∴ P =5

10= 0.5.

Alternativa a).

Solucao 148

2x2 − (6 + p)x− c 6 0

A solucao da inequacao acima e [0, 2]. Sendo uma funcao de segundograu com concavidade para cima, tem-se que 0 e 2, que sao os limitesdo conjunto-solucao, sao as raızes da equacao:

2(x− 0)(x− 2) = 2x2 − (6 + p)x− c =⇒

2x2 − 4x = 2x2 − (6 + p)x− c

p = −2 e

c = 0

Resolvendo a equacao com exponencial:

22|x+1| = 16.2|x+1| − 64 =⇒ 22|x+1| − 16.2|x+1| + 64 = 0

Fazendo y = 2|x+1|:

y2 − 16.y + 64 = 0 =⇒ (y − 8)2 = 0 ∴ y = 8

Resolvendo x:

2|x+1| = 8 =⇒ |x+ 1| = 3

∴ x = 2 ∨ x = −4

O maior valor de x e 2 =⇒ q = 2.Entao, o numero complexo e −2 + i2.

• Modulo =√

(−2)2 + 22 = 2√

2.

• Argumento = cotg2

−2= cotg(−1) =

4.

174 J. Bartasevicius

Page 175: bartasevicius.com.brbartasevicius.com.br/sites/default/files/2018-12/... · Sobre este trabalho. E permitido: • Compartilhar - copiar e redistribuir o material em qualquer meio

CAPITULO 2. SOLUCOES

Portanto, a representacao trigonometrica e 2√

2

(cos

4+ i sin

4

).

Alternativa b).

Solucao 149

a0q3 = 5 =⇒ a0 =

5

q3(A)

log5 (a0a1 . . . a9) = 10− 15 log5 2 =⇒

=⇒ log5

(a10

0 q0+1+2+...+9)

= log5 510 − log5 215 =⇒

=⇒ log5

(a10

0 q45)

= log5

(510

215

)=⇒

=⇒ a100 q45 =

510

215=⇒ a2

0q9 =

52

23(B)

Substituindo (A) em (B):(5

q3

)2

q9 =52

23=⇒ 52q3 =

52

23=⇒ q =

1

2

Com q =1

2, de (A), a0 = 40.

Calculando a soma S:

S = 40 + 20 + 10 + . . . =⇒ S

2= 20 + 10 + 5 + . . . =⇒

=⇒ S

2= S − 40 =⇒ S = 80

Logo, log2 80 = log2

(24 × 5

).

∴ log2 S = 4 + log2 5.Alternativa c).

Solucao 150

4x2(3x− 1)− (3x− 1) = 0 =⇒

J. Bartasevicius 175

Page 176: bartasevicius.com.brbartasevicius.com.br/sites/default/files/2018-12/... · Sobre este trabalho. E permitido: • Compartilhar - copiar e redistribuir o material em qualquer meio

CAPITULO 2. SOLUCOES(4x2 − 1

)(3x− 1) = 0 =⇒

(2x− 1) (2x+ 1) (3x− 1) = 0 =⇒

a =

1

2

b = −1

2

c =1

3√(1

2

)3

+

(−1

2

)3

+

(1

3

)3

+ 1 =

√28

27=

2√

7

3√

3

Racionalizando o denominador:

∴2√

21

9

Alternativa a).

Solucao 151

f(81) = 3 + f(79) = 3 + 3 + f(77) = 3 + 3 + . . .+ f(1) =⇒

f(81) = 3.81− 1

2+ f(1) =⇒ f(81) = 3.40 + 5 = 125

f(70) = 3 + f(68) = 3 + 3 + f(66) = 3 + 3 + . . .+ f(0) =⇒

f(70) = 3.70

2+ f(0) =⇒ f(70) = 3.35 + 10 = 115

√f(81)− f(70) =

√125− 115 =

√10

Alternativa b).

Solucao 152Com y ∈ R, x > 0.Para uma reta ser perpendicular a 4x− 3y + 2 = 0, o produto de seuscoeficientes angulares deve ser −1.

Na reta dada, a equacao definida em x e y =4

3x+

2

3. Seu coeficiente

angular e4

3, implicando o coeficiente da reta perpendicular ser −3

4.

176 J. Bartasevicius

Page 177: bartasevicius.com.brbartasevicius.com.br/sites/default/files/2018-12/... · Sobre este trabalho. E permitido: • Compartilhar - copiar e redistribuir o material em qualquer meio

CAPITULO 2. SOLUCOES

Portnto, o ponto de tangencia a curva y2 = 2x3 deve produzir o valorda derivada igual ao coeficiente angular da reta perpendicular.Calculo da derivada da curva:

d(y2) = d(x3) =⇒ 2y.dy = 6x2.dx =⇒

dy

dx=

3x2

y

Como y2 = 2x3, y = ±√

2x3 = ±x√

2x. Devido a restricao do domınio,x > 0, implicando x

√2x > 0. Como o valor do coeficiente angular e

negativo, resta utilizar y = −x√

2x:

dy

dx=

3x2

−x√

2x=−3√

2x

2

dy

dx= −3

4, entao

−3√

2x

2= −3

4=⇒ x =

1

8.

x =1

8=⇒ y2 =

2

83:

y = ±√

1

256= ± 1

16

Como foi utilizado o valor de y negativo, devera ser considerado apenaso ponto onde y < 0.

∴ o ponto e

(1

8,− 1

16

).

Alternativa a).

Solucao 153Calculando o domınio do sistema: x > 0 e y 6= 0.Linearizando as equacoes com logaritmo:

y log x = −2 log y =⇒ log y =y log x

−2

x log y = −1

2log x

Substituindo log y da primeira equacao na segunda:

x.y log x

−2= −1

2log x =⇒ xy = 1

J. Bartasevicius 177

Page 178: bartasevicius.com.brbartasevicius.com.br/sites/default/files/2018-12/... · Sobre este trabalho. E permitido: • Compartilhar - copiar e redistribuir o material em qualquer meio

CAPITULO 2. SOLUCOES

y =1

xSubstituindo na primeira equacao original:

x(1/x) =1(1

x

)2=⇒ x( 1

x−2) = 1

Da equacao acima, x = 1 ou1

x− 2 = 0 =⇒ x =

1

2.

Se x = 1, entao y = 1.

Se x =1

2, entao y = 2.

∴ 1 + 1 +1

2+ 2 =

9

2

Alternativa b).

Solucao 154Potencias de i:

i0 = 1

i1 = i

i2 = −1

i3 = −ii4 = 1

=⇒

i0 = i4a, a ∈ Ni1 = i4a+1, a ∈ Ni2 = i4a+2, a ∈ Ni3 = i4a+3, a ∈ N

Facilitando o calculo de z:i8n−5 = i8n−5+8 = i8n+3 = i3 = −ii4n−8 = i8n−8+8 = i8n = 1

Calculando z:

z = (1− i)3 + 2i = 1− 3i+ 3(−1)− (−i) + 2i = −2

Calculo de P (−2):

P (−2) = −2(−2)3 + (−2)2 − 5(−2) + 11 = +16 + 4 + 10 + 11 = 41

Alternativa b).

178 J. Bartasevicius

Page 179: bartasevicius.com.brbartasevicius.com.br/sites/default/files/2018-12/... · Sobre este trabalho. E permitido: • Compartilhar - copiar e redistribuir o material em qualquer meio

CAPITULO 2. SOLUCOES

Solucao 155

Considerar os tres numeros em PG: nq, n en

q.

nq + n+

n

q= 13 =⇒ n

(q + 1 +

1

q

)= 13

(nq)2 + (n)2 +

(n

q

)2

= 91 =⇒ n2

(q2 + 1 +

1

q2

)= 91

n

q

(q2 + q + 1

)= 13

n2

q2

(q4 + q2 + 1

)= 13× 7

n2

q2

(q4 + q2 + 1

)= 7× n

q

(q2 + q + 1

)(× qn

(q2 + q + 1

))

n

q

(q2 + q + 1

)︸ ︷︷ ︸

13

(q4 + q2 + 1

)= 7

(q2 + q + 1

)2=⇒

13(q4 + q2 + 1

)= 7

(q2 + q + 1

)2=⇒ (A)

13(q4 + q2 + 1

)= 7

(q4 + q2 + 1 + 2

(q3 + q2 + q

))=⇒

6(q4 + q2 + 1

)= 14q

(q2 + q + 1

)=⇒

3(q4 + q2 + 1

)= 7q

(q2 + q + 1

)(B)

Dividindo (A) por (B):

13

3=q2 + q + 1

q=⇒ 3q2 + 3q + 3 = 13q =⇒

3q2 − 10q + 3 = 0 =⇒

q = 3 ou

q = 13

Nao ha diferenca entre os valores de q, pois ele aparecem na soma comos termos invertidos.Substituindo q = 3:

n

(3 + 1 +

1

3

)= 13 =⇒ n× 13

3= 13

J. Bartasevicius 179

Page 180: bartasevicius.com.brbartasevicius.com.br/sites/default/files/2018-12/... · Sobre este trabalho. E permitido: • Compartilhar - copiar e redistribuir o material em qualquer meio

CAPITULO 2. SOLUCOES

∴ n = 3

Finalmente, calculando o numero de comissoes:

C28,3 =28!

3!25!= 3276

Alternativa c).

Solucao 156 √√√√√√3

√√√√√5

√3

√5√

3 . . .︸ ︷︷ ︸r

= r =⇒

√3√

5r = r =⇒

3√

5r = r2 =⇒ 5r =r4

9=⇒ r3 = 45

Volume da esfera:4πr3

3=

4π45

3= 60π cm3.

1 dm3 = 1000 cm3. Portanto a solucao e 60.10−3π dm3.Alternativa c).

Solucao 157Sequencia em PA:

b− a = 2− b =⇒ a− 2b = −2 (A)

Sequencia em PG:a

b=

2

a=⇒ a2 = 2b (B)

Substituindo (B) em (A):

a− a2 = −2 =⇒ a2 − a− 2 = 0 =⇒

a = 2 ou

a = −1

Como o enunciado diz que a sequencia nao e constante, a = 2 nao esolucao.

180 J. Bartasevicius

Page 181: bartasevicius.com.brbartasevicius.com.br/sites/default/files/2018-12/... · Sobre este trabalho. E permitido: • Compartilhar - copiar e redistribuir o material em qualquer meio

CAPITULO 2. SOLUCOES

Se a = 1, b =1

2.

A curva y2 − 2y + x + 3 = 0 e uma parabola com eixo paralelo asabscissas. Orientando a parabola, tem-se: x = −y2 + 2y − 3.As coordenadas do vertice da parabola sao:

y =−2

2(−1)= 1

Logo x = −12 + 2.1− 3 = −2.

A reta y = cx+ d deve passar pelos pontos

(−1,

1

2

)e (−2, 1):

1

2= c(−1) + d

1 = c(−2) + d=⇒

c = −1

2d = 0

∴ a equacao e y = −1

2x, ou x+ 2y = 0.

Alternativa d).

Solucao 158Analise da inequacao |x|+ |y| < 1:

• x > 0 ∧ y > 0 (solucoes no primeiro quadrante):

x+ y < 1

1

1x

y

• x < 0 ∧ y > 0 (solucoes no segundo quadrante):

−x+ y < 1

J. Bartasevicius 181

Page 182: bartasevicius.com.brbartasevicius.com.br/sites/default/files/2018-12/... · Sobre este trabalho. E permitido: • Compartilhar - copiar e redistribuir o material em qualquer meio

CAPITULO 2. SOLUCOES

−1

1x

y

• x < 0 ∧ y < 0 (solucoes no terceiro quadrante):

−x− y < 1

−1

−1x

y

• x > 0 ∧ y < 0 (solucoes no quarto quadrante):

x− y < 1

−1−1x

y

Portanto, o evento A e a juncao dos quatro graficos acima:

1

1

x

y

−1−1

182 J. Bartasevicius

Page 183: bartasevicius.com.brbartasevicius.com.br/sites/default/files/2018-12/... · Sobre este trabalho. E permitido: • Compartilhar - copiar e redistribuir o material em qualquer meio

CAPITULO 2. SOLUCOES

Desenhando o espaco amostral Ω com o evento A:

1

1

x

y

−1−1

O evento A e um quadrado de lado√

2, portanto sua ocorrencia e(√2)2

= 2.O espaco amostral e um cırculo de raio 2, cuja area e 4π.

∴ Pevento A =2

4π=

1

Alternativa d).

Solucao 159

[(x− 1)(5x− 7)

3

]! = 1 =⇒

(x− 1)(5x− 7)

3= 0 ou

(x− 1)(5x− 7)

3= 1

•(x− 1)(5x− 7)

3= 0:x− 1 = 0 =⇒ x = 1 ou

5x− 7 = 0 =⇒ x =7

5

•(x− 1)(5x− 7)

3= 1:

5x2 − 5x− 7x+ 7 = 3 =⇒

5x2 − 12x+ 4 = 0 =⇒

x = 2 ou

x =2

5

J. Bartasevicius 183

Page 184: bartasevicius.com.brbartasevicius.com.br/sites/default/files/2018-12/... · Sobre este trabalho. E permitido: • Compartilhar - copiar e redistribuir o material em qualquer meio

CAPITULO 2. SOLUCOES

A menor raiz inteira e m = 1.O termo medio de

(√y − z3

)12 e a potencia 6 para cada membro:

C12,6 (√y)6(−z3

)6=

12!

6!6!y3z18

Alternativa e).

Solucao 160

f(2 + 3√

2) = f(2)f(3√

2)

Calculo de f(2):

f(2) = f(1 + 1) = f(1)f(1) = 3× 3 = 9

Calculo de f(3√

2):

f(3√

2) = f(√

2 + 2√

2) = f(√

2)f(2√

2) =

f(√

2)f(√

2 +√

2) = f(√

2)f(√

2)f(√

2) = 2× 2× 2 = 8

Por fim:∴ f(2 + 3

√2) = f(2)f(3

√2) = 9× 8 = 72

Alternativa b).

Solucao 161Seja p o numero de pessoas, m o numero de mulheres, e h o numero dehomens:

p < 150, p ∈ Nm

h=

2

5=⇒ m =

2h

5p = m+ h =⇒ m+ h < 150

2h

5+ h < 150 =⇒ 7h

5< 150 =⇒ h <

750

7O valor maximo inteiro de h e 107.Mas para que o valor de m seja inteiro, h deve ser multiplo de 5.Entao, h = 105 =⇒ m = 42.Portanto, o numero de homens excede em 105− 42 = 63 o numero demulheres.Alternativa e).

184 J. Bartasevicius

Page 185: bartasevicius.com.brbartasevicius.com.br/sites/default/files/2018-12/... · Sobre este trabalho. E permitido: • Compartilhar - copiar e redistribuir o material em qualquer meio

CAPITULO 2. SOLUCOES

Solucao 162Fatorando G(x):

G(x) = 2(x2 + x− 6) = 2(x+ 3)(x− 2)

Para queF (x)

G(x)seja uma divisao exata, F (x) deve possuir os mesmos

fatores de G(x).Sendo F (x) uma equacao de 3º grau, ela deve possuir tres fatores deprimeiro grau:

F (x) = (x+ 3)(x− 2)(x+N) = x3 + x2(1 +N) + x(N − 6)− 6N

Comparando com F (x) do enunciado:

x3 + x2(1 +N) + x(N − 6)− 6N = x3 + ax+ b

Do termo de 2º grau tem-se 1 +N = 0 =⇒ N = −1.

x3 + x2(0) + x(−7)− 6(−1) = x3 + ax+ b

De onde se extrai a = −7 e b = +6.

∴ a+ b = −7 + 6 = −1

Alternativa b).

Solucao 163

Calculo das potencias do binomio(2x−1 + x3

)7para obter o termo x5:(

2x−1)a (

x3)7−a

= x5 =⇒ −1a+ 3(7− a) = 5 =⇒ a = 4

Calculo do termo:

C7,4

(2x−1

)4 (x3)3

= 560x5

Alternativa e).

J. Bartasevicius 185

Page 186: bartasevicius.com.brbartasevicius.com.br/sites/default/files/2018-12/... · Sobre este trabalho. E permitido: • Compartilhar - copiar e redistribuir o material em qualquer meio

CAPITULO 2. SOLUCOES

Solucao 164

(1 + i) =√

4=⇒ (1 + i)n = 2

n2

4Para que (1 + i)n seja um numero real,

4deve ser um multiplo de π.

O menor n paranπ

4ser multiplo de π e 4.

Alternativa c).

Solucao 165Sao:

• 2 livros de Calculo

• 3 livros de Historia

• 4 livros de Eletricidade

Os dois livros de Calculo podem se dispor em 2! possibilidades.Os tres livros de Historia podem se dispor em 3! possibilidades.Os quatro livros de Eletricidade podem se dispor em 4! possibilidades.As materias de Calculo, Historia e Eletricidade podem se dispor em 3!possibilidades.∴, o numero total de arranjos na prateleira e:

2!× 3!× 4!× 3! = 2× 6× 24× 6 = 1728

Alternativa a).

Solucao 166Domınio do numerador:

−1 6 cos a 6 1 =⇒ −1 6 log( x

10

)6 1 =⇒

10−1 6x

106 101 =⇒

1 6 x 6 100 (A)

Domınio do denominador:√4x− x3 6= 0 ∧ 4x− x3 > 0 =⇒ 4x− x3 > 0 =⇒

x(

4− x2)> 0 (B)

Analise de sinais de (B):

186 J. Bartasevicius

Page 187: bartasevicius.com.brbartasevicius.com.br/sites/default/files/2018-12/... · Sobre este trabalho. E permitido: • Compartilhar - copiar e redistribuir o material em qualquer meio

CAPITULO 2. SOLUCOES

• x:

x0

− +

• 4− x2:

4− x2−2 2

− + −

• x(4− x2

):

x0

4− x2−2 2

x(4− x2

)0−2 2

− − + +

− + + −

+ − + −

Fazendo (A) ∩ (B):

(A)

(B)

(A)∩(B)

1 100

−2 0 2

1 2

∴ S = x ∈ R | 1 6 x < 2.Alternativa d).

Solucao 167

xn =1 + 2 + 3 + . . .+ n

1 + 2 + 4 + . . .+ 2n−1=⇒

xn =

(n+ 1)n

22n − 1

=⇒

J. Bartasevicius 187

Page 188: bartasevicius.com.brbartasevicius.com.br/sites/default/files/2018-12/... · Sobre este trabalho. E permitido: • Compartilhar - copiar e redistribuir o material em qualquer meio

CAPITULO 2. SOLUCOES

xn =(n+ 1)n

2 (2n − 1)

Alternativa e).

Solucao 168Seja p cada real aumentado no valor do quilo.Para cada real aumentado, diminui 8 clientes que consomem 500g cada.Ou seja, para cada real aumentado, sao 4 quilos a menos vendido.Transformando em expressoes em funcao de p:

• Preco por quilo: 40 + p

• Numero de quilos consumidos: 200− 4p

A receita do restaurante e o valor do preco do quilo pelo peso vendido:(40 + p)× (200− 4p):

(40 + p)× (200− 4p) = 8000 + 40p− 4p2

Por se tratar de uma equacao de 2º grau com concavidade para baixo,ela possui valor maximo.Seu valor maximo e quando:

p = − b

2a= − 40

2(−4)= 5

Portanto, o preco do quilo que produzira a maior receita e 40 + p = 45.Alternativa d).

Solucao 169Convertendo z para coordenadas polares:

|z| =

√√√√(1

2

)2

+

(√3

2

)2

= 1

ϕ = arctan

√3/2

1/2= arctan

√3 =

π

3

188 J. Bartasevicius

Page 189: bartasevicius.com.brbartasevicius.com.br/sites/default/files/2018-12/... · Sobre este trabalho. E permitido: • Compartilhar - copiar e redistribuir o material em qualquer meio

CAPITULO 2. SOLUCOES

Calculando z.z2.z3 . . . zn:

z.z2.z3 . . . zn = z1+2+3+...+n = z

n(n+ 1)

2

Substituindo z pela representacao em coordenada polar:

z

n(n+ 1)

2 =(

3

)n(n+ 1)

2 = 1n(n+ 1)π

6

Para que z seja um numero real positivo, o argumenton(n+ 1)π

6deve

ser multiplo de 2π, ou seja,n(n+ 1)

6deve ser um multiplo de 2:

n(n+ 1)

6= 2k, k ∈ N =⇒

n(n+ 1) = 12k, k ∈ N =⇒ n = 3

Alternativa c).

Solucao 170De acordo com o enunciado, como ninguem esperara alem das 13 horas,o encontro devera ocorrer entre 12 e 13 horas.Cada uma das pessoas chega aleatoriamente entre 12 e 13 horas. Cha-mando de tx e ty o tempo de cada uma das pessoas envolvidas, o espacoamostral pode ser definido como um quadrado de 12 as 13 horas, ondecada eixo representa o horario de chegada de cada uma das pessoas aoponto de encontro:

ty

tx12 13

13

espacoamostral

J. Bartasevicius 189

Page 190: bartasevicius.com.brbartasevicius.com.br/sites/default/files/2018-12/... · Sobre este trabalho. E permitido: • Compartilhar - copiar e redistribuir o material em qualquer meio

CAPITULO 2. SOLUCOES

A area do espaco amostral e 1.Cada um espera pelo outro pelo perıodo maximo de 15 minutos, naoimportando quem chega primeiro. Esta afirmacao pode ser traduzidapara a seguinte inequacao:

|tx − ty| 61

4

Neste caso, ha duas hipoteses:

• Se tx > ty ou tx − ty > 0:

tx − ty 61

4=⇒ ty > tx −

1

4

Para estas condicoes, desenham-se os seguintes graficos:

ty

Grafico A

tx

tx = ty

tx > ty

12 13

13

ty

1/4

3/4Grafico B

tx

ty = tx −1

4

ty > tx −1

412 13

13

(Grafico A) ∩ (Grafico B) ∩ Espaco Amostral:

190 J. Bartasevicius

Page 191: bartasevicius.com.brbartasevicius.com.br/sites/default/files/2018-12/... · Sobre este trabalho. E permitido: • Compartilhar - copiar e redistribuir o material em qualquer meio

CAPITULO 2. SOLUCOES

1/4

3/4

ty

tx

ty = tx −1

4

ty = tx

Probabilidade de realizacaodo encontro quando tx > ty

12 13

13

Desta forma, a area do encontro quando tx > ty e:

Ax>y =1

2−

(3

4

)2

2=

7

32

• Se tx < ty ou tx − ty < 0:

ty − tx 61

4

Analogamente, a area do encontro quando tx < ty, sera a mesma,porem acima da reta tx = ty.

1/4

3/4

ty

tx

ty = tx +1

4

ty = tx

Probabilidade de realizacaodo encontro quando tx < ty

12 13

13

Entao, Ax<y =7

32

A area total do encontro e Atotal = Ax>y + Ax<y =7

16.

J. Bartasevicius 191

Page 192: bartasevicius.com.brbartasevicius.com.br/sites/default/files/2018-12/... · Sobre este trabalho. E permitido: • Compartilhar - copiar e redistribuir o material em qualquer meio

CAPITULO 2. SOLUCOES

1/4

3/4

ty = tx −1

4

1/4

3/4

ty

tx

ty = tx +1

4 ty = tx

Probabilidade total

12 13

13

Calculo da probabilidade:

P =Atotal

Espaco Amostral=

7

161

=7

16

Alternativa c).

Solucao 171

As expressoes sao da forma f(n) =n

(n− 1)2 + n.

Do enunciado:

x = f(1, 00000000002)

y = f(1, 00000000004)

z = f(19, 00000000002)

Analise da variacao da expressao f(n):

f ′(n) =n(2n− 1)− 1(n2 − n+ 1)(

n2 − n+ 1)2 =

n2 − 1(n2 − n+ 1

)2f ′(n) < 0 se −1 < n < 1. Quando f ′(n) < 0, a funcao e decrescente.n > 1 =⇒ f ′(n) > 0. Quando f ′(n) > 0, a funcao e crescente.Todos os valores sao para n > 1, portanto a funcao e crescente.Logo, z > y > x.Alternativa e).

192 J. Bartasevicius

Page 193: bartasevicius.com.brbartasevicius.com.br/sites/default/files/2018-12/... · Sobre este trabalho. E permitido: • Compartilhar - copiar e redistribuir o material em qualquer meio

CAPITULO 2. SOLUCOES

Solucao 172x+ y = 2(x+ y)2 = 4 =⇒ x2 + 2xy + y2 = 4 =⇒ x2 + y2 = 4− 2xy

(x+ y)3 = 8 =⇒ x3 + 3xy (x+ y)︸ ︷︷ ︸2

+y3 = 8 =⇒ x3 + y3 = 8− 6xy

x2 + y2

x3 + y3= 4 =⇒ 4− 2xy

8− 6xy= 4

Fazendo xy = z:

4− 2z

8− 6z= 4 =⇒ 2− z = 16− 12z

∴ z = xy =14

11

Alternativa c).

Solucao 173N =z y x z y x z y x

N = (100z+10y+x)×106+(100z+10y+x)×103+(100z+10y+x) =⇒

N = (100z+10y+x)×(

106 + 103 + 1)

= (100z+10y+x)×(1001001)

1001001 = 333667× 3

N = (100z + 10y + x)× 333667× 3

∴ N e multiplo de 333667.Alternativa d).

Solucao 174

b =1

a

a+ b = 2 =⇒ a+1

a= 2 =⇒

J. Bartasevicius 193

Page 194: bartasevicius.com.brbartasevicius.com.br/sites/default/files/2018-12/... · Sobre este trabalho. E permitido: • Compartilhar - copiar e redistribuir o material em qualquer meio

CAPITULO 2. SOLUCOES

a2 − 2a+ 1 = 0 =⇒ a = 1 =⇒ b =1

a=⇒ b = 1

Calculo de(a3 + b3

) (a4 − b4

):(

13 + 13)(

14 − 14)

= 0

Alternativa e).

Solucao 175

2

x− 1+

3

x+ 1= 1 =⇒ 2x+ 2 + 3x− 3 = x2 − 1 =⇒

x5 − 5x = 0 =⇒

x = 0 ou

x = 5

∴ 02 + 52 = 25.Alternativa d).

Solucao 176 4x2 − 4x+ 1 = (2x− 1)2

−1 + 6x− 12x2 + 8x3 = (2x− 1)3

√(2x− 1)2 =

3√

(2x− 1)3 =⇒ |2x− 1| = 2x− 1

A equacao acima possui resposta somente se 2x− 1 > 0.

∴ S = x ∈ R | x > 1

2.

Alternativa d).

194 J. Bartasevicius

Page 195: bartasevicius.com.brbartasevicius.com.br/sites/default/files/2018-12/... · Sobre este trabalho. E permitido: • Compartilhar - copiar e redistribuir o material em qualquer meio

CAPITULO 2. SOLUCOES

Solucao 177

Aposta x:

Ganha - fica 1.25x

Perde - fica com 0.25x

Ganhou duas apostas e perdeu duas apostas, sempre apostando o valortotal. Calculo do montante final:

(1.25)2 (0.25)2 x =25

256x ≈ 10%x

Alternativa e).

Solucao 178 (3 + 2

√2)2008(

5√

2 + 7)1338

=

(3 + 2

√2)2008 (

5√

2− 7)1338(

5√

2 + 7)1338 (

5√

2− 7)1338

=

=

(3 + 2

√2)2008(

5√

2 + 7)1338

=

(3 + 2

√2)2008 (

5√

2− 7)1338[(

5√

2 + 7) (

5√

2− 7)]1338

=

=

(3 + 2

√2)2008 (

5√

2− 7)1338

[1]1338=(

3 + 2√

2)2008 (

5√

2− 7)1338

=

=[(

3 + 2√

2)(

5√

2− 7)]1338 (

3 + 2√

2)670

=

=[−1 +

√2]1338 (

3 + 2√

2)670

=

=[(−1 +

√2)(

3 + 2√

2)]670 (

−1 +√

2)668

=

=[1 +√

2]670 (

−1 +√

2)668

=

=[(

1 +√

2)(−1 +

√2)]668 (

−1 +√

2)2

=

= (2− 1)668(

3 + 2√

2)

= 3 + 2√

2

J. Bartasevicius 195

Page 196: bartasevicius.com.brbartasevicius.com.br/sites/default/files/2018-12/... · Sobre este trabalho. E permitido: • Compartilhar - copiar e redistribuir o material em qualquer meio

CAPITULO 2. SOLUCOES

Calculado

(3 + 2

√2)2008(

5√

2 + 7)1338︸ ︷︷ ︸

3+2√

2

+3− 2√

2:

3 + 2√

2 + 3− 2√

2 = 6

Alternativa d).

Solucao 179ax2 + bx+ c =⇒ raızes x1 e x2, com x1 < x2.

√mx2 = x1 =⇒ mx2 = x2

1 =⇒ m =x2

1

x2

x2 =√nx1 =⇒ nx1 = x2

2 =⇒ n =x2

2

x1

m+ n =x2

1

x2+x2

2

x1=x3

1 + x32

x1x2

x31 + x3

2

x1x2=

(x1 + x2)3 − 3x1x2 (x1 + x2)

x1x2x1 + x2 = − b

2ax1x2 =

c

a

m+n =(x1 + x2)3 − 3x1x2 (x1 + x2)

x1x2=

(− b

2a

)3

− 3( ca

)(− b

2a

)c

a

=

=3abc− b3

a2c

Alternativa a).

Solucao 180Calculo do domınio:

3x+ 1 6= 0 =⇒ x 6= −1

3

196 J. Bartasevicius

Page 197: bartasevicius.com.brbartasevicius.com.br/sites/default/files/2018-12/... · Sobre este trabalho. E permitido: • Compartilhar - copiar e redistribuir o material em qualquer meio

CAPITULO 2. SOLUCOES

Analise da expressao:(2x− 1)10 > 0 ∧ (3x+ 1)8 > 0,∀x.

Entao, (x− 5)15 definira o sinal da expressao. Como o expoente eımpar, so e necessario analisar x− 5. Portanto, se x > 5, a expressao emaior ou igual a zero.Contudo, se a parcela (2x− 1)10 for igual a zero, a expressao se torna

igual a zero. Neste caso, x =1

2tambem e solucao.

∴ S = x ∈ R | x =1

2∪ x > 5

Alternativa e).

Solucao 181Para ser possıvel calcular o valor da expressao, e necessario que 103x−x2 − 300 > 0:

−x2 + 103x− 300 = 0 =⇒

x = 3

x = 100

Os valores de x que satisfazem a inequacao sao 3 6 x 6 100.Portanto, a quantidade de numeros que satisfazem a inequacao e 100−3 + 1 = 98.Alternativa c).

Solucao 182

I - Verdadeiro.Se n e um numero primo, sua forma fatorada e 1× n.

II - Verdadeiro.24 = 23 × 3 e mdc(24, a) = 1,Logo, a nao pode possuir fatores 2 ou 3.∴ a ∈ 1, 5, 7, 11, 13, 17, 19, 23

III - Verdadeiro.

a2 + 2ab+ b2 = a2 + 2ac+ c2 =⇒ 2a(b− c) = (c− b)(c+ b)

J. Bartasevicius 197

Page 198: bartasevicius.com.brbartasevicius.com.br/sites/default/files/2018-12/... · Sobre este trabalho. E permitido: • Compartilhar - copiar e redistribuir o material em qualquer meio

CAPITULO 2. SOLUCOES

A expressao acima e valida, somente se c+ b = −2a ou c− b = 0.Sendo a, b, c ∈ N∗, nao e possıvel existir solucao para c+ b = −2a.Neste caso, a igualdade e valida se, e somente se c− b = 0, ou seja,se c = b.

IV - Verdadeiro.

a < b (×c)Sendo a, b, c ∈ N∗, logo e possıvel afirmar a.c < b.c.

Alternativa e).

Solucao 183

11y2 − 3 = 5(

175308− x2)

Para haver solucao, 11y2 − 3 deve ser multiplo de 5.Logo, 11y2 ≡ 3 (mod 5).Os restos de y2 dividido por 5 sao: 0, 1, 4.Consequentemente, os restos de 11y2 sao: 0, 1, 4.∴ 6 ∃y ∈ N | 11y2 ≡ 3 (mod 5).Alternativa a).

Solucao 184

4√

(2x− 1)4 = |2x− 1|A equacao pode ser simplificada para:

|2x− 1| = 3x+ 2 =⇒

2x− 1 = 3x+ 2, se x >

1

2

1− 2x = 3x+ 2, se x <1

2

Se x >1

2, x = −3 (impossıvel!).

Se x <1

2, x = −1

5.

Portanto, a solucao x = −1

5e unica.

Alternativa b).

198 J. Bartasevicius

Page 199: bartasevicius.com.brbartasevicius.com.br/sites/default/files/2018-12/... · Sobre este trabalho. E permitido: • Compartilhar - copiar e redistribuir o material em qualquer meio

CAPITULO 2. SOLUCOES

Solucao 185Da primeira equacao:

x =y√3

Substituindo na segunda equacao:(y√3

)2

y2=

1

3=⇒ 1

3=

1

3, ∀(x, y) ∈ R2 | y 6= 0

Portanto, o sistema e possıvel e indeterminado, onde x =y√3

, com

y 6= 0.Neste caso, somente x ou y podem ser inteiros, sendo impossıvel haversolucoes inteiras para ambos.Alternativa a).

Solucao 186

375y2x− 125y3 − 375yx2 +125x3 = 125b =⇒

=⇒ x3 − 3x2y + 3xy2 − y3 = b

y2 + x2 + 2yx = a2 =⇒ (x+ y)2 = a2

x3 − 3x2y + 3xy2 − y3 = b =⇒ (x− y)3 = b =⇒ (x− y)6 = b2 (A)

(x+ y)2 = a2 =⇒ (x+ y)6 = a6 (B)

Fazendo (A) × (B):

(x− y)6 × (x+ y)6 = a6b2 =⇒ [(x− y)× (x+ y)]6 = a6b2

∴(x2 − y2

)6= a6b2

Alternativa c).

J. Bartasevicius 199

Page 200: bartasevicius.com.brbartasevicius.com.br/sites/default/files/2018-12/... · Sobre este trabalho. E permitido: • Compartilhar - copiar e redistribuir o material em qualquer meio

CAPITULO 2. SOLUCOES

Solucao 187

3x3 − 5 =√

33x3 + 97 =⇒ 9x6 − 30x3 + 25 = 33x3 + 97 =⇒

9x6 − 63x3 − 72 = 0 =⇒ x6 − 7x3 − 8 = 0 =⇒(x3 − 8

)(x3 + 1

)= 0 =⇒

x3 = 8 =⇒ x = 2

x3 = −1 =⇒ x = −1

A expressao foi elevada ao quadrado, portanto e necessario testar asraızes, pois elas podem ter sido produzidas para valores positivos enegativos da expressao:

• x = 2:

3(2)3 −√

33(2)3 + 97 = 24−√

361 = 24− 19 = 5

Portanto, x = 2 e raiz.

• x = −1:

3(−1)3 −√

33(−1)3 + 97 = −3−√

64 = −3− 8 = −11

Portanto, x = −1 nao e raiz.

Portanto, ha somente uma raiz, x = 2.Alternativa a).

Solucao 188Transformando todos os numeros fracionarios em fracoes, e fatorandoos numeros inteiros:√√√√√

32 × 1

3+

7

√22 ×

√1

16−

(3 +

4

9

)+

(4 +

5

9

)3√

26=

=

√3× 1

3+

7√

22 ×√

2−4 −7 +

9

922

=

=

√1 +

7√

22 × 2−2 − 8

4=

200 J. Bartasevicius

Page 201: bartasevicius.com.brbartasevicius.com.br/sites/default/files/2018-12/... · Sobre este trabalho. E permitido: • Compartilhar - copiar e redistribuir o material em qualquer meio

CAPITULO 2. SOLUCOES

=√

1 + 1− 2 =√

2− 2

Alternativa d).

Solucao 189

A solucao parte da hipotese de que3√

26− 15√

3 e um numero da formaA+B

√3, A,B ∈ N.

26− 15√

3 =(A−B

√3)3

=⇒

26− 15√

3 = A3 − 3A2B√

3 + 3A3B − 3√

3B3 =⇒26 = A3 + 9AB

−15√

3 = −3A2B√

3− 3√

3B3 =⇒ 5 = A2B +B3

Na segunda equacao, B(A2 +B2

)= 5. Por hipotese, A,B ∈ N. Entao

B = 1 e A = 2.Testando os valores na primeira equacao: 23 + 9× 2× 1 = 26, confirmaa solucao para A e B.

Logo,3√

26− 15√

3 = 2−√

3 =

√(2−√

3)2

=√

7− 4√

3.

Alternativa b).

Solucao 190Convertendo 21 para a base 7:

(21)10 = (30)7 = (3× 10)7

Fazendo a exponenciacao:

(21)201210 = (3× 10)2012

7 =(

32012 × 102012)

7

Somente o fator 107 produz algarismos zeros na exponenciacao.Logo, (21)2012

10 terminara com 2012 zeros na base 7.Alternativa a).

J. Bartasevicius 201

Page 202: bartasevicius.com.brbartasevicius.com.br/sites/default/files/2018-12/... · Sobre este trabalho. E permitido: • Compartilhar - copiar e redistribuir o material em qualquer meio

CAPITULO 2. SOLUCOES

Solucao 191Simplificando a expressao:

2160x+ 1680y = 240(9x+ 7y)

O menor valor positivo de 9x + 7y para x, y ∈ Z e quando x = −3 ey = 4 =⇒ 9x+ 7y = 1.∴ o menor valor inteiro de 240(9x+ 7y) e 240.Alternativa d).

Solucao 192 10x = 100y =⇒ x = 10y

x+ y = 44=⇒

x = 40

y = 4

Analise das alternativas:

a) yx = 440 = 280

b)√xy =

√4× 44 = 4

√11

c) 10√yx =

10√

280 = 28 = 256

d) 4√xy =

4√

404 = 40

e)

√y

x=

√4

40=

√1

10=

√10

10

Alternativa c).

Solucao 193Simplificando A:

A =3 +√

6

5√

3− 2× 2√

2− 4√

2 + 5√

2=

=3 +√

6√3 +√

2=

(3 +√

6) (√

3−√

2)

3− 2=

= 3√

3 + 3√

2− 3√

2− 2√

3 =√

3

202 J. Bartasevicius

Page 203: bartasevicius.com.brbartasevicius.com.br/sites/default/files/2018-12/... · Sobre este trabalho. E permitido: • Compartilhar - copiar e redistribuir o material em qualquer meio

CAPITULO 2. SOLUCOES

Calculo deA2

6√A7

:

A2

6√A7

=6√A12

6√A7

=6√A5

Substituindo A:6

√(√3)5

=12√

35

Alternativa e).

Solucao 194

P =

(4

3

)(6

5

)(8

7

)(10

9

)(12

11

)Q =

(4

5

)(6

7

)(8

9

)(10

11

)P

Q=

(4

3

)(6

5

)(8

7

)(10

9

)(12

11

)×(

5

4

)(7

6

)(9

8

)(11

10

)P

Q=

12

3= 4

√P

Q=√

4 = 2

Alternativa b).

Solucao 195 [(313

)27+ 221 − 5

√239 +

3√

64−(

313

)27

] 7√92

=

=[39 + 22 − 5

√239 + 4− 39

] 7√92=[4− 5√

243] 7√92

=

= [4− 3]7√92 = 1

Alternativa c).

J. Bartasevicius 203

Page 204: bartasevicius.com.brbartasevicius.com.br/sites/default/files/2018-12/... · Sobre este trabalho. E permitido: • Compartilhar - copiar e redistribuir o material em qualquer meio

CAPITULO 2. SOLUCOES

Solucao 196Simplificando k:

k = 2(a+b)2−(a−b)2 = 24ab

Substituindo ab = 5, k = 220.Substituindo k em x2 − y2 = 5

√k:

(x− y) (x+ y) =5√

220 =⇒ (x− y) (x+ y) = 24

Com x, y ∈ N∗, existem as seguintes possibilidades:

• x− y = 1 ∧ x+ y = 16

x =17

2e y =

15

2.

Nao e solucao valida, pois x, y 6∈ N∗.• x− y = 2 ∧ x+ y = 8x = 5 e y = 3.Solucao valida, com x, y ∈ N∗.

• x− y = 4 ∧ x+ y = 4x = 4 e y = 0.Nao e solucao valida, pois y 6∈ N∗.

Com x = 5 e y = 3, 35 − 53 = 118.Alternativa e).

Solucao 197Pelo enunciado conclui-se que n2 + 37 = k(n+ 5), com n, k ∈ N.

∴ n2 − kn+ 37− 5k = 0 =⇒ n =k ±√k2 − 148 + 20k

2

Logo, a condicao necessaria (mas nao suficiente) para que exista n ∈ Ne que k2 + 20k − 148 seja um quadrado perfeito.

k2 + 20k − 148 = k2 + 20k + 100− 248 = (k + 10)2 − 248

Fazendo k + 10 = x, tem-se:

x2 − 248 = y2, x, y ∈ N =⇒

x2 − y2 = 248 =⇒ (x+ y)(x− y) = 23 × 31

204 J. Bartasevicius

Page 205: bartasevicius.com.brbartasevicius.com.br/sites/default/files/2018-12/... · Sobre este trabalho. E permitido: • Compartilhar - copiar e redistribuir o material em qualquer meio

CAPITULO 2. SOLUCOES

Quanto maior o valor de x, maior o valor de k. Quanto maior o valorde k, maior o valor de n.Ha mais de uma possibilidade para x−y e x+y em (x+y)(x−y) = 23×31,mas a que produz o maior valor de x e aquela cujos fatores possuam amesma paridade, para que x, y ∈ N; e cuja distancia entre os fatores de248 seja maxima.Dentre as hipoteses de:

• x−y = 1∧x+y = 248 =⇒ fatores nao possuem mesma paridade.

• x− y = 2 ∧ x+ y = 124.Distancia entre fatores: 124− 2 = 122.

• x− y = 4 ∧ x+ y = 62.Distancia entre fatores: 62− 4 = 58.

• x− y = 8∧ x+ y = 31 =⇒ fatores nao possuem mesma paridade.

Resolvendo

x− y = 2

x+ y = 124=⇒

x = 63

y = 61

x = 63 =⇒ k = 53.

Entao, n =53±

√532 + 20× 53− 148

2=⇒

n = 57 ou

n = −4 6∈ N∴ a soma dos algarismos de n e 5 + 7 = 12.Alternativa d).

Solucao 198Simplificando a expressao:

16a4b2 − 8a3b3 + a2b4 =(

4a2b− ab2)2

= [ab (4a− b)]2 =

= (ab)2 (4a− b)2

Resolvendo a primeira equacao do sistema:

1 +2

ab= 5 =⇒ ab =

1

2

Resolvendo a segunda equacao do sistema:

5−2b2 = 16a2−b2 =⇒ 5 = 16a2+b2 =⇒ 5 = 16a2−8ab+b2+8ab =⇒

J. Bartasevicius 205

Page 206: bartasevicius.com.brbartasevicius.com.br/sites/default/files/2018-12/... · Sobre este trabalho. E permitido: • Compartilhar - copiar e redistribuir o material em qualquer meio

CAPITULO 2. SOLUCOES

Substituindo ab:

5 = (4a− b)2 + 8

(1

2

)=⇒ (4a− b)2 = 5− 4 =⇒

(4a− b)2 = 1

Aplicando na expressao:

(ab)2 (4a− b)2 =

(1

2

)2

1 =1

4

Alternativa e).

Solucao 199

√x+ 4 = 5−

√x− 1 =⇒

x+ 4 = 25− 10√x− 1 + x− 1 =⇒

√x− 1 = 2 =⇒ x− 1 = 4

∴ x = 5

Alternativa d).

Solucao 200As contas sao muito grandes para serem feitas manualmente.Fazendo n = 2014 e substituindo na equacao, o calculo se torna simples:

nx2 − (n+ 1)x− (2n+ 1) = 0

n =n+ 1±

√[− (n+ 1)]2 − 4n [− (2n+ 1)]

2n=⇒

n =

n+ 1 + 3n− 1

2n= 2 ou

n+ 1− 3n+ 1

2n=−n+ 1

n

206 J. Bartasevicius

Page 207: bartasevicius.com.brbartasevicius.com.br/sites/default/files/2018-12/... · Sobre este trabalho. E permitido: • Compartilhar - copiar e redistribuir o material em qualquer meio

CAPITULO 2. SOLUCOES

A solucao inteira e 2.a

b=−n+ 1

n.

Substituindo n por 2014:

a

b=−2014 + 1

2014= −2013

2014∴

a = 2013 e

b = 2014

a+ b = 2013 + 2014 = 4027, cuja soma dos algarismos e 13.Alternativa d).

J. Bartasevicius 207